Download as pdf or txt
Download as pdf or txt
You are on page 1of 88

CURRENT AFFAIRS

JULY - 2022

for

All Competitive Exams

Vani Nikethan Building, Kaleswararao Road, Suryaraopet.

VIJAYAWADA - 2. Ph. : (0866) 2438383, 2441177, 9392668899


www.sreedharscce.com & www.sreedharscce.org
INDEX

S.No. Topics Page No.

1. APPOINTMENTS 1
2. AWARDS & HONOURS 9
3. BANKING 13
4. BOOKS & AUTHORS 21
5. OBITUARIES 23
6. ECONOMY 26
7. INTERNATIONAL 37
8. NATIONAL 40
9. PERSONS IN NEWS 50
10. PLACES IN NEWS 53
11. SCIENCE & TECHNOLOGY 57
12. SEMINARS & MEETINGS 60
13. SPECIAL DAYS 63
14. SPORTS 67
15. STATE 75
16. SURVEYS 84


Sreedhar’s C C E CURRENT AFFAIRS -JULY-2022

1. APPOINTMENTS
1. Who has taken oath as the new CM of Maharashtra?
1) Nitish Kumar 2) Pema Khandu 3) Bhagwant Mann
4) Eknath Shinde 5) Himanta Biswa Sarma
[Explanation: Shiv Sena leader Eknath Shinde took oath as the 20th new chief minister
of Maharashtra. Devendra Fadnavis was the former Chief Minister.Governor: Bhagat Singh
Koshyari. Maharashtra is the third-largest state by area in India.]
2. Who has been appointed as England’s new white-ball captain?
1) Eoin Morgan 2) Joe Root 3) Jos Buttler 4) Ben Stokes 5) Matthew Potts

r’s
[Explanation: Jos Buttler has been appointed England’s new white-ball captain following
Eoin Morgan’s retirement from international cricket. The wicketkeeper-batter looked the
obvious candidate having served as Morgan’s vice-captain and established himself as one
of the most accomplished white-ball players in the world.]
3. Who Among the following has Become the 14th Prime Minister of Israel?
1) Lee Hsien Loong 2) Hun Sen 3) Lotay Tshering
4) Jacinda Ardern
ha 5) Yair Lapid
[Explanation: Yair (born 5 November 1963) is an Israeli politician and former journalist
who is the chairman of the centrist Yesh Atid party, and became the 14th prime minister of
Israel on 1 July 2022. He previously served as the alternate prime minister of Israel and

E
minister of Foreign Affairs from 2021 to 2022. Israel Former PM: Naftali Bennett; Israel
Capital: Jerusalem; Currency: Shekel.]
d
4. Who has been appointed as the new Deputy Chairman of Cochin Port Authority?
1) Vini Mahajan 2) Vikas Narwal 3) M. Beena
4) K.Rajendran 5) Bharat Lal
ee

[Explanation: The Ministry of Ports, Shipping and Waterways has appointed Vikas
Narwal, a Madhya Pradesh cadre Indian Administrative Service (IAS) officer, as the Deputy
Chairman of Cochin Port Authority for a five-year term.]
5. Who has been elected as Speaker of Maharashtra Legislative Assembly?
C
1) Sandeep Kumar Gupta 2) Rahul Narvekar
3) Rajan Salvi 4) Nitin Gupta 5) Tapan Kumar
Sr

[Explanation: Bharatiya Janata Party (BJP) MLA Rahul Narvekar was elected the Speaker
of the Maharastra Legislative Assembly. Mr. Narvekar, who polled 164 votes, defeated
Opposition candidate Rajan Salvi who got 107 votes in the elections. Mr. Narvekar, 45, is
the youngest ever Assembly Speaker in the country.]
6. Who has been appointed Air Officer Commanding-in-Chief (AOC-in-C) Central Air
Command (CAC)?
1) AP Singh 2) Seema Kushwaha 3) Manoj Pande
4) Dalbir Singh 5) Karambir Singh
[Explanation: Air Marshal AP Singh assumed the appointment of Air Officer Commanding-
C
in-Chief (AOC-in-C) Central Air Command. The Air Marshal was commissioned into the
fighter stream of IAF on 21 December 1984.]
7. Who has been appointed as the president of the Association of Indian Universities
(AIU)?
1) G Thiruvasagam 2) Suranjan Das 3) Sandeep Sancheti
4) Dr Tej Partap 5) M M Salunkhe
[Explanation: Jadavpur University Vice-Chancellor Suranjan Das was appointed the
president of the Association of Indian Universities (AIU). Former AIU president G
Thiruvasagam. Association of Indian Universities Secretary General: Dr. (Mrs.) Pankaj
Mittal;Association of Indian Universities Formation: 1925 as Inter-University Board.]
8. Who among the following has been appointed as the new chief secretary of Punjab?
1) K Shanmugam 2) Vijay Kumar Janjua 3) Anirudh Tewari
4) Somesh Kumar 5) S.C Gupta
[Explanation:Punjab Government led by the CM BhagwantMann appointed Vijay Kumar
Janjua as the new Chief Secretary of the State, replacing Anirudh Tewari.V.K. Janjua is a
1989-batch IAS officer and is currently holding the position of the Special Chief Secretary
Jails and in addition Special Chief Secretary Elections.]

JULY- 1 Visitus:www.sr eedharscce.com


Sreedhar’s C C E CURRENT AFFAIRS -JULY-2022
9. Who has been appointed as the Force Commander of UN Mission in South Sudan?
1) Lt Gen Amardeep singh bhinder 2) Lt Gen Shantanu Dayal
3) Lt Gen Subramanian
4) Lt Gen Baggavalli Somashekar Raju 5) Lt Gen MM Narvane
[Explanation: United Nations Secretary-General António Guterres announced the
appointment of Lt Gen Mohan Subramanian of India as Force Commander of the United
Nations Mission in South Sudan (UNMISS). He succeeds Lt Gen Shailesh Tinaikar also
from India.]
10. Who among the following has been named as next vice-chancellor of Rabindra
Bharati University?
1) Jagdeep Dhankhar 2) Bimal Prasad Singh 3) Mahua Mukherjee

r’s
4) Sitarama Rao 5) Ch. Gopal Reddy
[Explanation: West Bengal Governor Jagdeep Dhankhar has appointed the next vice
chancellor of the Rabindra Bharati University. Dr Mahua Mukherjee, a professor in the
university’s department of dance, will replace Sabyasachi Basu Roy as the chancellor.]
11. Who has been sworn into the Supreme Court as the first Black woman on the
nation’s highest court? ha
1) Imani Nova 2) Jada Serenity 3) Ketanji Brown Jackson
4) Clarence 5) Aaliyah
[Explanation: Ketanji Brown Jackson has been sworn into the Supreme Court, shattering
a glass ceiling as the first Black woman judge on the nation’s highest court.]
12.
1) Campbell Wilson
4) Amish Mehta

E
Who has been appointed as the new CEO of Aviva India?
2) Parameswaran Iyer 3) Shahid Kapoor
5) Asit Rath
d
[Explanation: Aviva India has appointed Asit Rath as chief executive officer and managing
director. Rath will succeed Amit Malik who is leaving the business after 10 years. Aviva
India has appointed Asit Rath as chief executive officer and managing director.]
ee

13. Who has been assigned additional charge of the Ministry of Minority Affairs?
1) Smriti Irani 2) Jyotiraditya Scindia 3) Giriraj Singh
4) Dharmendra Pradhan 5) Narendra Modi
[Explanation: Union minister Smriti Irani has taken charge of the Ministry of Minority
C
Affairs. Former Ministry of Minarity affairs was Mukhtar Abbas Naqvi resigned from the
Cabinet.]
Sr

14. Jyotiraditya Scindia has been assigned as an additional charge of the Ministry of
Steel. He replaced__________.
1) Dharmendra Pradhan 2) Kiren Rijiju
3) Mukhtar Abbas Naqvi
4) Ram Chandra Prasad Singh 5) Hardeep Singh Puri
[Explanation: Union Minister Jyotiraditya Scindia will be assigned the Ministry of Steel
after the resignation of Union Minister Ram Chandra Prasad Singh.]
15. Who has appointed as President Commissioner for Unilever Indonesia?
C
1) Sudhir Sitapati 2) Harshavard Neotia 3) Sanjiv Mehta
4) Nitin Paranjpe 5) Dev Bajpai
[Explanation: Sanjiv Mehta, CEO and managing director of packaged consumer goods
company Hindustan Unilever is set to be appointed as the president commissioner (non-
executive chairman) of Unilever Indonesia.]
16. Who has been elected as the President Designate of CII for 2022-23?
1) T V Narendran 2) Rakesh Bharti Mittal 3) R. Dinesh
4) Ajay S 5) Adi Godrej
[Explanation: ITC chairman and managing director Sanjiv Puri has been elected the Vice
President of CII for 2022-23, while TVS Supply Chain Solutions Executive Vice Chairman
R Dinesh has become the new President Designate.]
17. Who has been appointed as the new G20 Sherpa, in 2022?
1) Rajiv Kumar 2) Vinod Kumar Paul 3) Amitabh Kant
4) Suman Bery 5) Parameswaran Iyer
[Explanation: Amitabh Kant has been appointed, as India is scheduled to assume the G-
20 presidency in 2022. He will replace Piyush Goyal as G-20 Sherpa. He was appointed
as G-20 Sherpa in September 2021. He is a retired 1980-batch IAS officer from Kerala
cadre. He assumed charges as NITI Aayog CEO in 2016.]

JULY- 2 Visitus:www.sr eedharscce.com


Sreedhar’s C C E CURRENT AFFAIRS -JULY-2022
18. Who has been appointed as the seventh President of the International Fund for
Agricultural Development (IFAD)?
1) Kanayo F. Nwanze. 2) Ajay Devgn 3) Gilbert Houngbo
4) Alia Bhatt 5) Alvaro Lario
[Explanation: The governing council of the International Fund for Agricultural Development
(IFAD) has been appointed as Spain’s Alvaro Lario new President. Lario will take office on
1 October 2022 and serve a four-year term. He will succeed Gilbert Houngbo who has led
the organisation since 2017. International Fund for Agricultural Development Founded –
1977 December, Rome, Italy.]
19. Who has been appointed as the deputy Election commissioner?
1) T Sreekanth 2) Umesh Sinha 3) RK Gupta

r’s
4) Ashok Lasava 5) Sunil Arora
[Explanation: Senior bureaucrat R K Gupta was appointed as the deputy election
commissioner, an order issued by the Personnel Ministry. He comes in place of T
Sreekanth. Gupta, a Central Secretariat Service (CSS) officer, will serve as deputy election
commissioner (Joint secretary level) till 28 February next year, the date of his
superannuation. Chief Election Commissioner of India- Rajiv Kumar; Election Commissioner-

20.
Anup Chandra Pandey.]
ha
Who has been elected as the new President of Market Research Society of India
(MRSI)?
1) Droupadi Murmu 2) Ramesh Bais 3) Arif Moham Khan
4) Manish Makhijani

E 5) Snusuiya Uikey
[Explanation: Market Research Society of India (MRSI), the industry-led market research
d
body, has elected Manish Makhijani, Global Insights Director at Hindustan Unilever, as its
new president for the 2022-24 term.]
21. Who has been appointed as Chairman and Managing Director of Goa Shipyard
ee

Limited (GSL)?
1) Paritosh Tripathi 2) Asit Rath 3) Sandeep Kumar Gupta
4) Brajesh Kumar Upadhyay 5) Ben Silbermann
[Explanation: The Appointments Committee of the Cabinet (ACC) has approved the
C
appointment of Brajesh Kumar Upadhyay as Chairman & Managing Director (CMD) of Goa
Shipyard Limited (GSL), a PSU under the Ministry of Defence. Upadhyay has been
Sr

appointed to the post for a period of five years from the date of his assumption of charge of
the post, or until further orders, whichever is earlier. Goa Shipyard Limited Founded -
1957; Goa Shipyard Limited Headquarters - Goa.]
22. Who has been appointed as the new Chief Financial Officer of IKEA Organisation?
1) Sunil Agrawal 2) Dinesh Thapar 3) Amol Joshi
4) Sanjeev Churiwala 5) Murali Iyer
[Explanation: Mr. Iyer would be part of IKEA India leadership team reporting to Susanne
Pulverer, CEO & CSO (Chief Sustainability Officer). The new CFO has over two decades
C
of experiences in financial leadership, having worked with the Volvo Group in India and
Sweden in multiple roles.]
23. Who has taken over as NITI Aayog’s as the new Chief Executive Officer?
1) Gaurav Kapoor 2) Vinayak Pai 3) Vijayalakshmi Swaminathan
4) Ajay Mahajan 5) Parameswaran Iyer
[Explanation:AFTER A six-year-long stint, NITI Aayog CEO Amitabh Kant is set to leave
the government policy think tank at the end of June. He will be replaced by former Secretary
of the Ministry of Drinking Water and Sanitation Parameswaran Iyer, who spearheaded
the government’s Swachh Bharat Mission.He also worked with the erstwhile Mayawati
government in Uttar Pradesh in the field of education.]
24. RBI has approved the appointed of whom as part-time Chairman of Karur Vysya
Bank for 3 years?
1) Ramesh Babu Boddu 2) Shaktikanta Das 3) Osborne Smith
4) Dinesh Kumar Khara 5) Meena Hemchandra
[Explanation: The Reserve Bank of India (RBI) has approved the appointment of former
RBI executive Meena Hemchandra as part-time chairman of the Private sector lender Karur
Vysya Bank for three years. RBI Governor- Shaktikanta Das.]

JULY- 3 Visitus:www.sr eedharscce.com


Sreedhar’s C C E CURRENT AFFAIRS -JULY-2022
25. Who has been appointed as MD and CEO of SBI General Insurance?
1) Manoj Kumar 2) Vineet Saran 3) Suranjan Das
4) R.Dinesh 5) Paritosh Tripathi
[Explanation: Private non-life insurer SBI General Insurance had appointed Paritosh
Tripathi as managing director and chief executive officer. He succeeds P.C. Kandpal, who
has been posted as Deputy Managing Director (DMD)-(P&RE) at the Corporate Center,
State Bank of India (SBI).]
26. Who has been appointed as the MD of National Bank for Financing Infrastructure
and Development (NaBFID)?

r’s
1) Rakesh Makhija 2) Meena Hemchandra 3) Amitabh Chaudhry
4) Sunit Malhotra 5) G Rajkiran Rai
[Explanation: Financial Services Institutions Bureau (FSIB) has recommended former
Union Bank of India NSE 1.33 % managing director Rajkiran Rai G as the head of the
newly set-up Rs 20,000-crore NaBFID.]
ha
27. Who has been appointed as the board of Google parent Alphabet Inc?
1) Sergey Brin 2) Marty Chavez 3) Larry Page
4) Sundar Pichai 5) Mark Zuckerberg
[Explanation: Google Parent Alphabet appoints Goldman Sachs Veteran, Marty Chavez

E
to Board. Adding significant finance muscle to the technology giant. Alphabet Inc. CEO:
Sundar Pichai;Alphabet Inc. Chairperson: John L. Hennessy; Founded: 2 October 2015,
d
California, United States; Founders: Larry Page, Sergey Brin.]
28. Who has been appointed as the Director General of the Federation of Indian
ee

Fantasy Sports (FIFS)?


1) Bimal Julka 2) Anwar Shirpurwala 3) Mitali Gupta
4) Virat Kohli 5) Joy Bhattacharjya
[Explanation: Federation of Indian Fantasy Sports names Joy Bhattacharjya as Director-
C
General. The Federation of Indian Fantasy Sports (FIFS), the country’s first and only self-
regulatory industry body for fantasy sports, has announced the appointment of Joy
Sr

Bhattacharjya as its Director-General.]


29. Who has been appointed as the next MD&CEO of NSE?
1) Ashish kumar Chauhan 2) Vikram Limaye 3) Chitra Ramkrishna
4) Ravi Narain 5) Anand Subramanian
[Explanation: The National Stock Exchange (NSE) has announced the appointment of
Ashish Kumar Chauhan as its new managing director and CEO. He will succeed Vikram
Limaye whose 5-year term ended on 16 July 2022. He was one of the founders of NSE
C
where he worked from 1992 to 2000. NSB Location: Mumbai, Maharashtra; NSB Founded:
1992; Chairperson of NSB: Girish Chandra Chaturvedi.]
30. Who has been appointed as non-independent non-executive director of IndiGo
board?
1) M Damodaran 2) Venkataramani Sumantran 3) Rono Dutta
4) Rahul Bhatia 5) Rakesh Gangwal
[Explanation: InterGlobe Aviation, the parent company of IndiGo airline,its board has
approved the appointment of former SEBI Chairman M Damodaran as non-independent
non-executive director.]
31. Manipur Governor La Ganesan has given additional charge to ________.
1) Tripura 2) Nagaland 3) Chhattisgarh 4) Mizoram 5) West Bengal
[Explanation: Manipur Governor La Ganesan has been given additional charge of West
Bengal after incumbent Jagdeep Dhankhar resigned. Dhankhar had resigned as he is
contesting the elections to the post of vice president of India as an NDA candidate.]

JULY- 4 Visitus:www.sr eedharscce.com


Sreedhar’s C C E CURRENT AFFAIRS -JULY-2022
32. Who has become the new chief secretary of Himachal Pradesh?
1) R D Dhiman 2) Ram Subhag Singh 3) Nishan Singh
4) Sanjay Gupta 5) Anil Khachi
[Explanation: Shimla’s IAS officer R D Dhiman has appointed as the new Chief Secretary
of Himachal Pradesh.The 1988-batch IAS officer replaces Ram Subhag Singh, who has
been appointed as the Principal Advisor (Administrative Reforms).]
33. Who has been appointed as the Chairman of Khadi and Village Industries
Commission (KVIC)?
1) Manoj Kumar 2) Ashish Kumar Chauhan
3) M Damodaran 4) R D Dhiman 5) Vinay Kumar Saxena

r’s
[Explanation: Manoj Kumar has been promoted to take charge as Chairman of the
Government of India’s statutory body. Manoj Kumar, former expert member of marketing
at Khadi and Village Industries Commission (KVIC), has been promoted to take charge as
Chairman of the Government of India’s statutory body.]
34. Who has been appointed as a wholetime member of the Insolvency and Bankruptcy
Board of India (IBBI)?
1) Navrang Saini
4) Sudhaker Shukla
ha 2) Jayanti Prasad
5) Ritesh Kavdia
3) Ravi Mital

[Explanation: The Central Government has appointed, Jayanti Prasad as a Whole-time

E
Member of the Insolvency and Bankruptcy Board of India (IBBI) for five years. The Central
Government has appointed, Jayanti Prasad as a Whole-time Member of the Insolvency
d
and Bankruptcy Board of India (IBBI) for five years.]
35. Who has been appointed as a member of the Union Public Service Commission
ee

(UPSC)?
1) Raj Shukla 2) Manoj Soni 3) Rajam Malhotra
4) Jayanti Prasad 5) Arvind Saxena
[Explanation: Retired Army officer Raj Shukla has been appointed as a member of the
C
Union Public Service Commission (UPSC). The UPSC recommends candidates to the
government for appointment to the Indian Administrative Service (IAS), Indian Foreign Service
Sr

(IFS), Indian Police Service (IPS) and the Central Services — Group A and Group B. UPSC
Chairperson: Manoj Soni; UPSC Founded: 1 October 1926.]
36. Which State Govt. has appointed Deloitte India as consultant to achieve the target
of making the state a $1-trillion economy?
1) Gujarat 2) Maharashtra 3) Uttar Pradesh
4) Tamil Nadu 5) Rajasthan
[Explanation: The Uttar Pradesh cabinet appointed Deloitte Touche Tohmatsu India as
C
consultant to achieve the target of making the state a $1-trillion economy. The consultant
quoted a bid price of Rs 120 crore.]
37. Who has been appointed as the BCCI ombudsman and ethics officer?
1) Narinder Batra 2) Vineet Saran 3) Anil Khanna
4) Sanjay Agrawal 5) Manoj Kumar
[Explanation: Former Supreme Court judge Vineet Saran has taken over as the BCCI’s
ethics officer and ombudsman, filling the dual posts lying vacant for a year. Saran has
succeeded Justice (Retd) D K Jain, whose term ended in June last year.]
38. Who has been appointed as the MD of Rattan India Power?
1) Brijesh Gupta 2) Rajiv Rattan 3) Vineet Saran
4) Sanjay Agrawal 5) Manoj Kumar
[Explanation: RattanIndia Power announced hiring Brijesh Gupta as its new managing
director. He has experience working across the renewable, steel, mining and commodities
sector in India and abroad. He has over three decades of experience in the industrial
sector.]

JULY- 5 Visitus:www.sr eedharscce.com


Sreedhar’s C C E CURRENT AFFAIRS -JULY-2022
39. Who has been appointed as the acting president of FIH?
1) Sourav Ganguly 2) Rupinder Pal Singh 3) Manpreet Singh
4) Seif Ahmed 5) Anil Khanna
[Explanation: The International Hockey Federation (FIH) has appointed Egypt’s Seif
Ahmed as its acting president following Indian administrator Narinder Batra’s resignation
from the post. Batra resigned as FIH president and also quit as Indian Olympic Association
(IOA) chief. FIH Headquarters: Lausanne, Switzerland; FIH CEO: Thierry Weil; FIH
Founded: 7 January 1924, Paris, France.]
40. Who has been appointed as managing director of ONGC Videsh?
1) Vineet Saran 2) Jayanti Prasad 3) Rajarshi Gupta

r’s
4) Raj Shukla 5) Ashish Kumar Chauhan
[Explanation: Rajarshi Gupta has been appointed as Managing Director of ONGC Videsh.
Rajarshi Gupta has been appointed as Managing Director of ONGC Videsh. He was
recommended for the position by the Public Enterprise Selection Board (PESB).]
41. Who has been elected as 9th President of Sri Lanka in Parliament vote?

3) Ranil Wickremesinghe
ha
1) Anura Kumara Dissanayake

4) Mahinda Yapa Abeywardena


2) Gotabaya Rajapaksa

5) Maithripala Sirisena
[Explanation: Ranil Wickremesinghe won the election with 134 votes and was elected

E
as the ninth President (eighth executive President) of Sri Lanka. The election, which was
held amid a political crisis, was the first time in the history of Sri Lanka that a vote took
d
place in Parliament to elect a President. Sri Lanka Capitals: Colombo, Sri Jayawardenepura
Kotte; Sri Lanka Currency: Sri Lankan rupee.]
ee

42. Who has been appointed as the next CEO of Vodafone Idea Ltd?
1) Ravinder Takkar 2) Balesh Sharma 3) Akshay Moondra
4) Nick Read 5) Gopal Vittal
[Explanation: Debt-ridden telecom operator Vodafone Idea has announced the
C
appointment of it’s chief financial officer Akshaya Moondra as the new CEO for a period of
three years effective August 19, 2022. Moondra will succeed managing director Ravinder
Sr

Takkar in the role.]


43. IAPH has appointed whom as its representative in India?
1) Ennarasu Karunesan 2) Patrick Verhoeven
3) Shashi Verma 4) KP Kumaran 5) Seif Ahmad
[Explanation: Ennarasu Karunesan has been appointed as the International Association
of Ports and Harbors’ (IAPH) official representative in India. He currently serves as an
advisor to Indian Ports. It is headquartered in Tokyo, Japan. It was established in 1955
C
and is now acknowledged as the NGO that represents ports globally. Managing director
of the IAPH: Patrick Verhoeven.]
44. Who has been appointed as Managing Director of TATA Projects?
1) Vinayak Pai 2) Vinayak Deshpande
3) Sowmyan Ramakrishnan 4) N Chandrasekaran 5) Marc Llistosella
[Explanation: Vinayak Pai has been named as the managing director of TATA Projects.
Vinayak Deshpande had been replaced by Pai. Vinayak Pai has been named as the
managing director of TATA Projects.]
45. Who has been appointed as the CEO of Paytm Payments Services Ltd (PPSL)?
1) Praveen Sharma 2) Nakul Jain 3) Vinayak Pai
4) Akshaya Moondra 5) Jayanti Prasad
[Explanation: Banking Veteran Nakul Jain Joins As CEO Of Paytm Payments Services.
Listed fintech unicorn Paytm’s parent One97 Communications has appointed Nakul Jain
as the CEO of Paytm Payments Services Ltd (PPSL). MD and CEO of Paytm: Vijay Shekhar
Sharma; Paytm Founded: August 2010; Paytm Headquarters: Noida, Uttar Pradesh, India.]

JULY- 6 Visitus:www.sr eedharscce.com


Sreedhar’s C C E CURRENT AFFAIRS -JULY-2022
46. Sheikh Ahmad Nawaf Al Ahmed Al-Sabah has been appointed as the prime minister
of ___________.
1) Iran 2) Iraq 3) Oman 4) Kuwait 5) Saudi Arabia
[Explanation: Kuwaiti Emir has appointed Ahmad Nawaf Al-Ahmad Al-Sabah as the new
Prime Minister of the country. Sheikh Ahmad Al Nawaf has been the First Deputy Prime
Minister and Minister of Interior of Kuwait since March 2022.Kuwait Capital: Kuwait City;
and Currency: Kuwaiti dinar.]
47. Which bank has appointed RS Gandhi as additional independent director?
1) HDFC Bank 2) ICICI Bank 3) YES Bank 4) Axis Bank 5) RBL Bank
[Explanation: YES Bank has appointed RS Gandhi as additional independent director.
The appointment of Rama Subramaniam Gandhi as an additional independent director of

r’s
YES Bank for the period of five years.]
48. Who has been appointed as chief economist of World Bank?
1) Carmen Reinhart 2) Kaushik Basu 3) Indermit Gill
4) Raghuram Rajan 5) Gita Gopinath
[Explanation: The World Bank has appointed Indermit Gill as the chief economist and
senior vice president for development economics. Gill will be the second Indian, after
ha
Kaushik Basu, to become the chief economist of the international financial institution that
grants loan to middle and lower-income countries.]
49. Who has been appointed as CEO of Indian Deaf Cricket Association?
1) Roma Balwani 2) Sumit Jain 3) Ajay Kumar
4) Imran Sheikh

E 5) Mahantesh
[Explanation: Roma Balwani takes charge as CEO of India Deaf Cricket Association.
Indian Deaf Cricket Association (IDCA), the governing body for deaf cricket in India, has
d
appointed Roma Balwani as its CEO and Brand Custodian effective from July 2022.
President of IDCA: Sumit Jain.]
50. Who has been elected as the new President of Albania?
ee

1) Dinesh Gunawardena 2) Mario Draghi 3) Bajram Begaj


4) Anthony Albanese 5) Ranil Wickremesinghe
[Explanation: Albania’s parliament has elected a top military official, General Major Bajram
Begaj as its new president.Albania Capital: Tirana; Albania Currency: Albanian lek; Albania
C
Prime Minister: Edi Rama.]
51. Who has been appointed as the Indian women’s gymnastics coach for
Sr

Commonwealth Games 2022?


1) Bisheshwar Nandi 2) Rohit Jaiswal 3) Anil Khanna
4) Narinder Batra 5) Seif Ahmed
[Explanation: Bisheshwar Nandi has been appointed as the coach of the women’s
gymnastics team for the Commonwealth Games (CWG) 2022. Gymnastics coach Bisheshwar
Nandi, who guided Dipa Karmakar to a historic fourth place finish at the 2016 Rio Olympics,
has been named coach of the Indian women’s team for the Commonwealth Games in
Birmingham in place of controversial Rohit Jaiswal.]
C
52. Who has been appointed as the CEO of Air India?
1) Campbell Wilson 2) N Chandrasekaran 3) V Sumantran
4) Rono Dutta 5) Pieter Elbers
[Explanation: Mr Campbell Wilson’s appointment as the Chief Executive Officer (CEO) &
Managing Director (MD) of Air India. He has 26 years aviation industry expertise across
both full service and low-cost airlines. He is the chief executive of Scoot, a subsidiary of
Singapore Airlines.]
53. Who has been appointed as the interim CEO of the National Land Management
Corporation (NLMC)?
1) Nakul Jain 2) Vinayak Pai 3) Seif Ahmed
4) Sanjay Kumar Jain 5) Ashish Kumar Chauhan
[Explanation: The Government has appointed a joint secretary to the Department of Public
Enterprises (DPE) Sanjay Kumar Jain, as the interim chief executive officer (CEO) of NLMC.
National Land Management Corporation (NLMC) was incorporated on June 3 as a wholly-
owned Government of India company with an initial authorised share capital of Rs 5,000
crore and paid-up share capital of Rs 150 crore.]

JULY- 7 Visitus:www.sr eedharscce.com


Sreedhar’s C C E CURRENT AFFAIRS -JULY-2022

54. Pranay Kumar Verma named as new High Commissioner of India to _______.
1) Japan 2) Russia 3) Bangladesh 4) China 5) Sri Lanka
[Explanation: Pranay Kumar Verma appointed India’s next High Commissioner to
Bangladesh. A seasoned diplomat and IFS officer of 1994 batch Pranay Kumar Verma
has been appointed as India’s next High Commissioner to Bangladesh. He is currently
serving as India’s Ambassador to Vietnam.]
55. Droupadi Murmu had served as the 9th Governor of which state between 2015
and 2021?
1) Jharkhand 2) Maharashtra 3) Odisha

r’s
4) Gujarat 5) Meghalaya
[Explanation: She is the first person belonging to the indigenous, scheduled tribe
community (Santal) to be elected as the President of India. Prior to her presidency she
served as the ninth Governor of Jharkhand between 2015 and 2021, and held various
ha
portfolios in the cabinet of Government of Odisha between 2000 to 2004.]
56. Who has been elected as the 15th President of India?
1) Droupadi Murmu 2) Yeshwant Sinha 3) Ram Nath Kovind
4) Mahesh Jethmalani 5) Ranjan Gogoi

E
[Explanation: Former Jharkhand Governor and National Democratic Alliance (NDA)
d
candidate Droupadi Murmu was elected the 15 th President of India, the first tribal woman
to be elected to the position and the youngest as well.]
57. Who has sworn as the Sri Lanka’s new prime minister?
ee

1) Dinesh Gunawardena 2) D.M. Jayaratne 3) Mahinda Rajapaksa


4) Ratnasiri Wickremanayake 5) Chandrika Kumaratunga
[Explanation: Dinesh Gunawardena took oath as Sri Lanka’s new prime minister. Former
C
PM: Mahinda Rajapaksa; Sri Lanka President: Ranil Wickremesinghe.]
Sr

58. Who has been appointed as UK Prime Minister?


1) Dominic Raab 2) Liz Truss 3) Priti Patel
4) Jeremy Hunt 5) Rishi Sunak
[Explanation: Liz Truss is holding a commanding lead over Rishi Sunak in the race to
become the next UK prime minister. If Truss wins, she would become the third woman
British prime minister after Margaret Thatcher and Theresa May.UK Captial: London; UK
Currency: Pound sterling.]
C
59. Which former Indian cricketer has been appointed to ICC Cricket Committee?
1) Sachin Tendulkar 2) V.V.S. Laxman 3) Virender Sehwag
4) Anil Kumble 5) Sourav Ganguly
[Explanation:“Danie l Vettori and VVS Laxman have also been appointed to the ICC Men’s
Cricket Committee as current player representatives, while Roger Harper has been
appointed as the second past player representative, joining Sri Lanka’s Mahela
Jayawardena,” stated ICC in an official statement.]
60. Who has been appointed as Additional Director of LIC HFL Board?
1) Ravi Kishan Takkar 2) Nakul Jain 3) Akshya Moondra
4) Manoj Kumar 5) Narinder Batra
[Explanation: The Board of Directors of LIC Housing Finance Ltd (LIC HFL) has appointed
Ravi Kishan Takkar, former MD & CEO of UCO Bank, as an Additional Director
(Independent) on the company’s board.]

JULY- 8 Visitus:www.sr eedharscce.com


Sreedhar’s C C E CURRENT AFFAIRS -JULY-2022

2.AWARDS AND HONOURS


1. Who has been conferred with CII Quality Ratna Award 2021?
1) Ashok Soota 2) Suresh Krishna 3) Ramdarash Mishra
4) VasdevMohi 5) Ram Vilas Sharma
[Explanation: The CII Awards Committee unanimously concurred to bestow the 2021
Award to Mr. Ashok Soota considering his significant contribution to building the
competitiveness of the Indian industry through Quality Initiatives.]
2. Who has received the coveted ‘Shiromani Award’ at NRI World Summit, UK?
1) Amrita Murali 2) Michelle Poonawalla 3) VittalRangan

r’s
4) Venkatanagarajan 5) AparnaSatheesan
[Explantion:Michelle Poonawalla receives the coveted Shiromani Award at NRI World
Summit 2022, United Kingdom. Michelle Poonawalla, well known artist, businesswoman
and philanthropist, from the renowned Poonawalla family received the coveted Shiromani
Award from the hands of Mr. G. P.Hinduja, the co-chairman of Hinduja Group at NRI
ha
World Summit 2022.]
3. Who has been awarded with Ayurveda Ratna Award 2022?
1) Ashok Soota 2) Ramdarash Mishra 3) TanujaNesari
4) SharankumarLimbale 5) Siddamshetty Mahesh kumar

E
[Explanation: TanujaNesari, Director of All India Institute of Ayurveda (AIIA) was awarded
with the Ayurveda Ratna Award by the UK Parliament. The UK’s All-Party Parliamentary
d
Group on Indian Traditional Sciences (ITSappg) conferred the award recognising her
contribution to prmoting the growth of Ayurveda in India and abroad.]
4. Who among the following has won the F1 British Grand Prix 2022?
ee

1) Charles Leclerc 2) Carlos Sainz Jr. 3) Sergio Pérez


4) Lewis Hamilton 5) Max Verstappen
[Explanation: Ferrari’s Carlos Sainz registered his first Formula One win at British Grand
Prix 2022, with the Spaniard finishing ahead of the Red Bull driver Sergio Perez and
C
Mercedes’ Lewis Hamilton. Carlos Sainz claimed his first Formula One victory in his 150th
race.]
Sr

5. Who among the following has won Femina Miss India World 2022?
1) Sini Shetty 2) RubalShekhawat 3) RitikaKhatnani
4) Manasa Varanasi 5) Shinata Chauhan
[Explanation: Sini Shetty from Karnataka was announced the winner of Femina Miss
India World 2022 title at the grand finale of VLCC Femina Miss India. At the ceremony,
hosted at the Jio World Convention Centre, Rajasthan’s RubalShekhawat emerged as
Femina Miss India 2022 first runner-up and Shinata Chauhan from Uttar Pradesh was
named Femina Miss India 2022 second runner-up.]
C
6. Which of the following has bagged the Microsoft partner of the Year award 2022?
1) Tech Mahindra 2) Accenture 3) HCL Technologies
4) Infosys 5) Capgemini
[Explanation: HCL Technologies was recognised at Microsoft Partner of the Year Awards
2022. HCL Technologies was recognised at Microsoft Partner of the Year Awards 2022,
for innovation and implementing customer solutions based on Microsoft technology.]
7. India’s Pallavi Singh has bagged the Mrs Universe Divine Crown in South Korea,
she is from __________.
1) Punjab 2) Telangana 3) Mumbai 4) Uttar Pradesh 5) Haryana
[Explanation: India’s Pallavi Singh has won the Mrs Universe Divine Title at the finals
held at Yeosu City, South Korea. She hails from Kanpur, India, and has made her country
proud in this contest that saw participation from 110 countries. This is a proud moment for
India. Pallavi Singh was the contestant from Asia in the Mrs Universe contest and has
demonstrated the strong will and commitment of Indian women to excel in all walks of life.
Pallavi Singh won the title of Mrs India held in Jaipur in 2020. She then won the title of Mrs
Indo-Asia Universe in the Asia-level competition held in Delhi in October 2021.]

JULY- 9 Visitus:www.sr eedharscce.com


Sreedhar’s C C E CURRENT AFFAIRS -JULY-2022
8. Who has bagged ‘young professional award’ award by Renewx?
1) Rajesh Shah 2) Arth Patel 3) VenkatRajaraman
4) Manish Tiwari 5) sanjay Patel
[Explanation:Arth Patel, Co-founder and CEO Tirex Chargers, bags young professional
award from Renewx.]
9. Who has announced the setting up of Dr Rajendra Prasad Memorial Award in
public administration in field of academic excellence?
1) Narendra Modi 2) Nitin Jairam Gadkari 3) Dharmendra Pradhan
4) Dr. Jitendra Singh 5) Rao Inderjit Singh
[Explanation:Science and Technology Minister Dr. Jitendra Singh in New Delhi. Dr. Singh

r’s
presided over a meeting of the Executive Council of the Indian Institute of Public
Administration (IIPA). To achieve the objectives established by Prime Minister Narendra
Modi, the institute must have a futuristic vision for the following 25 years.Minister of
Defence-Rajnath Singh; Minister of Science and Technology-Jitendra Singh.]
10. Who has become AATC’s Youngest Analog Astronaut?
1) Chris Hadfield 2) Musgrave 3) JahnaviDangeti
4) Yuri Gagarin
ha 5) John Glenn
[Explanation: 19-year-old JahnaviDangeti has scripted history by becoming the youngest
to complete the Analog Astronaut programme from the Analog Astronaut Training Centre
(AATC) at Krakow in South Poland. Ms Jahnavi had completed NASA’s International Air

E
and Space Programme (IASP) at the Kennedy Space Centra in the U.S. NASA administrator:
Bill Nelson; Headquarters of NASA: Washington D.C., United States; NASA Founded: 1
d
October 1958.]
11. Who has won first gold medal at ISSF World Cup in South Korea ?
1) Arjun Babuta 2) Lucas Kozeniesky 3) Sergey Richter
ee

4) PaarthMakhija 5) Sergey Richter


[Explanation: In Shooting, India’s Arjun Babuta have won his first gold medal at the
International Shooting Sport Federation (ISSF) World Cup stage at Changwon in South
Korea. He defeated Tokyo 2020 silver medallist Lucas Kozeniesky of the USA 17-9 in the
C
final.]
Sr

12. Who has won Japan’s ‘Order of the Rising Sun’ award?
1) Rajendra Prasad 2) Narayanan Kumar 3) Gita Gopinath
4) Sini Shetty 5) Afroz Shah
[Explanation: The government of Japan has conferred the ‘Order of the Rising Sun, Gold
and Silver Star’ award on Sanmar Group Vice Chairman Narayanan Kumar in recognition
of his contribution toward strengthening economic relations between Japan and India.
Kumar was honoured by the Consul General of Japan in Chennai, Taga Masayuki.]
13. Whom among the following have felicitated with the prestigious Mother Teresa
C
Memorial Awards for Social Justice 2021?
1) Ms Dia Mirza 2) Mr Afroz Shah 3) Susan Abraham
4) Both A&B 5) Both B&C
[Explanation: The United Nations Environment Program (UNEP) National Goodwill
Ambassador Ms Dia Mirza, and Environmental Activist Mr Afroz Shah were felicitated with
the prestigious Mother Teresa Memorial Awards for Social Justice 2021. The award is
given by the Governor of Maharashtra Hon. Bhagat Singh Koshyari at Raj Bhavan, Mumbai.
Both were awarded for their commendable and notable achievements in Environmental
Sustainability.]
14. Who among the following has won Paracin Open title 2022?
1) AlexandrPredke 2) AlisherSuleymenov 3) AL Muthaiah
4) R Praggnanandhaa 5) V Pranav
[Explanation: Young Indian Grandmaster R Praggnanandhaa won the Paracin Open ‘A’
chess tournament 2022. The 16-year-old remained unbeaten and finished half a point
ahead of the field. AlexandrPredke took the second spot with 7.5 points ahead of
AlisherSuleymenov and India’s AL Muthaiah, who both scored 7 points.]

JULY- 10 Visitus:www.sr eedharscce.com


Sreedhar’s C C E CURRENT AFFAIRS -JULY-2022
15. Which among the following has won Sardar Patel award of ICAR?
1) NAARM 2) IIT Delhi 3) IISC Bangalore
4) University of Delhi 5) Amity University
[Explanation: The National Academy of Agricultural Research Management (NAARM) has
bagged the Sardar Patel Outstanding ICAR Institute Award 2021 (in the Large Institute
Category) for its overall performance. Ch Srinivasa Rao, Director of NAARM, received the
award from Narendra Singh Tomar in New Delhi. The awards were given away at a function
to mark the 94th Foundation Day of the ICAR.]
16. Who has been honoured with Kerala’s highest film award, the JC Daniel Award?
1) Anil Khanna 2) Sanjay Agrawal 3) Vineet Saran
4) KP Kumaran 5) Narinder Batra

r’s
[Explanation: Malayalam filmmaker, KP Kumaran has been honoured with Kerala’s
highest film award, the JC Daniel Award. Kumaran won the award for his contribution to
the Malayalam film industry. The award comprises of a cash prize of Rs 5 lakh, a citation,
and a plaque. The 2021 jury included singer P Jayachandran, director SibiMalayil, film
academy chairman Ranjith, and Rani George, the principal secretary of the Department of
ha
Cultural Affairs.]
17. Which among the following has won the Osman Sagar Cup?
1) Gitanjali Rao 2) Emma Lou 3) Malala Yousafzai
4) Orion Jean 5) Mare Lagos

E
[Explanation: The five-year-old mare Lagos, ridden by SurajNarredu, won the Osman
Sagar Cup. The main event of the opening day Monsoon races to be held in Hyderabad.
d
SurajNarredu guided the D Netto-trained Lagos to victory in the Osman Sagar Cup 1200
metres.]
18. Who has won the Best Actor Award at 68th National Film Awards 2022?
ee

1) Ajay Devgn 2) Suriya 3) Biju Menon 4) Both A &B 5) All A, B and C


[Explanation: The winners of the 68th National Film Awards were announced in New
Delhi. This year’s awards ceremony honoured films from 2020 as well due to COVID-19-
related delays. Union Minister of Information and Broadcasting, Anurag Thakur announced
C
the award winners. SooraraiPottru won three of the four biggest awards of the night,
while TanhajiThe Unsung Warrior also took home major trophies. Best Actor: Suriya for
Sr

SooraraiPottru and Ajay Devgn for Tanhaji.]


19. Who has won the Global Energy Prize 2022?
1) Ashok Soota 2) Rashmi Sahoo 3) RJ Umar
4) Kaushik Rajashekara 5) A Gopalakrishnan
[Explanation: Kaushik Rajashekara, an Indian-origin professor of engineering at the
University of Houston, has won the prestigious Global Energy Prize for outstanding
contributions to transportation electrification and energy efficiency technologies while
reducing power generation emissions. Only three people in the world were selected this
C
year for the honour, bestowed by the Global Energy Association, out of a record 119
nominations from 43 countries.]
20. Which of the following film has bagged the Best Feature Film award in 68th
National Film Awards?
1) AyyappanumKoshiyum 2) Tanhaji 3) SooraraiPottru
4) Testimony of Anna 5) Kachichinithu
[Explanation:Tamil movie “SooraraiPottru” bagged national awards for the Best Feature
Film, Best Actor, Best Actress, Best Background Score and the Best Screenplay at the
68th National Film Awards-2020.This movie title in telugu “aakaasam nee haddhu”. Hero
is Suriya.]
21. Who among the following has won Tour de France 2022?
1) Jonas Vingegaard 2) Jasper Philipsen 3) Dylan Groenewegen
4) Alexander Kristoff 5) Tom Dumoulin
[Explanation:Jonas Vingegaard wins 2022 Tour de France men’s title as Jasper Philipsen
claims final stage in Paris. Denmark’s Vingegaard held a comfortable overall lead ahead of
the ceremonial ride to Paris.]

JULY- 11 Visitus:www.sr eedharscce.com


Sreedhar’s C C E CURRENT AFFAIRS -JULY-2022
22. Who presented National Highway Excellent Award 2021?
1) Narendra Modi 2) Nitin Gadkari 3) Raj Nath Singh
4) Om Birla 5) Both A & C
[Explanation: Lok Sabha Speaker Om Birla along with Minister for Road Transport and
Highways, Nitin Gadkari gave away National Highways Excellence Awards-2021. Awards
were given to the stakeholders and companies engaged in the highway construction and
maintenance of road assets in New Delhi.]
23. Who has won BabuJagjivan Ram AbhinavKisan award?
1) Narendra Singh Tomar 2) PurushottamRupala 3) Anil Bonde

r’s
4) Ramesh Chandra 5) RavindraMetkar
[Explanation: A farmer from village Mhasla in Amravati district in Maharashtra,
RavindraMetkar was honoured with BabuJagjivan Ram AbhinavKisan award for innovative
experiments in farming. The ceremony of ICAR was organized at A P Shinde auditorium of
PUSA area in Delhi.]
24. Who has received the ICCR’s ‘Distinguished Indologist for 2021’ award?
1) Jeffrey Armstrong
ha
3) Maulana Abul Kalam Azad
2) Heinrich Freiherr von Stackelberg

4) Sahasrabuddhe 5) Yu Long Yu

E
[Explanation: Canadian scholar has been awarded the Indian Council for Cultural
Relations (ICCR) Distinguished Indologist for 2021. Jeffrey Armstrong was presented the
d
award in a ceremony by Manish, India’s consul-general in Vancouver.]
25. DSF has instituted a first-of-its-kind ‘Dinesh Shahra Lifetime Award’ for Excellence
in ________.
ee

1) Social Service 2) Teaching 3) Medicine


4) Music 5) Sports
[Explanation: Dinesh Shahra Foundation (DSF) has taken a major stride in its efforts to
C
promote soulful Indian music among the masses. The Foundation has instituted a first-of-
its-kind ‘Dinesh Shahra Lifetime Award’ for Excellence in Music. The initiative is supported
Sr

by the Indian Arts & Cultural Society. The Announcement of this novel recognition was
made by Mr Dinesh Shahra, Trustee –DSF during a cultural event titled “Mausiqui’ in
Mumbai.]
26. Mathematician MarynaViazovska wins prestigious Fields Medal 2022, she belongs
to which country?
1) China 2) Russia 3) Ukraine 4) United States 5) Japan
[Explanation: Ukrainian maths professor, MarynaViazovska, who won the top
C
mathematics prize, the Fields Medal 2022. Viazovska and three other mathematicians
received the Fields Medal, dubbed the Nobel prize in maths, in Helsinki after the ceremony
was moved from Saint Petersburg to the Finnish capital in response to Moscow’s war.
Maryna is only the second woman to receive the award, which is awarded to mathematicians
under 40, since it was created in 1936. The other female laureate, Iran’s Maryam
Mirzakhani]
27. Global Airwards Best Drone Organization Award has won by which of the following
Industry?
1) ID Drone 2) Multi-Rotor Drones. 3) Fixed Rotor Drones
4) Single Rotor Drones 5) Fixed Wing Hybrid VTOL
[Explanation: The Delhi-based leading Drone enterprise platform company IG Drones
has been felicitated with the “Best Drone Organization – Start-up Category” by Airwards.
IG Drones has been selected for the award for its approach of helping local communities
and giving access to technology to various stakeholders during natural disasters and
calamities.]
JULY- 12 Visitus:www.sr eedharscce.com
Sreedhar’s C C E CURRENT AFFAIRS -JULY-2022

3. BANKING
1. Union Government has authorized to which bank for issue and encash Electoral
Bonds?
1) Indian Bank 2) Union Bank of India 3) Canara Bank
4) Punjab National Bank 5) State Bank of India
[Explanation: The government has approved the issuance of the 19th tranche of electoral
bonds, which will be open for sale from January 1 to 10, 2022, ahead of assembly elections

r’s
in five states Uttar Pradesh, Uttarakhand, Punjab, Himachal Pradesh & Goa. There is no
limit on the number of bonds an individual or company can purchase. An electoral bond
will be valid for 15 days. SBI chairman: Dinesh Kumar Khara.]
2. Which Bank has started offering WhatsApp-based bankingservices to its customers?
1) Union Bank of India ha 2) Bank of India 3) Punjab National Bank
4) State Bank of India 5) Canara Bank
[Explanation: State Bank of India (SBI) will provide its customers with WhatsApp-based
banking services. Dinesh Khara, the chairman of SBI, announced this while announcing a
few retail initiatives. Now, SBI customers can check account balance and view mini

E
statements using WhatsApp. According to an official SBI tweet, “Your bank is now on
d
WhatsApp”. A peer-to-peer money transfer tool called WhatsApp Payment is presently
only accessible in India.]
3. Agri finance company Samunnati has a partnership with ________for financing
ee

farmer producer organizations?


1) Bank of Baroda 2) State Bank of India 3) Indian Bank
4) Union Bank of India 5) Canara Bank
[Explanation: Agri enterprise Samunnati has entered into a co-lending partnership aimed
C
at the Farmer Producer Organization (FPO) space with State Bank of India NSE 0.65 %.
Sr

The initial programme amount is Rs 100 crore. This partnership will enable Samunnati to
scale its outreach to FPOs with customized financial solutions at affordable interest rates,
leveraging the vast resources available with SBI.]
4. Which bank has signed MoU with the Indian Air Force for the Defence Salary
Package?
1) Indian Bank 2) Axis Bank 3) Punjab National Bank
4) ICIC Bank 5) State Bank of India
C
[Explanation: SBI has signed MoU with the Indian Air Force for the Defence Salary
Package (DSP) scheme. Under this scheme, it will offer various benefits and features to all
serving and retired Air Force personnel and their families. The lender offers extensive
benefits to the Air Force personnel such as complimentary personal accidental insurance,
air accidental insurance, and additional cover in case of on-duty death.]
5. Which of the following has partnered with Aditya Birla Finance to launch Aditya
Birla credit Card?
1) ICICI Bank 2) SBI Card 3) Bajaj Finserv Ltd
4) PayPal 5) HDFC Bank
[Explanation: SBI Card in partnership with Aditya Birla Finance Ltd (ABFL) announced,
that the launch of ‘Aditya Birla SBI Card’, a “highly rewarding lifestyle credit card”. The
rewards-centric credit card has been launched in two variants - ‘Aditya Birla SBI Card
SELECT’ and ‘Aditya Birla SBI Card’, on the Visa platform. SBI Card MD & CEO: Rama
Mohan Rao Amara.]

JULY- 13 Visitus:www.sr eedharscce.com


Sreedhar’s C C E CURRENT AFFAIRS -JULY-2022

6. Paschim Banga Gramin Bank has signed bancassurance with which Company?
1) LIC 2) Aviva Life Insurance 3) ICICI Prudential Life Insurance
4) HDFC Life 5) SBI Life Insurance
[Explanation: PaschimBangaGramin Bank signs bancassurance pact with SBI Life
Insurance. Through the cooperation, SBI Life’s assortment of protection, wealth
development, credit life, annuity, and savings products will be made available at all Paschim
Banga Gramin Bank branches throughout West Bengal, therefore enhancing the region’s
access to life insurance solutions.]
7. SEBI has imposed penalty on Kotak AMC, how much it is ____________.

r’s
1) 50 Lakh 2) 1 Crore 3) 25 Lakh 4) 3.2 Crore 5) 1.6 Crore
[Explanation: Capital markets regulator Sebi has slapped a penalty of Rs 1.6 crore on
seven entities, including Kotak Mahindra Trustee Company (Rs 40 lakh) and Managing
Director of Kotak AMC Nilesh Shah (Rs 30 lakh), for flouting rules pertaining to mutual
funds while investing in Essel Group companies. SEBI chairman: Madhabi Puri Buch.]
8.
ha
In which of the following Small Finance bank, RBI has imposed a penalty of Rs
57.75 lakh for non-compliance with its directions on “frauds classification”?
1) Au Small Finance Bank 2) Capital Small Finance Bank

E
3) Fincare Small Finance Bank
4) Janalakshmi Small Finance Bank. 5) Suryoday Small Finance Bank
d
[Explanation: RBI has imposed a monetary penalty of ?57.75 lakh on Suryoday Small
Finance Bank for non-compliance with the ‘Reserve Bank of India (Frauds classification
ee

and reporting by commercial banks and select FIs) directions 2016’ issued by RBI.]
9. SEBI imposed how much penalty on National Stock Exchange?
1) 8 Lakh 2) 7 Crore 3) 12 Crore 4) 53 Lakh 5) 9 Crore
C
[Explanation: The Securities and Exchange Board of India (Sebi) has slapped heavy
penalties in a 2015 ‘dark fiber’ case that saw certain brokers exploiting the National Stock
Sr

Exchange’s (NSE’s) internet infrastructure to get faster connectivity to its colocation (colo)
facilities. The markets regulator has imposed a penalty of Rs 7 crore on NSE and Rs 5
crore on former MD & CEO Chitra Ramkrishna.]
10. RBI has imposed monetary penalty on which of the following Banks for non-
compliance with certain RBI directions?
1) IndusInd Bank 2) Kotak Mahindra Bank 3) HDFC Bank
C
4) ICICI Bank 5) Both 1 & 2
[Explanation: Reserve Bank of India (RBI) has imposed a monetary penalty of Rs 1.05
crore and Rs 1 crore on Kotak Mahindra Bank and IndusInd Bank, respectively, for
deficiencies in regulatory compliance. RBI’s these penalties are based on deficiencies in
regulatory compliance and are not intended to pronounce upon the validity of any transaction
or agreement entered into by the bank with its customers.]
11. RBI has imposed penalty of Rs _________on Shri Chhatrapati Rajarshi Shahu
Urban Co-operative Bank Limited
1) 6 lakhs 2) 10 lakhs 3) 15 lakhs 4) 5 lakhs 5) 12 lakhs
[Explanation: RBI has imposed several restrictions on Mumbai-based Raigad Sahakari
Bank. These restrictions include a withdrawal cap of Rs.15,000 per customer, due to
deterioration in the financial condition of the lender. RBI has imposed a penalty of Rs.6
lakh on Shri Chhatrapati Rajarshi Shahu Urban Co-operative Bank Limited, Beed, for
contravention of provisions related to ‘Frauds - Classification and Reporting’.]

JULY- 14 Visitus:www.sr eedharscce.com


Sreedhar’s C C E CURRENT AFFAIRS -JULY-2022

12. RBI has imposed a restriction on Raigad Sahakari Bank including a withdrawal
cap of Rs 15,000 per customer. It is located in which city?
1) Bangalore 2) Mumbai 3) Kolkata 4) Jaipur 5) Pune
[Explanation: RBI has imposed several restrictions on Mumbai-based co-operative bank
Raigad Sahakari Bank due to worsening of financial situation of the lender. The restrictions
include a withdrawal cap of Rs 15,000 per customer and it will remain in force for a
period of six months. RBI has imposed a restriction on Raigad Sahakari Bank should not
be construed as cancellation of banking license.]
13. In which of the following bank, RBI has imposed a penalty of Rs 70 lakh for non-

r’s
compliance with KYC norms?
1) Indian Bank 2) Bank of Baroda 3) State Bank of India
4) Bank of India 5) Canara Bank
[Explanation: A penalty of Rs 70 lakh has also been imposed on Bank of India for non-
compliance with certain provisions of Know Your Customer (KYC) norms and instructions
ha
on ‘compliance function in banks’ issued by RBI. BOI: HQ- Mumbai, CEO: Atanu Kumar
Das, Tagline: Relationships beyond Banking.]
14. Which of the following has set up a mechanism to settle trade payments in Indian
rupee?
1) Ministry of Finance

E 2) RBI 3) IMF
d
4) SEBI 5) NITI Aayog
[Explanation: RBI has set up a mechanism to settle trade payments in Indian rupee. The
central bank has put in place this additional arrangement for invoicing, payment, and
ee

settlement of exports and imports in rupee. Reserve Bank of India has unveiled a rupee
settlement system for international trade, aiming to promote growth of global trade with
emphasis on exports from India and to support the increasing interest of global trading
C
community in Indian Rupee.]
Sr

15. Which bank will be among the world’s top 10 most valuable banks after its merger
with its parent?
1) State Bank of India 2) Axis Bank 3) Punjab National Bank
4) HDFC Bank 5) Union Bank
[Explanation: HDFC Bank has a market capitalization of about $108 billion. It has recently
edged past Citigroup (over $100. 5 billion), which was at number 17. HDFC Bank is
worth more than UBS and DBS Group (both valued at around $58 billion). The country’s
C
largest lender SBI ($57 billion) is at number 32 ahead of BNP Paribas ($55 billion) in terms
of valuation. The combined market cap of HDFC Bank and HDFC ($52 billion) would be
around $160 billion.]
16. Which bank has launched “ai1” mobile banking app, which is a one-stop solution
having over 250 features fulfilling all the banking needs of customers?
1) Axis Bank 2) Canara Bank 3) Andhra Bank
4) Bank of India 5) HDFC Bank
[Explanation: The latest app eliminates the need of having multiple mobile apps working
in silos for availing different specific services. After launching the app, Canara Bank MD &
CEO L V Prabhakar.The lender’s core vision is “e-transactions for everyone, everywhere
and every time” and the bank is leveraging technology to enable the customers to use
banking services at their fingertips. Canara ai1 has advanced features such an intuitive UI
& UX having multiple themes and customizable menus/ dashboards which can be
personalized as per the user’s choice.]

JULY- 15 Visitus:www.sr eedharscce.com


Sreedhar’s C C E CURRENT AFFAIRS -JULY-2022

17. Which bank has signed an agreement with Kerala’s Forest and Wildlife Department
for digital payment collection?
1) Federal Bank 2) Axis Bank 3) South Indian Bank
4) Yes Bank 5) Induslnd Bank
[Explanation: The South Indian Bank has signed an agreement with Kerala’s Forest and
Wildlife Department to enable digital collection of payments at eco-tourism centres, Vanasree
shops, mobile Vanasree units and eco-shops across the State. The tie-up will commence
with the setting up of 124 POS machines at various eco-tourism centres. MD &CEO: Murali
Ramakrishnan.]

r’s
18. Which bank has acquired Agri and healthcare equipment financing portfolio of
DLL India?
1) State Bank of India 2) ICICI Bank 3) Axis Bank
4) Kotak Mahindra Bank ha 5) HDFC Bank
[Explanation: Kotak Mahindra Bank has announced it has acquired the Agri and
Healthcare Equipment financing portfolio of De Lage Landen Financial Services India (DLL
India), a subsidiary of Rabobank-owned De Lage Landen International. It reiterates our
continued commitment towards a strong presence in this space and gives us access to a

E
high-quality customer base,” D Kannan, Group President – Commercial Banking, Kotak
d
Mahindra Bank.]
19. SEBI has banned which of the following company from the securities market for
six months, for flouting listing norms?
ee

1) Prestige Group 2) Godrej Properties Limited 3) Unitech Group


4) Parsvnath Developers 5) OMAXE
[Explanation: SEBI has banned Parsvnath Developers (PDL) from the securities market
C
for six months and imposed a fine of ?15 lakh on the company for flouting listing norms.
Sr

SEBI discovered that PDL failed to make provision for the outstanding amount in ledger
accounts of contractors and sub-contractors, and also failed to strictly comply with
Accounting Standards 7 which deals with construction contracts.]
20. Which General Insurance Company has launched motor insurance product named
“SWITCH”?
1) Bajaj Allianz General Insurance 2) Future Generali General Insurance
3) Edelweiss General Insurance
C
4) Aditya Birla General Insurance 5) Bharti AXA General Insurance
[Explanation: Edelweiss General Insurance (EGI), has launched SWITCH, a one-of-its-
kind on-demand, mobile telematics-based comprehensive motor insurance, for customers
who want their vehicle insurance plans to be hassle-free. SWITCH is a completely digital,
mobile telematics-based motor policy.ED & CEO: Shanai Ghosh.]
21. Ministry of defence has allowed which of the following Banks to provide financial
services in overseas procurement?
1) HDFC Bank 2) AXIS Bank 3) ICICI Bank 4) Both 1 & 2 5) ALL 1, 2 & 3
[Explanation: The defence ministry has allowed three private sector banks to provide
financial services to it for overseas procurement of military hardware. The selected banks
may be allocated with LC business of Rs 2,000 crore, each on the capital and revenue
side, for a period of one year on concurrent basis (Rs 666 crore for each bank under both
capital as well as Revenue).]

JULY- 16 Visitus:www.sr eedharscce.com


Sreedhar’s C C E CURRENT AFFAIRS -JULY-2022
22. Which among the following has Signed MOU with Punjab National Bank to provide
specially designed products to the defence personnel under ‘PNB Rakshak Plus’?
1) BSF 2) Indian Navy 3) CISF 4) Indian Air Force5) Indian Army
[Explanation: The MoU focus on providing personal accident insurance as well as air
accidental insurance to serving, retired as well trainees of the defence forces. It also covers
retired defence pensioners, along with personnel of the central armed police forces, state
police force, and metro police. PNB MD & CEO: Atul Kumar Goel, Chief of Air Staff, Air
Chief Marshal Vivek Ram Chaudhari.]
23. Which Bank has signed agreement with Shriram General Insurance to distribute
the insurer’s products?
1) Tamilnad Mercantile Bank 2) City Union Bank 3) RBL Bank

r’s
4) Yes Bank 5) South Indian Bank
[Explanation: City Union Bank (CUB) and Shriram General Insurance has inked an
agreement in a corporate setup to offer Shriram General Insurance’s insurance products
through its network of 727 offices throughout India. According to this arrangement, Shriram
General Insurance will provide customers of the bank with personal lines of insurance
products including auto, personal injury, house, and travel insurance as well as business

24.
ha
lines of insurance goods like property, marine, and engineering insurance.]
Which Bank has launched Metaverse Virtual lounge ‘Uni-verse’ and Open Banking
Sandbox environment?
1) United Bank of India 2) State Bank of India 3) United Community Bank
4) Corporation Bank

E 5) Union Bank of India


[Explanation: Union Bank of India (UBI) has introduced a Metaverse Virtual Lounge -
d
Uni-verse, & Open Banking Sandbox environment in collaboration with Tech Mahindra in
an effort to elevate consumers’ banking experiences. The bank will work with fintech’s
and start-ups to develop cutting-edge banking products under the Open Banking Sandbox
ee

environment. HQ: Mumbai Tagline: Good People to Bank With.]


25. Recently which of the following Bank has launched video banking services for
customers?
1) AU Small Finance Bank 2) Ujjivan Small Finance Bank
C
3) Equitas Small Finance Bank
4) ESAF Small Finance Bank 5) Fincare Small Finance Bank
Sr

[Explanation: Fincare SFB has announced an expansion in its state-of-the-art banking


services with the launch of its video banking services for customers.Aimed at reducing
waiting lines and offering location agnostic banking services, the Bank launched the video
banking platform whereby customers can connect face-to-face with a customer service
representative of the bank and also chat, simply by initiating a video call using the Bank’s
WhatsApp channel.]
26. IMGC has partnered with which bank to offer mortgage guarantee-backed home
loans?
C
1) Induslnd Bank 2) IDBI Bank 3) Punjab National Bank
4) RBL Bank 5) Citi Bank
[Explanation: India Mortgage Guarantee Corp. (IMGC) has partnered with with RBL Bank
to offer mortgage guarantee-backed home loan products for salaried and non-salaried
customers. This partnership with IMGC will help RBL Bank extend ortgage guarantee-
backed home loans to new borrower segments through its network of over 500 branches
across the country. The collaboration will also allow RBL Bank to mitigate credit risks by
leveraging IMGC’s expertise.]
27. Which bank has launched customers have to pay their direct taxes through the e-
pay tax tab on the portal?
1) HDFC Bank 2) ICICI Bank 3) Axis Bank
4) KotakMahindra bank 5) IDBI Bank
[Explanation: Kotak Mahindra Bank has announced the completion of its technical
integration with the new income tax e-filing portal, the banks make it one of the first private
banks to be fully integrated with the new portal. In October 2021, Kotak Mahindra Bank
has become the first scheduled private sector bank to receive approval as a collections
partner for taxes.]

JULY- 17 Visitus:www.sr eedharscce.com


Sreedhar’s C C E CURRENT AFFAIRS -JULY-2022
28. RBI has signed MoU with which of the following bank to improve mutual cooperation?
1) Bank of America 2) Barclays Bank 3) Doha Bank
4) Bank Indonesia 5) Royal Bank of Scotland
[Explanation: An agreement was reached between the Reserve Bank of India and Bank
Indonesia to increase collaboration in payment systems, digital financial innovation, anti-
money laundering, and countering the funding of terrorism (AML-CFT). On the sidelines
of the G20 Finance Ministers and Central Bank Governors Meeting in Bali, the two central
banks agreed to a Memorandum of Understanding (MoU) to further their mutual
cooperation.]
29. Which bank has named ‘World’s Best SME Bank’ by Euromoney for second time?
1) NDB 2) ADB 3) HSBC 4) DBS 5) AIIB

r’s
[Explanation: Development Bank of Singapore Limited (DBS Bank) has been recognized
as the ‘World’s Best SME Bank’ by Euromoney for the 2nd time. The bank has established
its position as a global industry leader in association with Small to Medium Enterprises
(SMEs) to enhance growth and development. DBS’ latest global best SME bank accolade
comes from leading UK-based financial publication Euromoney, which has honoured DBS
ha
with the ‘World’s Best SME Bank’ title for the second time since 2018.]
30. Which among the following has launched cyber VaultEdge insurance cover for
individuals?
1) New India Assurance Insurance 2) Bajaj Allianz General Insurance

4) SBI General Insurance

E
3) Reliance General Insurance
5) Bharti AXA General Insurance
d
[Explanation: SBI General Insurance has launched the Cyber VaultEdge insurance plan,
a comprehensive cyber insurance cover for individuals that provides protection against
financial losses arising from cyber risks and attacks. It covers unauthorised e-transactions,
loss of wages resulting from identity theft and instances impacting online reputation
ee

including social media trolling, bullying, and stalking.]


31. Which of the following has launched ‘pay as you drive’ add-on feature for motor
insurance Own Damage (OD) policies?
C
1) Go Digit General Insurance 2) SBI General Insurance
3) Bajaj Allianz General Insurance
Sr

4) HDFC ERGO General Insurance 5) Bharti AXA General Insurance


[Explanation: Go Digit General Insurance is the first insurer to offer a ‘pay as you drive’
(PAYD) add-on feature for motor insurance own damage (OD) policies. The Insurance has
launched an add-on feature for motor insurance Own Damage policies, named ‘Pay as
you Drive’. Vehicle-owners can purchase this benefit as part of comprehensive coverage,
along with motor own damage (OD) policies.]
32. On which platform South Indian Bank’s corporate and retail customers have able
to pay customs duty through the its internet banking?
C
1) SbierNet 2) SibcORNet 3) SIBiNTER 4) SIBerNet 5) SIBBiNKING
[Explanation: The facility allows customers to make online payments by selecting South
Indian Bank from the list of banks on CBIC’s portal. The facility was kicked-off with the
inauguration of the bank’s e-FP2) The e-FPB will be attached to the bank’s corporate branch
in Delhi. Consequently, the bank will collect customs duty on behalf of the CBIC under
authorization from the RBI. MD and CEO: Murali Ramakrishnan.]
33. Which of the following Bank has associated with PFRDA for opening NPS Account
through Mobile phone?
1) Union Bank of India 2) Canara Bank 3) Indian Bank
4) Corporation Bank 5) Bank of India
[Explanation: Pension Fund Regulatory and Development Authority (PFRDA) and Bank
of India launched the digital platform for NPS enrolment in association with K-fintech.
Customers can now open NPS (National Pension System) account in a hassle-free and
paperless manner by scanning a QR code. PFRDA chairman Supratim Bandhyopadhyay
inaugurated the digital platform in presence of A K Das, MD & CEO, Bank of India, and
Swarup Dasgupta, executive director of the bank.]

JULY- 18 Visitus:www.sr eedharscce.com


Sreedhar’s C C E CURRENT AFFAIRS -JULY-2022

34. Which general insurance Company has launched automation of vehicle inspection
for insurance renewals with AI technology?
1) Kotak Mahindra General Insurance 2) SBI General Insurance
3) Bajaj Allianz General Insurance
4) HDFC ERGO General Insurance 5) Bharti AXA General Insurance
[Explanation: The insurance company has partnered Inspektlabs to implement this for
its policies. Through this feature, policyholders can take photos or videos of their vehicle
and upload them on the cloud-based application for policy renewal. The inspection report,
including details of damages, if any, will be automatically generated within a few seconds

r’s
after the customer uploads inspection photos or videos.]
35. Which Bank has collaborated with CBDT for new online tax payment service?
1) Kotak Mahindra Bank 2) South Indian Bank 3) HDFC Bank
4) Federal Bank 5) ICICI Bank
[Explanation: Federal Bank partners with the Central Board of direct taxes to assist
ha
taxpayers in making their payments via the e-pay tax facility in the e-filing portal of the
income tax. Anyone can now pay their taxes instantly via any of their payment modes such
as Debit/Credit Card, UPI, Net banking, cash, NEFT/RTGS etc. NRIs and Domestic

E
Customers of the bank, as well as any tax-paying citizen in India can generate a tax challan
and make the payment through the Bank’s branches. Federal Bank HQ: Aluva, Kerala,
d
Tagline: Your Perfect Banking Partner.]
36. Which Bank has raised Rs 2,000 crore by issuing Basel III compliant bonds?
1) Canara Bank 2) State Bank of India 3) Syndicate Bank
ee

4) Bank of India 5) Punjab National Bank


[Explanation: The bank has received total bid amount of Rs 5,719 crore, out of which
full issuance of Rs 2,000 crore was accepted at 8.24 per cent. The non-convertible,
C
perpetual, taxable, subordinated, fully paid-up secured Basel III compliant additional tier
Sr

I bonds carry coupon at 8.24 per cent per annum. Under the Basel-III capital regulations,
banks globally need to improve and strengthen their capital planning processes.]
37. Which bank has partnered with EazyDiner to launch a co-branded credit card?
1) Federal Bank 2) Kotak Mahindra Bank 3) HDFC Bank
4) AXIS Bank 5) IndusInd Bank
[Explanation: IndusInd Bank has announced a partnership with table reservation platform
EazyDiner to launch a tailor-made co-branded credit card - EazyDiner IndusInd Bank
C
Credit Card. The cardholder will get complimentary EazyDiner Prime membership that
will offer a guaranteed discount of up to 25% at select restaurants. This discount can be
clubbed with an additional 25% discount up to Rs.1,000 every time a customer pays on
the EazyDiner app via PayEazy.]
38. Which of the following have Received the RBI Nod for Cross-Border Payments?
1) Cashfree Payments 2) PayNearby 3) neo-banking unicorn
4) Open Financial Technologies 5) All of the above
[Explanation: RBI has approved products of four entities after they tested acceptable
under the second cohort of the regulatory sandbox for cross border payments. In December
2020, the RBI had announced the opening of the second cohort, of those eight, four entities
– payment solution providers, Cashfree Payments and Fairex Solutions; financial services
provider Nearby Technologies, which operates PayNearby, and neo-banking unicorn,
Open Financial Technologies - have been found viable within the boundary conditions
defined during testing.]

JULY- 19 Visitus:www.sr eedharscce.com


Sreedhar’s C C E CURRENT AFFAIRS -JULY-2022
39. Which bank has raised Rs-11,000 Crore in Basel III bonds in FY23?
1) Union Bank of India 2) UCO Bank 3) State Bank of India
4) Bank of Baroda 5) Bank of India
[Explanation: The bank’s central board has approved raising capital by way of issuance
of Basel lll compliant debt instrument in USD/INR and/or any other convertible currency,
in FY23, according to a regulatory filing. The country’s largest lender in terms of asset
size and customer base plans to raise fresh Additional Tier 1 (AT1) capital up to Rs 7,000
crore, subject to the government’s concurrence. Also, it plans to raise fresh Tier 2 capital of
up to Rs.4,000 crore. Shares of SBI gained 2.13 per cent to close at Rs 508.60 apiece on

r’s
BSE.]
40. Which of the following Enterprise got RBI nod to start NBFC operations?
1) Power Finance Corporation Limited 2) L&T Finance Holdings
3) Muthoot Finance Ltd.
4) HDB Financial Servicesha 5) Piramal Enterprises
[Explanation: RBI has granted a certificate of registration to the company to commence
the business of non-banking financial institution without accepting public deposits. The
approval from the RBI comes after the company received clearance of the shareholders to
demerge its pharmaceuticals business and simplify its corporate structure. Earlier, the

E
company had acquired Dewan Housing Finance (DHFL) for Rs 34,250 crore. The
acquisition will diversify Piramal Enterprises’ loan book towards retail financing by scaling
d
up the retail loan.]
41. Which bank has signed bancassurance with Aditya Birla Health Insurance?
ee

1) Federal Bank 2) City Union Bank 3) Bandhan Bank


4) CSB Bank 5) Axis Bank
[Explanation: This is the third such tie-up for CUB after pacts with Star Health and Care
Health. Old private sector lender City Union Bank and standalone health insurer Aditya
C
Birla Health Insurance Co Ltd (ABHICL), the health insurance subsidiary of Aditya Birla
Sr

Capital Ltd, have announced a bancassurance partnership for the distribution of health
insurance products of the latter through the former’s network of branches across the
country.]
42. Tata Motors has joined hands with which bank to provide retail car loans?
1) Bank of Baroda 2) Union Bank of India 3) State Bank of India
4) Indian Bank 5) Bank of India
[Explanation: As per the understanding, Indian Bank will provide customized car loans
C
from its 5,700+ branches across the country for Tata Motors passenger vehicle range. The
company says the partnership will see car loans with interest rates starting at 7.8 per cent
with an option to avail up to 90 per cent financing of the PVs. The customers can repay the
loans by up to 7-years and will be able to foreclose or provide partial payments at no
additional fees. Indian Bank HQ: Chennai,Tagline: Your Own Bank, Banking That’s Twice
as Good.]
43. Which Bank has opened a new currency chest in Deedargunj, Patna for cash
management?
1) Federal Bank 2) Indian Bank 3) Bandhan Bank
4) Bank of India 5) Bank of Baroda
[Explanation: This will help the private lender manage cash for its branches and ATMs
there. This currency chest will also help its customers, micro, small, medium enterprises
and small traders with supply of currency notes to the bank branches and the ATMs as
and when required. As Patna sees a lot of cash transactions on a daily basis, the chest
will also benefit the bank branches with storage.Bandhan Bank MD and CEO C S Ghosh.]

JULY- 20 Visitus:www.sr eedharscce.com


Sreedhar’s C C E CURRENT AFFAIRS -JULY-2022

4.BOOKS AND AUTHORS


1. Who is the Author of the book titled, “The McMAHON LINE: A CENTURY OF
DISCORD”?
1) Prarthna Batra 2) JJ Singh 3) Susan Ninan
4) RN Bhaskar 5) Sonu Phogat
[Explanation: The Governor of Himachal Pradesh, RajendraVishwanathArlekar, recently
released a book titled “The McMahon line: A century of discord”. The book has been

r’s
authored by General JJ Singh (Retd), the former Governor of Arunachal Pradesh and
former Chief of Army Staff (CoAS). The book is based on the experiences and research of
General JJ Singh on the India-China border dispute.]
2. Who among the following is the author of the book ‘Shinrin Yoku’?
1) Hector Garcia 2) Francesc Miralles 3) Dr. Shashi Tharoor
4) Both A&B 5) Both A&C
ha
[Explanation: The authors Hector Garcia and Francesc Miralles has released another
self-help book, titled “Shinrin Yoku: The Rejuvenating Practice of Forest Bathing”. ‘Shinrin
Yoku’, which translates as ‘forest bathing’, refers to taking mindful walks amidst nature to

3.

E
relieve stress and realign our energies.]
Who has launched the book ‘Cavalier’s Take – Memoir of a Soldiers General’?
d
1) Manoj Pande 2) B.S. Raju 3) Rana Pratap Kalita
4) Bipin Rawat 5) Devendra Pratap Panday
[Explanation: Army Chief General Manoj Pande has unveiled the book ‘Cavalier’s Take
ee

– Memoir of a Soldiers General’ written by Lt Gen RajanBakhshi, former GOC-in-C, Central


Army and GOC, 14 Corps.]
4. who is the author of the Book titled “Getting the Bread: The Gen-Z Way to Success”?
C
1) RN Bhaskar 2) Prarthna Batra 3) Vinit Karnik
4) Chetan Prakash 5) Prasanth Kumar
Sr

[Explanation: Young YouTuber Prarthna Batra’s debut book ‘Getting the Bread: The
Gen-Z Way to Success’ launched by sporting icon Sakshi Malik. Prarthna Batra shares
her worldview as well as her experiences of interviewing eminent leaders, entrepreneurs,
and media personalities for her popular YouTube channel.]
5. Who is the author of the book titled, “The Resilient Entrepreneur”?
1) Dhruti Shah 2) Ajay Singh 3) Jimmy Soni
4) Ravi Venkatesan 5) J J Singh
C
[Explanation: The Resilient Entrepreneur written by Dhruti Shah, was launched globally
in the presence of entrepreneurs from Singapore, India, and the United States. The book
is published by Ocean Reeve Publishing. This book should be a part of your library
whether you are embarking on your first entrepreneurial journey or lead an established
organization pursuing intrapreneurial thinking and strategy.]
6. Who is the author of the book titled,“Beyond the Misty Veil, Temple Tales of
Uttarakhand”?
1) Ruskin Bond 2) Aradhana Johri 3) Dhruti Shah
4) JJ Singh 5) Prarthna Batra
[Explanation: CM Pushkar Singh Dhami launches book Beyond the Misty Veil — Temple
Tales of Uttarakhand. Uttarakhand Chief Minister Pushkar Singh Dhami has launched a
book titled Beyond the Misty Veil — Temple Tales of Uttarakhand by Aradhana Johri,
which seeks to create awareness about the temples and promote tourism in the State.The
book based on the mythological temples of Devbhoomi Uttarakhand informs us about our
culture and mythology. Apart from Chardham, religious tourism should also be promoted.]

JULY- 21 Visitus:www.sr eedharscce.com


Sreedhar’s C C E CURRENT AFFAIRS -JULY-2022

7. Who is the author of the book ‘The Light We Carry’?


1) Jhumpa Lahiri 2) Anita Desai 3) Sudha Murty
4) Michelle Obama 5) Betty Ford
[Explanation: After the huge success of her acclaimed memoir “Becoming” in 2018, former
US first lady Michelle Obama has announced the release of her second book, “The Light
We Carry: Overcoming in Uncertain Times”, later this year. Published by Penguin Random
House. In the book, Michelle Obama offers readers a series of fresh stories and insightful
reflections on change, challenge, and power, including “her belief that when we light up
for others, we can illuminate the richness and potential of the world around us, discovering

r’s
deeper truths and new pathways for progress”.]
8. Who is the author of the book ‘In the Shadow of a Legend: Dilip Kumar’?
1) Shashi Tharoor. 2) Faisal Farooqui 3) Kiran Desai
4) DevduttPattanaik. ha 5) Amish Tripathi
[Explanation: Faisal Farooqui’s book Dilip Kumar: The Shadow of a Legend is best
approached not as a biography of a celebrity but as an intimate portrait of a man that he
loves and looks up to. The book focuses on what was impressive about Kumar but it does

E
not resort to deificatio. The book celebrates Dilip Kumar’s amazing versatility as an actor.
The alcoholic Devdas, the lover who always seems to lose his lady, the Salim who defies
d
Akbar, the heartwarming character from Ram aurShyam, the dacoit from Ganga]
9. Who is the author of the book titled, “Sing, Dance and Pray”, a biography of Srila
Prabhupada, the founder of ISKCON?
ee

1) Aradhana Johri 2) Dhruti Shah 3) JJ Singh


4) Prarthna Batra 5) Hindol Sengupta
[Explanation: Vice President of India M. Venkaiah Naidu released the biography Sing,
C
Dance and Pray: The Inspirational Story of Srila Prabhupada, by noted historian Hindol
Sr

Sengupta.He had founded the International Society for Krishna Consciousness (ISKCON),
colloquially known as the ‘Hare Krishna Movement’, and saw it grow into a worldwide
confederation of more than 100 temples, ashrams and cultural centers.]
10. Who among the following has launched a book named ‘Swadhinata Sangram Na
Surviro’?
1) Meenakshi Lekhi 2) Amit Shah 3) Subrahmanyam Jaishankar
4) Anurag Singh Thakur 5) G. Kishan Reddy
C
[Explanation: Union Minister of State for External Affairs and Culture, Meenakshi Lekhi
has launched a book in Gujarati commemorating the contributions of freedom fighters at a
function. The book, ‘Swadhinata Sangram Na Surviro’, celebrates 75 freedom fighters
and shares the stories of the sacrifices made by them for the country.]
11. Who has launched ‘Connecting through Culture’, an anthology of essays on various
aspects of India’s soft power strengths?
1) Arjun Munda 2) S. Jaishankar 3) Amit Shah
4) Narendra Singh Tomar 5) Meenakshi Lekhi
[Explanation: External Affairs Minister S. Jaishankar launched ‘Connecting through
Culture’, an anthology of essays on various aspects of India’s soft power strengths, at
Sushma Swaraj Bhavan in New Delhi, India. The Minister described the book as the “good
cop” in diplomacy and it can be used to encourage others to work with India because “it
creates a comfort about India.]

JULY- 22 Visitus:www.sr eedharscce.com


Sreedhar’s C C E CURRENT AFFAIRS -JULY-2022

5.OBITUARIES
1. Kishore Das passed away recently. He was a renowned ________.
1) Actor 2) Cricketer 3) Singer 4) Boxer 5) Journalist
[Explanation: Kishor Das was known for his roles in TV shows like “Bidhata and
Bandhun”, and has also starred in multiple short films. He also won the “Candid Young
Achievement Award” in 2019, and “Most Popular Actor 2020-2021” in the Asianet Icon
Award.]

r’s
2. Peter Brook passed away recently. He was a __________.
1) Cricketer 2) Film Director 3) Journalist
4) Scientist 5) Athlete
[Explanation: Peter Brook, one of the world’s most innovative theatre directors who
ha
perfected the art of staging powerful drama in bizarre venues, has died aged 97. He
brought a stunning nine-hour adaptation of the Sanskrit epic “The Mahabharata” from
France to New York. ]
3. EN Sudhir passed away recently. He belongs to which sports?
1) Kabaddi

E
2) Football 3) Cricket 4) Table tennis 5) Basketball
d
[Explanation: Sudhir also played for Young Challengers (Kerala), Vasco Sports Club
(Goa), and Mahindra & Mahindra at the club level. He played as a goalkeeper for India in
the 1970. At the domestic level, he played for three different states in the Santosh Trophy
ee

Kerala, Goa, and Maharashtra.]


4. Narendra Singhpassed away recently. He was a former agriculture minister of
which state?
C
1) Assam 2) Bihar 3) Sikkim 4) Punjab 5) Gujarat
Sr

[Explanation: A firebrand socialist leader Mr. Singh came from the upper caste Rajput
community from the Jamui and Banka region of Bihar. His son Sumit Kumar is a minister
in Chief Minister Nitish Kumar’s cabinet. He was 75 Years old.]
5. Gudipoodi Srihari passed away recently. He was a renowned _____.
1) Journalist 2) Art critic 3) Film Director 4) Both A & B 5) Both B & C
[Explanation: A popular newsreader on the radio for two decades, journalist and a
pioneering art and film critic in the Telugu states.A contributor to The Hindu for over five
C
decades, the writer and art critic set a precedent in film and art reviews Akashavani,
prantiyavaartalu, chaduvutunnadiGudipoodi Srihari.]
6. Nigeria’s Mohammed Barkindo recently passed away, he is the Secretary General
of which organization?
1) NATO 2) OECD 3) APEC 4) ASEAN 5) OPEC
[Explanation: He was The Secretary-General of the Organization of Petroleum Exporting
Countries, died at the age of 63. He’d held the post for six years since 2016. OPEC aims
to regulate the supply of oil in order to set the price on the world market. OPEC organization
was established in 1960.Head quarter: Vienna, Ausria.]
7. Gautham Raju passed awayrecently.He was a____________.
1) Journalist 2) Film editor 3) Singer 4) Vocalist 5) Urdu scholar
[Explanation: Gautham Raju started his film career as an editor with ChattanikiKallulevu
and has over 800 films to credit in Telugu, Tamil, Kannada, Malayalam, and Hindi. He

JULY- 23 Visitus:www.sr eedharscce.com


Sreedhar’s C C E CURRENT AFFAIRS -JULY-2022
won Nandi Award in the best editing category for Aadi movie. He was 68years old.]
8. ‘Shinzo Abe’ Former Prime Minister of which country passed away recently?
1) China 2) Russia 3) Japan 4) USA 5) Chile
[Explanation: Abe became Japan’s youngest post-war prime minister and the first born
after World War II. Abe resigned as prime minister after one year due to ulcerative colitis
and his party’s recent losses.Ito Hirobumi became the first Prime Minister of Japan in
1885. Capital:Kyoto and Tokyo (japan has two capitals); currency:yen]
9. José Eduardo dos Santos has passed away recently, he is a former president of
_______.

r’s
1) Angola 2) Botswana 3) Zambia 4) Namibia 5) Zimbabwe
[Explanation: José Eduardo dos Santos, once one of Africa’s longest-serving rulers who
during almost four decades as president of Angola fought the continent’s longest civil war
and turned his country into a major oil producer as well as one of the world’s poorest and
ha
most corrupt nations,He was 79.]
10. Composer Monty Norman passed away recently. He is from which of the following
country?
1) Brazil 2) U.K

E 3) Germany 4) England 5) Sweden


[Explanation: Monty is the composer of the iconic James Bond theme. He most famously
d
composed the score for Dr. No, the 1962 James Bond film starring Sean Connery. He
was passed away at the age of 94. A jury in London’s High Court ruled in Norman’s favor
in 2001, awarding him 30,000 pounds plus court costs. Capital: London, Currency: the
ee

pound stirling (£).]


11. Padma Shri awardee AvdhashKaushal has passed away, he is a ___________.
1) Writer 2) Singer 3) Social activist4) Novelist 5) Actor
C
[Explanation: Padma Shri winning noted social worker AvdhashKaushal. He was 87.
Sr

He was the founder of the NGO named Rural Litigation and Entitlement Kendra (based in
Dehradun, Uttarakhand). He was known for his fight against human rights and the
conservation of the environment. Kaushal had been close to former prime minister Rajiv
Gandhi.]
12. PrathapPothen has passed away he is a __________.
1) Director 2) Politician 3) Actor 4) Writer 5) Choreographer
[Explanation: He was Born in Thiruvananthapuram, Pothen had acted in more than 100
C
films in the Tamil, Malayalam and Telugu besides directing a few super hit films. Well
known south Indian film actor and director.]
13. Shri Bhupinder Singh passed away recently. He was a noted __________.
1) Artist 2) Director 3) Singer 4) Author 5) Journalist
[Explanation: Renowned ghazal singer Bhupinder Singh, known for crooning classics
like “Naam Gum Jayega” and “DilDhoondta Hai”. the singer came to be best known for
songs like “Do DiwaneShehar Mein”, “EkAkela is Shehar Mein”.Bhupinder Singh has
won 1 award - Mirchi Music Award in 2021.]
14. AchuthanKudallur has passed away, he is a __________.
1) Writer 2) Journalist 3) Artist 4) Novelist 5) Socialist
[Explanation: A recipient of several awards, including the National Academy Award
and Tamil Nadu Lalit Kala Akademi Award. He also joined the Madras Arts Club at the
Fine Arts College.]

JULY- 24 Visitus:www.sr eedharscce.com


Sreedhar’s C C E CURRENT AFFAIRS -JULY-2022

15. West Germany ‘Uwe Seeler’ passed away recently. He was related to which sports?
1) Tennis 2) Golf 3) Cricket 4) Football 5) Hockey
[Explanation: Uwe Seeler, who led West Germany to the 1966 World Cup final as captain
of the national team. He was 85years old. Seeler won the German championship in 1960
and German Cup in 1963 with Hamburg. The Bundesliga is renowned for its exceptional
strikers and Hamburg and West Germany legend Uwe Seeler.]
16. Ajay Parida has passed away, he is a noted__________.
1) Journalist 2) Author 3) Scientist 4) Writer 5) Politician

r’s
[Explanation: He was scientist and director of the Institute of Life Sciences (ILS),
Bhubaneswar. Ajay Kumar Parida has been conferred the Padma Shri as part of the
Republic Day awards in 2014. Padma Shri is the fourth highest civilian award in the
Republic of India, after the Bharat Ratna, the Padma Vibhushan and the Padma Bhushan.
It is awarded by the Government of India.]
17.
ha
Padma Shri award Dr. SushovanBandyopadhyay passed away recently. In which
year he received the Award?
1) 2018 2) 2004 3) 2019 4) 2020 5) 2016

E
[Explanation: He was awarded the Padma Shri in 2020. The same year, his name was
d
recorded in the Guinness World World Records for treating the maximum number of
patients. SushovanBandyopadhyay, famous as Bengal’s ‘one rupee doctor and is fondly
known as ‘EkTakarDaktar’ (one rupee doctor). He is an ex-MLA from Bolpur seat and had
ee

contested on Congress ticket in 1984.]


18. David Warner was passed away recently. He was a ___________.
1) Singer 2) Scientist 3) Author 4) Actor 5) Politician
C
[Explanation: David Warner, the English actor who gave memorable performances on
Sr

the big screen, in a key role in “The Omen,” and as villains in “Time After Time,” “Time
Bandits” and “Tron”. Primetime Emmy Award for Outstanding Supporting Actor in a Limited
or Anthology Series or Movie.]
19. AtulanandaGoswami has passed away recently, he was an eminent _________.
1) Poet 2) Jounalist 3) Writer 4) Author 5) Singer
[Explanation: Goswami was awarded the SahityaAkademi award in 2006 for his novel
C
“SenehJorirGanthi”. His works like “Namghoria”, “HamdoiPulor Jon”, “Raajpat”,
“Polatok”, “SenehJorirGanthi”. He is most famous for “Namghoria” written in the early
1990s. He was conferred with the prestigious Assam Valley Literary Award in 2013.]
20. Renowned Singer Nirmala Mishra passes away recently, She is from __________.
1) New Delhi 2) Punjab 3) Odisha 4) West Bengal 5) Karnataka
[Explanation: West Bengal government conferred on her the ‘SangeetSamman’,
‘SangeetMahasamman’ and ‘Bangabhibhushan’ awards, she was given the
SangeetSudhakarBalakrushnaDas Award for her lifetime contribution to the Odia music.]
21. Rasik Dave has passed away recently, he is from __________.
1) Actor 2) Director 3) Singer 4) Writer 5) Journalist
[Explanation: Actor Rasik Dave, known for appearing in Hindi and Gujarati films and
shows. popular for her role of Daksha Virani in the TV show Kyunki Saas Bhi Kabhi
Bahu Thi. Rasik Dave was popular for playing the role of Nand in Mahabharat.]

JULY- 25 Visitus:www.sr eedharscce.com


Sreedhar’s C C E CURRENT AFFAIRS -JULY-2022

6.ECONOMY
1. India’s GST collections has jumped 56% to _______ in June.
1) Rs 1.55 lakh crore 2) Rs 1.45 lakh crore 3) Rs 1.65 lakh crore
4) Rs 1.40 lakh crore 5) Rs 1.60 lakh crore
[Explanation: India’s GST collections in June surged 56 per cent year-on-year to around
Rs 1.45 lakh crore on account of economic recovery and better anti-evasion measures.
GST collections in June 2021 stood at Rs 92,800 crore. The GST mop-up rose, though
slightly from Rs 1.41 lakh crore in May 2022.]
2. Rating agency ‘Crisil’ has lowered India’s real GDP growth forecast to _______ for

r’s
FY23.
1) 6.9% 2) 7.3% 3) 7.9% 4) 8.2% 5) 8.5%
[Explanation: Rating agency Crisil has lowered India’s real gross domestic product (GDP)
growth forecast to 7.3% from 7.8% for FY23 from. The fall has been attributed to the
downward revision to higher oil prices, slowing of export demand and high inflation.]
3. International Monetary Fund (IMF) has slashed India’s economic growth forecast
to_______%.
1) 7.5 2) 6.8
ha 3) 7.4 4) 8.5 5) 7.2
[Explanation: IMF has slashed India’s economic growth forecast to 7.4% from 8.2% it
estimated in April, citing the economy’s vulnerability to external shocks and rapid monetary

E
policy tightening. In its latest World Economic Outlook update, IMF suggested policymakers
prioritize taming inflation through further monetary policy tightening.]
d
4. Asian Development Bank (ADB), has slashed India’s GDP growth forecast to
_______ per cent for FY23.
1) 7.5 2) 7.8 3) 7.6 4) 7.2 5) 7.4
[Explanation: ADB has slashed its growth forecast for India to 7.2 per cent for FY23
ee

from 7.5 per cent estimated earlier citing higher than anticipated inflation since April and
subsequent monetary tightening by the central bank. The Manila-based multilateral
development bank also raised its inflation forecast for India to 6.7 per cent for FY23 from
5.8 per cent projected earlier.]
C
5. FICCI downgrades India’s GDP growth forecast to _____% from 7.4% for FY23.
1) 7.2 2) 6.8 3) 7.0 4) 7.1 5) 6.5
Sr

[Explanation: India’s annual Gross Domestic Product (GDP) growth forecast for the
financial year 2022-23 has been downgraded by the Federation of Indian Chambers of
Commerce & Industry (FICCI) at 7.0% with a minimum and maximum growth estimate of
6.5% and 7.3% respectively.]
6. Morgan Stanley cuts India’s FY23 GDP forecast to ____%.
1) 7.4 2) 6.7 3) 5.6 4) 7.2 5) 7.8
[Explanation: American brokerage Morgan Stanley cut its FY23 real GDP expansion
estimate for India by 0.40 per cent to 7.2 per cent on slower global growth and GDP growth
C
will slow down to 6.4 per cent in FY24. A majority of watchers are expecting FY23 GDP
growth to come at over 7 per cent levels. The RBI estimate also stands at 7.2 per cent.]
7. India’s Retail inflation rate, as measured by the Consumer Price Index (CPI) is
stood at ______% in June 2022.
1) 7.04 2) 6.26 3) 7.01 4) 6.95 5) 7.79
[Explanation: India’s retail inflation eased slightly to 7.01% in June against 7.04% in the
previous month. However, in June 2021, the retail inflation stood at 6.26%. The consumer
price-based inflation (CPI) has breached the upper limit of RBI’s tolerance band, ranging
from 2-6%, for the sixth consecutive month and has remained above 7% for third consecutive
month.]
8. The Wholesale Price Index-based inflation has projected________% in June 2022.
1) 15.88 2) 12.07 3) 15.18 4) 14.67 5) 12.65
[Explanation: The annual rate of inflation based on All-India Wholesale Price
Index (WPI) number is 15.18% (Provisional) for the month of June, 2022 (over
June, 2021). This is marginally lower than the WPI number of 15.88% in May,
2022, the high rate of inflation in June, 2022 is primarily due to rise in prices of
mineral oils, food articles, crude petroleum & natural gas, basic metals etc.]

JULY- 26 Visitus:www.sr eedharscce.com


Sreedhar’s C C E CURRENT AFFAIRS -JULY-2022
9. Nomura has cut India GDP forecast for 2022-23 to __________ percentage.
1) 4.7 2) 3.4 3) 4.3 4) 3.6 5) 4.2
[Explanation: Nomura has cut its 2023 forecast for economic growth in India, as
measured by the gross domestic product (GDP), to 4.7 per cent from its earlier
projection of 5.4 per cent amid recession fears and rising interest rates.]
10. Exports of agricultural and processed food products raised by _______% in FY23?
1) 14% 2) 10% 3) 16% 4) 15% 5) 22%
[Explanation: The exports of agricultural and processed food products rose by 14 percent
in the first three months of the current financial year compared to the corresponding period
of 2021-22. Commerce and Industry Ministry, the overall export has increased to five
thousand 987 million dollars from April to June this year from five thousand 256 million

r’s
dollars over the same period of the last fiscal. The exports of fresh fruits and vegetables
has registered 8.6 percent growth.]
11. Industrial production has rose to ___________ percentage in May 2022.
1) 19.6% 2) 15.88% 3) 14.55% 4) 19.08% 5) 17.64%
[Explanation: Index of Industrial Production (IIP) growth zoomed to 19.6% in May as
ha
against 7.1% in April, data released by the National Statistical Office. The manufacturing
sector’s output grew 20.6% in May this year. In May 2022, the mining output climbed
10.9%, and power generation increased 23.5%.]
12. Data released by Finance Ministry showed that the trade deficit widened in June

1) $25.4

E
to an all-time high of __________billion.
2) $26.2 3) $20.5 4) $27.3 5) $22.5
d
[Explanation: India’s trade deficit widens to $26.18 billion in June; exports rise 23%,
imports up 55% India’s overall exports, merchandise and services combined, are estimated
to have grown 22.95% year-on-year in June 2022, at around US$64.91 billion, the Ministry
of Commerce & Industry.]
ee

13. The Union Cabinet has approved a project for saturation of 4G mobile services in
over 24,000 villages across the country at a total cost of Rs. _________Cr.
1) 26,316 2) 25,234 3) 34,756 4) 14,725 5) 25,345
C
[Explanation: The project will provide 4G mobile services in 24,680 uncovered villages
in remote and difficult areas. It has a provision to include 20 per cent additional villages on
Sr

account of rehabilitation, new-settlements, withdrawal of services by existing operators


etc.]
14. Pre-packed and labelled food items will attract _________ percentage GST.
1) 12% 2) 5% 3) 10% 4) 8% 5) 4%
[Explanation: Customers will have to pay 5 per cent GST on pre-packed, labelled food
items like atta, paneer and curd, besides hospital rooms with rent above Rs 5,000. In
addition, hotel rooms with tariff of up to Rs 1,000/day, maps and charts, including atlases,
will attract a 12 per cent Goods and Services Tax (GST), while 18 per cent GST will be
C
levied on tetra packs and fees charged by banks for the issue of cheques.]
15. What is the GST rate for fresh Milk and pasteurized Milk?
1) 5% 2) 3% 3) 0% 4) 2% 5) 1%
[Explanation: Government has clarified that fresh milk and pasteurized milk are fully
exempted from Goods and Service Tax (GST). Further, milk products like curd, lassi,
butter milk and paneer are also exempted from GST if sold in forms other than those pre-
packaged and labelled. Finance Minister Nirmala Sitharaman told that a nominal GST of 5
per cent applies to curd, lassi, butter milk and paneer when sold in pre-packaged and
labelled form, and Ultra High-Temperature Milk.]
16. U.S. firm Panattoni to invest _________ mn dollar in logistics parks.
1) 200 2) 250 3) 150 4) 300 5) 450
[Explanation: Panattoni, a global leader in industrial & logistics real estate development,
has debuted in the Asian market with its entry in India. Indian venture, Panattoni India
Development Pvt Ltd, is headquartered in Bengaluru. Aim is to develop four industrial and
logistics parks as it seeks to tap the growing demand for warehousing spaces across
major cities.]

JULY- 27 Visitus:www.sr eedharscce.com


Sreedhar’s C C E CURRENT AFFAIRS -JULY-2022
17. India has contributed _______ million dollars to UN RWA for Palestine refugees.
1) 2 2) 5 3) 2.5 4) 9.1 5) 5.5
[Explanation: This timely contribution is a strong demonstration of India’s unwavering
support to the work of UNRWA and commitment to the wellbeing of Palestine refugees. On
behalf of UNRWA, I would like to express our deep appreciation for the Government of
India for its continued funding to the Agency and its backing of Palestine refugees across
the Middle East.]
18. Government has received record bids of ______lakh crore rupees on Day 1 of
India’s first-ever 5G spectrum auction.
1) 1.75 2) 1.45 3) 1.54 4) 1.95 5) 4.75
[Explanation: All three private telcos - Reliance Jio, Bharti Airtel and Vodafone Idea are
stated to have bid for mid- and high-band airwaves as they set the stage to launch services

r’s
which can offer 10 times higher mobile internet speeds than 4G. New entrant Adani Data
Networks, a unit of the diversified Adani Group, is expected to have put in bids only for
the high band, also known as the millimetre wave, for use in its captive network.]
19. Union Cabinet has approved ________lakh crore rupees revival package for BSNL.
1) 1.64 2) 1.50 3) 2.12 4) 5.46 5) 3.25
[Explanation: Cabinet has cleared a Rs 1.64-lakh crore package for revival of state-
ha
owned telecom service provider Bharat Sanchar Nigam Ltd (BSNL), with the focus on
fresh capital for upgrading the loss-making company’s network and allocating spectrum
for it to offer 4G services, de-stressing its balance sheet and augmenting its fiber network.]
20. Centre has likely to Extend the PMAY-Urban Mission Scheme till _______.
1) March 2024
4) April 2025

E 2) June 2023
5) June 2024
3) March 2023
d
[Explanation: The Ministry of Housing and Urban Affairs is implementing PMAY – Urban
(PMAY-U) – ‘Housing for All’ Mission, since June 25, 2015 for giving Central assistance to
States and Union Territories (UTs) for providing all-weather ‘pucca’ houses to all eligible
urban beneficiaries. Based on the project proposals submitted by States/ UTs, a total of
ee

122.69 lakh houses have been sanctioned during the Mission period till 31 March 2022.]
21. India’s bioeconomy likely to touch $300 billion by ________.
1) 2025 2) 2027 3) 2035 4) 2030 5) 2024
[Explanation: India’s bioeconomy is likely to touch USD 150 billion by 2025 and over
C
USD 300 billion by 2030.The India Bio Economy Report 2022, based on the data on
biotech sector’s contribution to the economy, has been brought out by the Biotechnology
Sr

Industry Research Assistance Council (BIRAC). The country’s bioeconomy has reached
over USD 80 billion in 2021, registering a 14.1 per cent growth over USD 70.2 billion in
2020.]
22. According to World Health Organization (WHO) report, Which Country was the top
remittance recipient in 2021, with $87 billion?
1) India 2) China 3) Mexico 4) Philippines 5) Egypt
[Explanation: Despite the pandemic which posed a serious threat to remittances to India
from countries like the US, UAE and Saudi Arabia, the country received $87 billion in
remittances in 2021, becoming the top remittance recipient.United States was the largest
C
source country for remittances in 2020, followed by the United Arab Emirates, Saudi
Arabia and Switzerland. Remittances increase or maintain consumer spending and soften
the blow of economic hardship, such as during the COVID-19 pandemic.]
23. According to the RBI report, India’s Foreign exchange reserves dropped to ____
billion.
1) $572 2) $632 3) $542 4) $687 5) $512
[Explanation: Foreign exchange reserves fell by $7.5 billion to $572 billion, according to
data released by the RBI. Reserves fell to a 20-month low following heavy intervention by the
central bank in the forex market as it sold dollars to prevent a sharp depreciation of the rupee.]
24. Insurance companies has made Rs.40,000 crore from__________ scheme in 5 years.
1) Pradhan Mantri Suraksha Bima Yojana
2) Pradhan Mantri Fasal Bima Yojana
3) Central Government Health Scheme 4) Employees State Insurance Scheme
5) Pradhan Mantri Jeevan Jyoti Bima Yojana
[Explanation: Insurance companies in the last over five years made nearly Rs 40,000
crore under the Centre’s flagship crop insurance scheme, shows the compiled data of the
Pradhan Mantri Fasal Bima Yojana (PMFBY) since its inception in 2016-17 to Kharif (summer
crops) 2021-22. PMFBY was launched in 18th February 2016.]

JULY- 28 Visitus:www.sr eedharscce.com


Sreedhar’s C C E CURRENT AFFAIRS -JULY-2022
25. Which company has donated $2mn to workforce development, education NGOs in
India?
1) Oracle Corporation 2) Amazon 3) IBM
4) Microsoft 5) Salesforce
[Explanation:Salesforce, a US-based tech major, has announced an investment of $2
million in direct company donations to over 22 non-profits across India. With a focus on
education and workforce development, these grants are expected to impact 40,000
individuals across India, at its annual flagship event ‘Salesforce Live’ in India.]
26. CCI has approved Google’s Proposed 1.28 Per Cent Stake Purchase in ______.
1) Bharti Airtel 2) Reliance Jio 3) Paytm
4) Tata Play 5) Flipkart

r’s
[Explanation: Google International LLC and Airtel have entered into an investment
agreement to buy a minority and non-controlling stake of 1.28 per cent of equity share
capital in the telecom player. The parties also intend to enter into certain other commercial
arrangements in future.]
27. HDFC and which of the following bank merger proposal got nod from the stock
exchanges? ha
1) IDFC First Bank 2) HDFC Bank 3) CSB Bank
4) Dhanlaxmi Bank 5) Karnataka Bank
[Explanation: The proposed merger of HDFC Bank and its parent firm HDFC NSE -
2.78 % is another step closer to its amalgamation as the proposal has been accepted by
stock exchanges National Stock Exchange and Bombay Stock Exchange, while noting that

E
they have “no objection”. Shares of HDFC Bank closed at Rs 1,355, up 0.5 per cent,
whereas that of HDFC at Rs 2,205, up 1.6 per cent.]
d
28. Which Bank has signed with JC Flowers ARC as a partner for sale of ?48,000
crore worth stressed loan portfolio?
1) HDFC 2) ICICI 3) YES Bank 4) AXIS Bank 5) IndusInd Bank
ee

[Explanation: The term sheet has now become effective as on 15 July 2022. The Bank
has decided that the JC Flowers ARC will be base bidder for a proposed sale of an identified
stressed loan portfolio of the Bank aggregating to up to Rs. 48,000 crores. The bank has
announced that it will sell bad loans worth ?48,000 crore at a value of ?11,100 crore. After
C
the sale, the gross NPA of the bank will drop to below 2% from the current 14% level.]
29. Which authority has banned hotels and restaurants from levying a service charge
Sr

on bills in India?
1) NFRA 2) CCPA 3) Ministry of Finance 4) CAG 5) DEA
[Explanation: Central Consumer Protection Authority (CCPA) banning restaurants and
hotels across the country from levying service charge by default on food bill and allowed
customers to file complaints in case of a violation of the norms. There should not be any
collection of service charge by any other name.The guidelines empower consumers to
complain against the practice under various provisions of the Consumer Protection Act,
tagging it as an ‘unfair trade practice,’ and a violation of consumer rights.]
30. __________ has issued a standard operating procedure (SOP) for e-commerce exports
C
of jewellery through courier mode.
1) RBI 2) Ministry of Finance 3) Exim Bank
4) CBIC 5) CBDT
[Explanation: CBIC has issued a simplified regulatory framework for e-commerce exports
of jewellery on June 30. CBIC standard operating procedure details the handling, movement
and procedural aspects for such exports, based on electronic declarations, through
international courier terminals. India’s gems and jewellery exports rose by 21.41 per cent
to ?25,295.69 crore (USD 3,241.38 million) in June compared to the same month last
year.]
31. Which of the following has approved $1 bn loan to strengthen India’s health sector?
1) World Bank 2) ADB 3) NDB 4) AIIB 5) IMF
[Explanation:World Bank has to assist the Pradhan Mantri-Ayushman Bharat
Health Infrastructure Mission (PM-ABHIM), which was launched by the government
in October 2021 for the development the public healthcare infrastructure across
the country.Two complementary loans totaling $500 million each were authorised
by the World Bank Board of Executive Directors to help and develop the health
sector in India.]

JULY- 29 Visitus:www.sr eedharscce.com


Sreedhar’s C C E CURRENT AFFAIRS -JULY-2022
32. Which Bank has approved a $96.3 million loan to provide safe drinking water in
Himachal Pradesh?
1) World Bank 2) AIIB 3) IMF 4) ADB 5) NDB
[Explanation: The loan has been provided under the Himachal Pradesh Rural Drinking
Water Improvement and Livelihood Project.The ADB project will connect 75,800 households
to the service, providing uninterrupted water supply to about 370,000 residents across
10 districts. ADB: - HQ: Mandaluyong, Philippines, 1966. President: Masatsugu Asakawa,
Vice-President: Ashok Lavasa.]
33. Which of the following has approved loan of $1.75 billion fund to the Prime Minister’s
Ayushman Bharat scheme?
1) ADB 2) AIIB 3) WHO 4) World Bank 5) NDB

r’s
[Explanation:World Bank has approved loans totaling USD 1.75 billion (about Rs
13,834.54 crore) to fund India’s PM Ayushman Bharat scheme and private investment in
order to boost the economic growth.Out of the total loan, USD 1 billion will go towards the
health sector, while the rest USD 750 million will be in the form of development policy loan
(DPL) to fill the financing gaps through private sector investment in the economy.]
34. Which bank has approved a loan of $ 79 million for the Meghalaya Ecotourism

1) World Bank 2) IMF


ha
Infrastructure Development Project?
3) ADB 4) AIIB 5) NDB
[Explanation: New Development Bank (NDB) has just approved five new projects totaling
USD 875 million for water, sanitation, ecotourism and transport. Investments will support

E
commitments by Brazil, China and India towards the Sustainable Development Goals
(SDGs). In Brazil, NDB will channel USD 300 million that will contribute to connecting 592
d
thousand households to water supply. In China, NDB approved EUR 265 million for the
expansion of the Lanzhou Zhongchuan International Airport.]
35. Pradhan Mantri Formalization of Micro food processing Enterprises (PMFME)
ee

Scheme completed _________ years recently.


1) Six 2) Two 3) Four 4) Five 5) Three
[Explanation:PMFMEScheme of M/o Food Processing Industries, under Aatmanirbhar
Bharat Abhiyan, Completed Two Years. PMFME Scheme is currently being implemented
in 35 States and Union Territories in the country. PMFME Scheme envisages financial
C
support of Rs. 40,000 for working capital and purchase of small tools for each member of
Sr

the Self-Help Group (SHG) engaged in food processing activitiesOver 1 lakh SHG members
have been identified and the seed capital amount of Rs. 203 Cr. has been released so
far.]
36. The Union Home Ministry has amended certain rules related to FCRA allowing
Indians to receive up to ______ in a year from kin staying abroad without disclosing.
1) Rs 1 lakh 2) Rs 2 lakh 3) Rs 5 lakh 4) Rs 8 lakh 5) Rs 10 lakh
[Explanation:The Union Home Ministry has amended certain rules related to the Foreign
Contribution (Regulation) Act (FCRA) allowing Indians to receive up to Rs 10 lakh in a year
from relatives staying abroad without informing the authorities. The earlier limit was Rs 1
C
lakh.If the amount exceeds, the individuals will now have 90 days to inform the government
instead of 30 days earlier.]
37. In which month, India has recorded its fastest growth in 11 years in services?
1) March 2022 2) July 2022 3) April 2022 4) June 2022 5) May 2022
[Explanation:India’s dominant services industry expanded at the fastest pace in over
eleven years in June amid strong demand but stubborn inflation remains a concern as
prices charged rose at the sharpest rate in almost five years. The S&P Global India Services
Purchasing Managers’ Index rose to 59.2 in June from 58.9 in May.]
38. Govt. has Launched Coking Coal Mission to Meet the Demand of Domestic Coking
Coal Target of 140 million Ton Raw Coking Coal Production by which year?
1) 2035 2) 2024 3) 2027 4) 2030 5) 2045
[Explanation: Country’s domestic raw coking coal output may touch 140 million tonnes
(MT) by 2030 from the present 51.7 MT. Coking coal is an essential raw material for the
production of iron and steel. To further increase the output of raw coking coal, the Centre
has auctioned 10 coking coal blocks to the private sector, with a peak rated capacity (PRC)
of 22.5 MT during the last two years. Most of these blocks are expected to start production
by 2025.]

JULY- 30 Visitus:www.sr eedharscce.com


Sreedhar’s C C E CURRENT AFFAIRS -JULY-2022
39. The amount of fund earmarked for PM-Daksh Yojana for FY 2022-23 is Rs.
____Crore.
1) 65 2) 84 3) 72 4) 45 5) 56
[Explanation: PM-DAKSH Portal and PM-DAKSH Mobile App have been launched by
Department of Social Justice and Empowerment on 07 August 2021. PM-DAKSH (Pradhan
MantriDakshtaAurKushaltaSampannHitgrahi) Yojana is a National Action Plan for skilling
of marginalized persons covering SCs, OBCs, EBCs, DNTs, Sanitation workers including
waste pickers.]
40. India’s FDI Flow in Manufacturing Sector Increases to _______Billion in FY22.
1) $34 2) $21 3) $26 4) $29 5) $31
[Explanation:India is rapidly emerging as a preferred country for foreign investments in

r’s
the manufacturing sector.Foreign direct investment (FDI) Equity inflow in Manufacturing
Sectors have increased by 76% in FY 2021-22 (USD 21.34 billion) compared to previous
FY 2020-21 (USD 12.09 billion).]
41. The total number of recognised startups in India rose from 471 in 2016 to ___ in
2022.
1) 72,993 2) 64,552 3) 74,672 4) 58,983 5) 89,321
ha
[Explanation: Startups and the entire technology ecosystem are the engines of growth
for any country. Recognizing this aspect, the Government has launched Startup India
initiative on 16th January 2016 with an aim to build a stronger ecosystem for nurturing
India’s startup culture. DPIIT has recognised startups which are spread across 56

42.
diversified sectors.]

E
which company has partnered Athletics Federation of India to enable the holistic
d
growth of athletics in India?
1) TCS 2) ITC Limited 3) Reliance Industries Ltd (RIL)
4) Google 5) Microsoft
[Explanation: The partnership aims to discover, nurture and develop Indian athletes
ee

from across the country and provide them with facilities, coaching and sports science and
medicine support by leveraging the foundation’s ecosystem.The partnership will focus on
girl athletes and aims to bridge the gender divide and enable them to achieve their dreams.
The ‘Reliance’ brand will appear across the national team’s jerseys and training kits at
C
key national, international competitions and training camp.]
43. Which of the following has signed agreement to acquire BASE life science for
Sr

€110 million?
1) Amazon 2) Park plus 3) Infosys 4) Wipro 5) Apple
[Explanation: Infosys bought BASE Life Science, a company based in Denmark, for
around 110 million euros (roughly Rs. 875 crore). The acquisition will expand Infosys’
knowledge in the life sciences industry and expand its presence in Europe. This purchase
strengthens Infosys ‘broad life sciences expertise, increases our foothold in the Nordics
and across Europe, and scales our industry-specific cloud-based digital transformation
solutions.]
C
44. Which of the following is the only country to administer 2 billion covid-19 vaccine
doses?
1) USA 2) India 3) Japan 4) China 5) Russia
[Explanation: India has administered 2 billion doses of the COVID-19 vaccine so far
under the nationwide vaccination drive. In order to ensure protection against COVID-19,
Union Health Minister Mansukh Mandaviya launched the “Covid Vaccination Amrit
Mahotsav” at the Covid Vaccination Camp in Nirman Bhawan, Delhi.]
45. Tata Steel has planned capital expenditure (capex) of Rs __________ on its India
and Europe operations in FY23
1) 12,000 crores 2) 10,000 crores 3) 15,000 crores
4) 8,000 crores 5) 20,000 crores
[Explanation: The domestic steel major plans to invest Rs 8,500 crore in India and Rs
3,500 crore on the company’s operations in Europe, Narendran, who is also the MD of
Tata Steel. In India, the focus will be on the Kalinga Nagar project expansion and mining
activity, and in Europe, it will be focused on sustenance, product mix enrichment and
environment-related capex, Narendran. The company is in process of expanding capacity
of its plant in Kalinga Nagar, Odisha to 8 MT from 3 MT.]

JULY- 31 Visitus:www.sr eedharscce.com


Sreedhar’s C C E CURRENT AFFAIRS -JULY-2022
46. Ola Electric has invested ___________ in Bengaluru based cell R&D facility.
1) $300 mn 2) $250 mn 3) $500 mn 4) $150 mn 5) $400 mn
[Explanation: Ola Electric is investing USD 500 million (about Rs 4,000 crore) in a cell
research and development facility in Bengaluru. SoftBank Group-backed Ola Electric is
also in talks with multiple global suppliers to build a battery cell manufacturing plant in
India with a capacity of up to 50 gigawatt hours. Ola Electric needs 40 Gwh of battery
capacity to power 10 million electric scooters annually. The remainder will be for its electric
cars, which the company plans to manufacture in the future. Founder and CEO: Bhavish
Aggarwal.]
47. Fintech startup, ‘OneCard’ has become the India’s _____ unicorn rising $100 mn
funding.

r’s
1) 100th 2) 105th 3) 120th 4) 104th 5) 116th
[Explanation: OneCard, a mobile-first credit card company, raised $100 million in a
series D round of funding backed by Temasek, making it the 104th unicorn in India. India
has created more than 20 financial unicorns so far in 2022, including OneCard, Open,
Oxyzo, and Yubi (formerly CredAvenue). OneCard which is owned by FPL Technologies
ha
of Pune.]
48. TATA AIG Life Insurance has partnered with which bank recently to offer life and
health insurance solutions to the Bank’s consumers?
1) HDFC 2) State Bank of India 3) City Union Bank
4) Indian Bank 5) Axis Bank

E
[Explanation: Tata AIA Life Insurance Co. Ltd has announced a strategic alliance with
d
City Union Bank (CUB) to offer life and health insurance solutions to the Bank’s consumers.
This partnership brings together two respected consumer centric brands that have been a
part of India’s consumer landscape for more than a century. This partnership will help
City Union Bank consumers (existing and new) to benefit from Tata AIA Life’s diverse and
ee

innovative products and services spanning term insurance, savings and investment,
retirement, and health.City Union BankHQ: Kumbakonam, Tamil Nadu.]
49. Which of the following Mutual Fund has launched ‘Focused Equity Fund’?
C
1) SBI Mutual Fund 2) ICICI Prudential Mutual Fund
3) Axis Mutual Fund 4) Edelweiss Mutual Fund
Sr

5) HDFC Mutual Fund


[Explanation: Edelweiss Asset Management Ltd has launched a new fund offer (NFO)
for Edelweiss Focused Equity Fund, that will invest in 25-30 stocks spread across three
key investing opportunities of brands, market share gainers, and innovators. The
benchmark for the fund will be Nifty500 Total Return Index. The Edelweiss Focused
Equity Fund will open for subscription between 12 and 25 July and offers both direct and
regular plans.]
50. Which of the following is principal sponsor of Indian Olympic Association?
C
1) Adani SportsLine 2) Tata SportsLine 3) Vivo
4) INOX Group 5) Edelweiss
[Explanation: Adani Sportsline, the sports arm of the Adani Group, has become an
official partner with the Indian Olympics Association (IOA) for the upcoming Birmingham
Commonwealth Games 2022, Hangzhou Asian Games 2022, and Paris Olympic Games
2024. IOA HQ: New Delhi.]
51. HDFC Securities has joined hands with which small Finance Bank to offer demat,
trading services?
1) Suryoday Small Finance Bank 2) ESAF Small Finance Bank
3) Equitas Small Finance Bank 4) Fincare Small Finance Bank
5) AU Small Finance Bank
[Explanation: With the new partnership, Equitas Small Finance Bank will offer its
customers a “3-in-1” account for accessing the services of HDFC Securities. Equitas SFB
customers would be able to open their demat and trading accounts for buying and selling
shares and trading in futures, options and even currencies.primary objective in forming
this alliance is to empower the customers of Equitas SFB with a robust investment platform.
Equitas SFB HQ: Chennai, TN.]

JULY- 32 Visitus:www.sr eedharscce.com


Sreedhar’s C C E CURRENT AFFAIRS -JULY-2022
52. Which General Insurance has launched the “Cyber VaultEdge” for protecting
individuals against financial losses from cyber risks?
1) Edelweiss General Insurance 2) Bharti AXA General Insurance
3) Aditya Birla General Insurance 4) SBI General Insurance
5) Bajaj Allianz General Insurance
[Explanation: SBI General Insurance has launched the Cyber VaultEdge insurance plan,
a comprehensive cyber insurance cover for individuals that provides protection against
financial losses arising from cyber risks and attacks. The policy takes care of legal costs
and expenses incurred in pursuing or defending legal action against/by a third-
party.Further, the policy reimburses the expenses incurred for the services of an IT specialist
or to restore data.]

r’s
53. Which is the first private entity to enter highly-regulated opium processing sector?
1) Bajaj Healthcare 2) Aster DM Healthcare Ltd.
3) Narayana Hrudayalaya Ltd 4) Care Health Insurance Ltd
5) Piramal Enterprises Ltd
[Explanation:Pharmaceutical company Bajaj Healthcare Limited has become the first
ha
private player in India to be awarded a government contract for the highly-regulated opium
processing sector — a development that means the company can use opium to extract
active pharmaceutical ingredients (APIs) used in medicines such as painkillers, cough
syrups, and even cancer drugs.]
54.

E
Paschim Banga Gramin Bank has joined with __________ for a bancassurance?
1) Bajaj Allianz Life Insurance 2) SBI Life Insurance
d
3) Bharti AXA Life Insurance 4) Birla Sun Life Insurance
5) Aviva Life Insurance
[Explanation: The partnership will see all the branches of Paschim Banga Gramin Bank
across West Bengal offer SBI Life’s range of protection, wealth creation, credit life, annuity
ee

and savings products, thereby increasing accessibility of life insurance solutions in the
region. Bancassurance is an arrangement between a bank and an insurance company, as
a part of which the latter can sell its products to the lender’s customers.]
C
55. Which General Insurance Co. Ltd. has launched a floater motor insurance policy?
1) HDFC ERGO General Insurance 2) ICICI Lombard General Insurance
Sr

3) Bajaj Allianz General Insurance4) TATA AIG General Insurance


5) SBI General Insurance
[Explanation:ICICI Lombard has launched a slew of innovative solutions — Motor Floater
Insurance, Pay-As-You-Use and Pay-How-You-Use – to further enrich customer experience
in motor insurance.The Motor Floater offer is in line with the recent announcement made
by IRDAI. It will enable those individuals owning multiple vehicles to ensure their vehicles,
including cars and two-wheelers, under one policy with a single renewal date and a
comprehensive cover.]
C
56. Which General insurance has launched its add-on motor insurance cover called
‘pay as you consume’ (PAYC)?
1) Bharti Axa General Insurance 2) Edelweiss General Insurance
3) Bajaj Allianz General Insurance 4) Go Digit General Insurance
5) Aditya Birla General Insurance
[Explanation: The company was the first to launch ‘Pay As You Consume’ under the
Irdai’s sandbox regulations and owing to the overwhelming response from customers,
Bajaj Allianz General Insurance launched this as a full-fledged cover under motor insurance
products.]
57. Which company has reported turnover of ?61,000 crore for the fiscal 2021-22?
1) Dodla Diary 2) Amul 3) Heritage Foods
4) Hatsun Agro 5) Britannia Industries
[Explanation: Dairy major and Amul marketer, Gujarat Cooperative Milk Marketing
Federation (GCMMF), has reported group turnover of ?61,000 crore for the fiscal 2021-
22. The dairy federation held its 48th Annual General Body Meeting. GCMMF is a federation
of 18 members district dairy unions of Gujarat. Turnover grew by 18.46% YoY - faster
than its 12-year CAGR of 16%.]

JULY- 33 Visitus:www.sr eedharscce.com


Sreedhar’s C C E CURRENT AFFAIRS -JULY-2022
58. Which of the following has signed (MoU) with the Karnataka Skill Development
Corporation (KSDC)?
1) Myntra 2) Flipkart 3) Facebook 4) Snapdeal 5) Amazon
[Explanation:This MOU was to launch the initiatives of the Supply Chain Operations
Academy (SCOA) in the state.This initiative will help in creating a pool of skilled supply
chain operations and provide relevant industry training and knowledge.Flipkart has
launched an industry-first Centre of Excellence (CoE) to build a skilled and trained
workforce for the fast-growing e-commerce industry in India.]
59. _____________has expanded its News Initiative Training Network in India to five
more languages.
1) Microsoft 2) Amazon 3) Google 4) TCS 5) Infosys

r’s
[Explanation: This Google initiative helps teach journalists and newsrooms learn digital
skills needed to verify and tackle misinformation online and will now be available in Punjabi,
Assamese, Odia, Gujarati, and Malayalam. Google News Initiative India Training Network
was launched four years ago in partnership with Data Leads and the company that till
date it has organised more than 700 workshops and trained more than 39,000 journalists
and media colleges in at least 10 languages.]
60.

1) Apple
ha
Which of the following company has announced Startup School in India for small-
city entrepreneurs?
2) Microsoft 3) Dell 4) Google 5) IBM
[Explanation:Google has announced a Startup School India aimed at enabling 10,000

E
startups in tier 2 and tier 3 cities understand the dynamics of entrepreneurship. The nine-
week virtual programme will involve fireside chats between Google leaders and
d
collaborators from across the startup ecosystem.]
70. NASSCOM has joined with which company for ‘Digi Vaani Call Center’ project?
1) Microsoft 2) Apple 3) Wipro 4) Google 5) TCS
[Explanation: Nasscom Foundation and Google have joined hands to set up a call centre
ee

in collaboration with a not-for-profit body Indian Society of Agribusiness Professionals


(ISAP) to help women farmers scale up their businesses. DigiVaani Call Center will act as
one place where rural women entrepreneurs will be able to call and get information about
various schemes available for them, which can help them scale up their businesses, be it
C
government schemes or any other information that can be of help for them.]
71. Neelachal Ispat Nigam Ltd (NINL) was handed over to which Indian conglomerate
Sr

group?
1) Adani Group 2) Croma 3) Bigbasket 4) Relianc 5) Tata Group
[Explanation:NINL is the second successful privatization by the current government.
Incidentally, Air India — the first company on the privatization list — too was bought by
Tata Group. NINL Strategic Disinvestment transaction has been completed with the transfer
of 93.71 per cent shares of the joint venture partners to the Strategic Buyer, M/s Tata Steel
Long products Ltd.]
72. Which of the following has launched ‘5G Innovation Lab’ in India?
C
1) Siemens Healthineers 2) Medtronic 3) Philips 4) GE Healthcare
5) Canon Medical System Corporation
[Explanation:GE Healthcare, a global medical technology, diagnostics, and digital solutions
innovator, announced the launch of its 5G Innovation Lab in India and across the globe.
The lab is designed to position GE Healthcare advances in patient care and to bring cutting-
edge technology to rural and sub-urban regions. HQ of GE Healthcare: Chicago, Illinois,
United States.]
73. Which among the following has developed ‘EV Mitra’ mobile app to provide
information on EV charging stations?
1) Tata Powe 2) B.BESCOM 3) Techser Power Solutions
4) SJVN Ltd 5) B B M P Electrical
[Explanation:The Bengaluru Electricity Company BESCOM has developed EV Mitra mobile
app which will provide information about EV charging stations in Bengaluru. electric vehicle
users can use EV Mitra mobile app to book in advance a slot in the station. BESCOM also
launched the EV Jagruthi Web portal as a joint initiative of NITI Ayog and the United Kingdom
to provide state-specific information on the availability of electric vehicles, incentives, support
mechanisms and initiatives of the state government regarding electric mobility.]

JULY- 34 Visitus:www.sr eedharscce.com


Sreedhar’s C C E CURRENT AFFAIRS -JULY-2022
74. Which company has announced that demat account mandatory for MF investment?
1) PhonePe 2) BharatPe 3) Google Pay 4) Paytm 5) Mastercard
[Explanation:Paytm Money has announced that it has extended the timeline for its direct
mutual fund users to update their KYC and open a Demat account by 31st October. Demat
account will remain free for life for investments in direct mutual funds,”. Paytm Money
CEO Varun Sridhar.Paytm Money it continues to offer direct mutual funds on the Paytm
Money app, with no change in investor experience even after integration with BSE StAR.HQ:
Noida, Uttar Pradesh.]
75. Which among the following has launched the biggest digital initiative for exams
“PARIKSHA SANGAM”?

r’s
1) AICTE 2) NCERT 3) UGC 4) NCTE 5) CBSE
[Explanation:CBSE Board has launched a portal called ‘Pariksha Sangam’ to streamline
board examination results, sample papers, and other details in one single window.
According to cbsedigitaleducation.com, the newly launched Pariksha Sangam portal will
integrate various examination-related processes to be conducted by the school’s regional
offices and the Headquarters of the CBSE Board.]
76.
Cooperation (JBIC)?
ha
Which inked a loan pact for JPY 30 billion with Japan Bank for International

1) Power Finance Corporation 2) NTPC


3) REC Ltd 4) PTC India Financial Services
5) ONGC

E
[Explanation:JBIC has provided this long-term facility to PFC under its initiative ‘Global
d
action for Reconciling Economic growth and Environmental preservation’ (GREEN), wherein
JBIC provides financing for projects which ensure the effective reduction of greenhouse
gas emissions and conservation of the global environment. PFC Chairman & MD: R S
ee

Dhillon.]
77. Which Company will be the world’s first intermodal partner of the star Alliance?
1) DBS Bank 2) Deutsche Bahn 3) KFW Bankgruppe
4) Norddeutsche Landesbank (Nord/LB) 5) Commerzbank
C
[Explanation: Deutsche Bahn (DB) will be the world’s first Intermodal Partner of Star
Alliance. With this, DB and the aviation industry are sending another strong signal for the
Sr

environment-friendly evolution of the travel industry. Under the new cooperation, DB


customers and passengers of Star Alliance member airlines will be able to start or end
their long-distance journey comfortably on the climate-friendly train. Germany is the first
market and DB is the world’s first partner in the new Star Alliance initiative.]
78. IRDAI has set up a 9-member committee to resolve issues between non-life
insurers and re-insurers under Chairmanship of ___________.
1) Tapan Kumar Singhel 2) Naveen Tahilyani 3) Bhargav Dasgupta
C
4) Mayank Bathwal 5) S. Srinivasan
[Explanation:The panel on life insurance has been asked to study and make
recommendations on stabilisation of reinsurance rates, capacity constraints with reinsurers,
securitisation of mortality risk and financial reinsurance solutions, among others.The panel
headed by Dasgupta has been tasked to suggest steps on applicability of compliance
requirement, and faster settlement and payment mechanism among the re-insurers and
cross border re-insurers (CBRs), among others.The task forces have been asked to file
their reports within three weeks.IRDAIHQ: Hyderabad, Founded in 1999.]
79. Which company has announced a five-year engagement with Nokia for digital
transformation?
1) TCS 2) Wipro 3) Microsoft 4) Infosys 5) Tech Mahindra
[Explanation: Wipro has announced a five-year engagement with global networking and
telecom player Nokia. Wipro will deliver business services in support of Nokia’s renewed
operating model, focusing on process optimization, touchless processing and enhanced
user and customer experience across order management supply chain, finance, and
accounting operations, according to the Bengaluru-based IT company’s exchange filing.]

JULY- 35 Visitus:www.sr eedharscce.com


Sreedhar’s C C E CURRENT AFFAIRS -JULY-2022
80. Google Maps has launched Street View Project or Project Gullify in India in
collaboration with which company?
1) Microsoft 2) Tech Mahindra 3) Wipro 4) TCS 5) Samsung
[Explanation: Google has launched its 360-degree interactive panorama feature for 10
Indian cities with data from local partners Tech Mahindra and Mumbai-based Genesis
International. The feature has been rolled out for the city of Bengaluru currently, with
releases in Hyderabad and Kolkata. Tech Mahindra will be responsible for GIS (Geographic
Information Systems) processes from data creation, resourcing, insights generation, and
the actual collection of the street-level imagery.]
81. Which of the following has set to replace Paytm as title sponsor for all BCCI
matches in India?

r’s
1) RuPay 2) PayPal 3) Visa
4) American Express. 5) Mastercard
[Explanation:The global payments major, Mastercard, is reportedly set to replace
homegrown Paytm as the title sponsor for BCCI’s international and domestic cricket matches
till 2023, after Paytm reportedly requested a withdrawal from the deal.
ha
Mastercard will reportedly be assigned the rights till 2023, as per the original terms of the
Paytm deal and will continue to pay INR 3.8 Cr per match. Paytm, on the other hand, will
have to pay a re-assignment fee of around INR 16.3 Cr.]
82. Which among the following company has acquired Germany-based automation
tech firm Hochrainer?

E
1) Wipro PARI 2) Siemens India Ltd. 3) Larsen and Toubro
d
4) Voltas Ltd 5) Rockwell Automation
[Explanation: Wipro PARI, the industrial automation business of Wipro Infrastructure
Engineering (a Wipro Enterprises entity), has signed a definitive agreement to acquire
Hochrainer GmbH, based in Freilassing, Germany. Hochrainer GmbH, a family-owned
ee

company, has been in operations since 1973. The company has established itself as an
important supplier of automation technology and assembly systems for numerous global
automobile manufacturers. Wipro HQ: Bengaluru.]
C
83. Which company has launched the ‘Viva Engage’ app?
1) Wipro 2) TCS 3) Microsoft 4) Amazon 5) Apple
Sr

[Explanation: Tech giant Microsoft has announced that it is introducing Viva Engage, a
new app in Teams that helps build community and connection, along with providing tools
for personal expression. It was Built on the foundation of Yammer, Viva Engage brings
people together across the organisation to connect with leaders and coworkers, find
answers to questions, share their unique story, and find belonging at work.]
84. Nokia has partnered with _________to set up networked robotics center of
excellence?
1) IIT Madras 2) IISc Bengaluru 3) IIT Mumbai
C
4) IIT Varanasi 5) IIM Ahmedabad
[Explanation:Nokia has partnered with Indian Institute of Science to set-up Nokia Center
of Excellence in networked robotics at IISc Bengaluru. The Center of Excellence (CoE) will
promote inter-disciplinary research involving robotics and advanced communication
technologies in 5G and Artificial Intelligence (AI). Nokia will fund the CoE for three
consecutive years in order to sustain the first phase of the partnership between Nokia and
Indian Institute of Science (IISc).]
85. Greenko has signed with ___________ to launch India’s first dedicated school for
sustainable science and technology.
1) IIT Kanpur 2) IIT Khragpur 3) IIT Hyderabad
4) IIT Bombay 5) IIT Madras
[Explanation:‘Greenko School of Sustainable Science and Technology’ (GSSST) aims to
plug the rising research, education, and skills gaps caused by the imperatives of sustainable
development.The school will be advancing knowledge in the key thrust areas of climate
change mitigation, artificial intelligence and space technology, energy transition and
industrial transformation, circular and regenerative economy among others.]

JULY- 36 Visitus:www.sr eedharscce.com


Sreedhar’s C C E CURRENT AFFAIRS -JULY-2022

7.INTERNATIONAL
1. India has signed with which country for building strategic relationship with focus
on defence & security?
1) France 2) Egypt 3) Russia 4) Ukraine 5) USA
[Explanation:India and Egypt are charting out an increasingly strategic bilateral
relationship with more focus on defence and security aspects. India and Egypt are now
partnering with each other across all spectrums of their bilateral ties based on a sound
historical and traditional relationship. From exporting wheat to Egypt despite a ban on its

r’s
shipments to greater military-to-military relationship, India is leaving no stone unturned to
bring Egypt under its strategic embrace.]
2. World’s most powerful particle collider “Large Hadron Collider” has been located
in which country?
1) Switzerland 2) Russia 3) China 4) Japan 5) USA
ha
[Explanation: The world’s most powerful particle collider, the Large Hadron Collider
(LHC), is set to function. It was built by the European Organization for Nuclear Research
located in Geneva. The Large Hadron Collider is a giant, complex machine built to study
particles, the smallest known building blocks of all things. AfterThree years it was shut

E
down for maintenance and upgrades, the collider was switched back recently.]
d
3. What is the Name of the Kid goat, which was born recently in Pakistan with
longest ears in the world?
1) Karna 2) Simba 3) Jika 4) Valika 5) Badra
ee

[Explanation: The goat named Simba was born in Sindh on June 5. The goat astounded
its owner Muhammad Hassan Narejo when it was born with its unusually long ears. She
has already become a local celebrity in Pakistan.They dangle down on either side of her
face and wiggle in the wind.Simba’s two ears each measure 21 inches long.]
C
4. Which of the following Countries are likely to be the new members of SCO?
Sr

1) Iran 2) Belarus 3) Mongolia 4) Both 1 & 2 5) Both 1 & 3


[Explanation: The Shanghai Cooperation Organisation is all set for expansion with Iran
joining the grouping as its ninth member while Belarus has applied for membership,
Shanghai Cooperation Organisation (SCO) summit 2022 is planned to held in September
2022 in Samarkand, Uzbekistan. India will host the SCO summit 2023 next year. Varanasi
has been selected as the SCO region’s first “Tourism and Cultural Capital”, a title it will
hold next year coinciding with India chairing the SCO grouping.]
5. Which of the following country has introduced “National Fuel Pass”, a fuel rationing
C
scheme?
1) Ukraine 2) Russia 3) China 4) Sri Lanka 5) India
[Explanation: The new pass will guarantee the allocation of the fuel quota on a weekly
basis. A QR code will be given for each National identity Card number (NIC), and turns for
getting fuel will be allocated on the basis of the last four digits of the NIC card holder’s
card number. Sri Lanka Capitals: Colombo, Sri Jayawardenepura Kotte. Currency: Sri
Lankan rupee.]
6. Which country has launched ‘Aviation Passenger Charter’ to help passengers
know their rights ?
1) Japan 2) UK 3) Mexico 4) Ecuador 5) Chile
[Explanation: To help passengers know their rights if they are faced with problems at
airports after the widespread disruption. The new charter will help passengers know
what to do if they are confronted by cancellations, delays or missing baggage. It has been
developed by the British government in partnership with the aviation sector & travel
industry.]

JULY- 37 Visitus:www.sr eedharscce.com


Sreedhar’s C C E CURRENT AFFAIRS -JULY-2022
7. Which country has become a major natural rubber supplier to India?
1) Ivory Coast 2) Gambia 3) Ghana 4) Nigeria 5) Senegal
[Explanation: The African nation is now the fourth largest producers of the industrial
commodity.India’s natural rubber imports from Ivory Coast (Cote d’Ivoire) made up 28 per
cent of total shipments into the country in May, making the West African country a major
competitor to many South-East Asian nations in the supply of the key raw material for the
tyre and rubber industries. Capital Yamoussoukro. Currency: West African CFA franc.]
8. Which country has confirmed its first outbreak of highly infectious Ebola-like
Marburg virus ?
1) Gambia 2) Mali 3) Nigeria 4) Ghana 5) Senegal

r’s
[Explanation: Marburg virus disease (MVD), earlier known as Marburg haemorrhagic
fever, was first detected in 1967. It is a severe, highly infectious, often fatal hemorrhagic
fever. Marburg, like Ebola, is a filovirus; and both diseases are clinically similar. Symptoms
- After the onset of symptoms, MVD can manifest itself in the form of high fever, muscle
aches and severe headache.]
9.
ha
India and ___________ has signed MoU for reintroduction of cheetahs into the
historical range in India.
1) Zambia 2) Namibia 3) Angola 4) Kenya 5) Chad
[Explanation: The first eight cheetahs are expected to arrive in Madhya Pradesh’s Kuno

E
National Park by August 15. Separately, India is anticipated to receive 12 cheetahs from
d
South Africa. Objective is to create a healthy meta-population of the animal in the nation that
will enable it to fulfil its functional role as a top predator and create room for it to spread out
within its historical range, supporting efforts to conserve it globally.]
ee

10. India has Signed MOU with which Country for Mutual Recognition of Educational
Qualifications?
1) Nepal 2) USA 3) UAE 4) UK 5) Russia
[Explanation: Including maritime education and a Framework agreement on the health
C
care workforce. In May 2021, during the Virtual Summit between Prime Ministers of India
Sr

and UK, a comprehensive Roadmap to 2030 was adopted for an enhanced bilateral
cooperation between the two countries. Both sides also agreed to a new Enhanced Trade
Partnership. Education forms an important pillar of this roadmap.]
11. India and __________ has signed an agreement to curb trans-border crimes.
1) Bhutan 2) Nepal 3) Bangladesh 4) Myanmar 5) Sri Lanka
[Explanation: The countries also decided to complete pending developmental work within
150 yards of the international border. The agreements were signed during the Director-
General level border co-ordination conference between India and Bangladesh, held in
C
Dhaka after two years.]
12. Which countries have signed an UN-backed deal to allow the export of millions of
tonnes of grain from blockaded Black Sea ports?
1) Russia 2) USA 3) Ukraine 4) Both A&C 5) Both B&C
[Explanation: The agreement potentially averted the threat of a catastrophic global food
crisis. Ukraine is one of the world’s largest exporters of wheat, corn and sunflower oil,
but Russia’s invasion of the country and naval blockade of its ports have halted shipments.
Some grain is being transported through Europe by rail, road and river.]
13. India and __________ has signed MoU on judicial cooperation ?
1) Male 2) Sri Lanka 3) Seychelles 4) Madagascar 5) Maldives
[Explanation: India and Maldives signed MoU for judicial cooperation to accelerate court
digitization and boost prospects for IT companies & start-ups in both countries. India was
among the first countries to recognize the Maldives after its independence in 1965 and to
establish diplomatic relations with the country.]

JULY- 38 Visitus:www.sr eedharscce.com


Sreedhar’s C C E CURRENT AFFAIRS -JULY-2022

14. Which among the following countries has received ICC Membership status ?
1) Cambodia 2) Uzbekistan 3) Cote D’Ivoire 4) Both A&B 5) All A&B&C
[Explanation: ICC’s total members to 108 countries, including 96 Associates. A major
criteria to gain ICC Membership is having in place a proper structure with minimum team
requirements for 50-over and 20-over tournaments, aside from clear junior and women’s
pathways. ICC headquarters: Dubai, United Arab Emirates.]
15. Largest pink diamond in 300 years has found in __________.
1) Angola 2) Mali 3) Mongolia 4) Peru 5) Paraguay
[Explanation: Pink diamonds belong to a subcategory called color diamonds. It is not yet

r’s
sure why they are pink in color; however, the prevailing explanation is that it is because of
the shear pressure during formation. The 59.6 carats Pink Star was sold at a Hong Kong
auction in 2017 for 71.2 million US dollars. It still remains the most expensive diamond
ever sold. Capital of Angola Luanda. Currency: Angolan Kwanza.]
16. Which country has decided to leave The International Space Station After 2024 ?
1) UK
ha
2) India 3) Russia 4) China 5) USA
[Explanation: The declaration comes amid heightened hostilities between Moscow and
the West over Moscow’s military action in Ukraine and several rounds of previously unheard-

E
of sanctions against Russia. The International Space Station (ISS), which has been in orbit
since 1998, has been jointly developed by Russia and the United States. Roscosmos
d
Chief: YuryBorisov, Russia President: Vl adimir Putin,Ukraine President:
VolodymyrZelenskyy.]
ee

17. Which country has signed MoU with India to set up a green hydrogen plant with
investments of $8 billion?
1) Egypt 2) Denmark 3) Nepal 4) Sri Lanka 5) Israel
C
[Explanation: ReNew is one of numerous Indian businesses exploring the potential of
green hydrogen, which is thought to be essential for decarbonizing difficult-to-reduce
Sr

heavy industries. In the upcoming years, ReNew plans to generate 2,20,000 tonnes of the
clean fuel yearly in Egypt with backing from investors including the Goldman Sachs Group
and the Abu Dhabi Investment Authority. Chairman of ReNew Power Private Limited:
Sumant Sinha.]
18. Which country has launched a gold coin namely ’Mosi-oa-Tunya’ as a legal tender
to control skyrocketing inflation?
C
1) New Zealand 2) Egypt 3) Zimbabwe 4) China 5) Taiwan
[Explanation: The governor of Zimbabwe’s central bank stated that the coins will be sold
based on the international price of gold and cost of production. They can be purchased in
Zimbabwean dollars, US Dollars, and other currencies starting July 25. The gold coins
will be called “Mosi-oa-tunya,” meaning “The Smoke Which Thunders,” which is a
reference to Victoria Falls.]
19. Who has released world’s most detailed map of the moon ?
1) USA 2) India 3) China 4) Japan 5) Russia
[Explanation: It is the most detailed to date, registering even finer details of the lunar
surface than mapped by the US in 2020. The new comprehensive geological map of the
moon released by China is to a scale of 1:2,500,000. It is the most detailed moon map to
date. The moon map includes the 12,341 impact craters, 17 rock types, 81 impact basins,
and 14 types of structures. The new significant details of moon map have provided
abundant information about the geology of the moon and its evolution.]

JULY- 39 Visitus:www.sr eedharscce.com


Sreedhar’s C C E CURRENT AFFAIRS -JULY-2022

8.NATIONAL
1. The ban on select single-use plastics by the Centre in order to reduce plastic
wastage comes into effect from ______.
1) January 01, 2023 2) August 01, 2022 3) July 01, 2022
4) September 01, 2022 5) February 01, 2023
[Explanation: The ministry is looking at a three-pronged strategy for banning single-use
plastic in India. High littering potential - This means products that are quickly thrown away

r’s
post usage.Low utility - These are plastic products that have the least amount of usage or
utility after being used. Availability of alternative - If other alternatives can be used in their
place.]
2. Narendra Modi has launched the year-long _________ birth anniversary celebration
of legendary freedom fighter Alluri Sitarama Raju?
ha
1) 115th 2) 125th 3) 150th 4) 175th 5) 100th
[Explanation: Prime Minister Narendra Modi stated that Alluri Sitarama Raju dedicated
his life to the rights of tribal society, their happiness and sorrow, and their freedom. He
stated that “Alluri Sitarama Raju represents the spirit of ‘Ek Bharat Shreshtha Bharat’ that

E
has been uniting the country in one thread of unity.” He was named “ManyamVeerdu”
which means Hero of the Jungle by the locals for his heroic exploits.He was born in Andhra
d
Pradesh and he was involved in the freedom fight against the British during the 1882
Madras Forest Act.]
ee

3. Who has inaugurated the Zonal Meeting on ‘8 Years’ Achievements – Impact on


Women and Children’ in Hyderabad?
1) Smriti Zubin Irani 2) Narendra Modi 3) Arjun Munda
4) K. Chandrashekar Rao 5) Dharmendra Pradhan
C
[Explanation: The Minister for Tribal Welfare, Women and Child Welfare of Telangana,
Sr

SmtSatyavati Rathod was also present in the review meeting. Beneficiaries of schemes
like ‘BetiBachoBetiPadhao’, ‘PM CARES’, “Sakhi One Stop Centres’, ‘PM MatritvaVandana
Yojana’ and “Poshan Abhiyan’ in Anganwadi centres participated the meeting and shared
their views.]
4. Who has inaugurated Akhil Bharatiya Shiksha Samagam in Varanasi?
1) Nirmala Sitharaman 2) Rajnath Singh 3) Amit Shah
4) Narendra Modi 5) Dharmendra Pradhan
C
[Explanation: The Shiksha Samagam is being put on by the Ministry of Education. Eminent
academics, policymakers, and academic leaders will have a forum to deliberate, exchange
their experiences, and discuss the National Education Policy (NEP) 2020 implementation
roadmap. More than 300 Academic, Administrative, and Institutional Leaders from
Universities, and Institutes of National Importance (IIT, IIM, NIT, IISER) from around the
nation are participating in the event as part of improving their capacities.]
5. Which state Govt has introduced India’s first “Right to Health Bill 2022”?
1)Andhra Pradesh 2) Karnataka 3) Haryana 4) Maharashtra 5) Rajasthan
[Explanation: It makes Rajasthan the first state in India to propose a guaranteed right to
healthcare to its residents. i. It aims to provide quality and affordable healthcare services
by govt and private healthcare providers without catastrophic out-of-pocket expenditure.
In January 2022, the draft of the Right to Health Bill was prepared. CM: Ashok Gehlot,
Governor: Kalraj Mishra, Ranthambore National Park, Sariska National Park,Desert National
Park, Keoladeo National Park, Mount Abu WLS.]

JULY- 40 Visitus:www.sr eedharscce.com


Sreedhar’s C C E CURRENT AFFAIRS -JULY-2022
6. Who has launched the India’s first-ever ‘Artificial Intelligence in Defence’ (AIDef)
symposium and exhibition?
1) Narendra Modi 2) Amit Shah 3) Rajnath Singh
4) Nitin Jairam Gadkari 5) Piyush Goyal
[Explanation: It sought to promote “Aatmanirbharta” in Defence sector. During the
exhibition, department will showcase AI-enabled solutions that have been developed by
Services, industry, research organisations, and start-ups & innovators. All these products
belong to the fields of automation or unmanned or robotics systems, human
behaviouranalysis, cyber security, logistics & supply chain management, intelligent

r’s
monitoring system, speech/voice analysis and C4ISR.]
7. Which authority has recommended two sites associated with Babsaheb Ambedkar
to be declared as Monuments of National importance?
1) National Archaeological Authority 2) Advertising Standards Council of India
3) National Monuments Authority
4) Express Industry Council of India 5) Bureau of Indian Standards
ha
[Explanation: The Culture Ministry said, Sankalp Bhoomi Banyan Tree Complex in
Vadodara, where Dr. Ambedkar took a pledge to eradicate untouchability on 23rd
September 1917, should be declared a monument of national importance. National

E
Monuments Authority has also recommended a place in Satara, Maharashtra where Dr.
Ambedkar received his primary education at Pratap Rao Bhosale High School to be declared
d
as a monument of national importance.]
8. Who has launched 75 newly-developed first -ever AI-enabled defence products in
Delhi?
ee

1) Narendra Modi 2) Anurag Thakur 3) Amit Shah


4) Raj Nath Singh 5) Amit shah
[Explanation: Amongst these, one of the most eye-catching product was the Silent Sentry
C
which is a key technology developed by the design bureau of the Indian Army to plug the
gaps in surveillance networks. At the event, products were launched under multiple
Sr

domains like cyber security, human behavioral analysis, block chain-based automation,
command, control, communication, computer & intelligence, surveillance and
reconnaissance, intelligent monitoring systems, speech, voice analysis using natural
language processing etc.]
9. India projected to surpass China as world’s most populous country in __________.
1) 2024 2) 2023 3) 2025 4) 2026 5) 2030
[Explanation: The latest projections by the United Nations have also projected that the
C
world’s population could grow to around 8.5 billion in 2030 and 9.7 billion in 2050. As
per the UN Report, the world population is growing at its slowest rate since 1950 as it has
fallen under 1% in 2020. World’s population is projected to reach a peak of around 10.4
billion people during the 2080s and to remain at that level till 2100.]
10. Who among the following has unveiled the new logo of the country’s Public Service
Broadcaster, ‘Prasar Bharati’?
1) Anurag Thakur 2) Narendra Modi 3) Ashwini Vaishnaw
4) Ram Nath Kovind 5) Apurva Chandra
[Explanation: The elements in the central circle and map of India signify the service of
trust, security, and perfection to the Nation, its colour, ‘Dark Moderate Blue’ represents
both the sky and the sea and is associated with open spaces, freedom, intuition,
imagination, inspiration, and sensitivity.Blue also represents meanings of depth, trust,
loyalty, sincerity, wisdom, confidence, stability, faith, and intelligence.Prasar Bharati
Founded on 23 November 1997, Headquarters: New Delhi.]

JULY- 41 Visitus:www.sr eedharscce.com


Sreedhar’s C C E CURRENT AFFAIRS -JULY-2022
11. Andhra Pradesh Govt has decided to rejoin the Pradhan Mantri FasalBima Yojana
(PMFBY). In which year PMFBY was launched?
1) 2018 2) 2016 3) 2015 4) 2019 5) 2014
[Explanation: It aims to provide a comprehensive insurance cover against the failure of
the crop in order to stabilise the income of the farmers. It covers all food & oilseed crops
and annual commercial/horticultural crops for which past yield data is available. The
prescribed premium is 2% to be paid by farmers for all Kharif crops and 1.5% for all rabi
crops while it is 5% for horticultural crops. Enrolment has been made 100% voluntary for
all farmers from 2020.]

r’s
12. PM Narendra Modi has unveiled the____ meter-tall National Emblem on new
parliament Building.
1) 5.6 2) 7.2 3) 6.8 4) 5.9 5) 6.5
[Explanation: The National Emblem is made of bronze with a total weight of 9500
kilograms and is 6.5 meters in height. A supporting structure of steel weighing around
ha
6500 kilograms has also been constructed to support the Emblem. The concept sketch
and process of casting the National Emblem on the roof of the New Parliament Building
has gone through eight different stages of preparation from clay modelling and computer
graphics to bronze casting and polishing.]
13.

E
Who launched ‘Connecting through Culture’, an anthology on India’s soft power?
1) Subramanyam Jai Shankar 2) Nirmala Sitharaman
d
3) Amit Shah 4) Narendra Modi 5) Raj Nath Singh
[Explanation: The Minister described the book as the “good cop” in diplomacy and said
ee

that it can be used to encourage others to work with India because “it creates a comfort
about India. It takes different aspects of India and to some degree, it brings out the familiarity
with different facets of India.” The anthology in many ways is a very notable contribution
to understanding India, appreciating India, and in many ways valuing India.]
C
14. Who among the following has launched the platform of platforms (POP) under
(eNAM)?
Sr

1)Narendra Modi 2) Amit Shah 3) Narendra Singh Tomar


4) Basavaraj Bommai 5) Thawar Chand Gehlot
[Explanation: With the introduction of POP, farmers will be facilitated to sell the produce
outside their state borders. This will increase farmers’ digital access to multiple markets,
buyers and service providers and bring transparency in business transactions with the
aim of improving price search mechanism and quality commensurate price realisation.]
15. BJP has retained who among the following as its leader in Rajya Sabha for the
C
third time?
1) Mukhtar Abbas Naqvi 2) Om Birla 3) Nirmala Sitharaman
4) Piyush Goyal 5) Narendra Modi
[Explanation: Goyal holds the charge of three ministries — commerce and industry,
food and public distribution and textiles. An all-India second-rank holder chartered
accountant, Goyal has also held the portfolios of railway, power and coal in the past. The
BJP will also have to pick its deputy leader in the Upper House of Parliament as the term
of former Union minister Mukhtar Abbas Naqvi, who held the position, ended recently.]
16. Which of the following country has signed an MOU with India on tele-education?
1) Egypt 2) Madagascar 3) USA 4) Japan 5) Poland
[Explanation: The tele-education offers lectures ranging from bachelor’s degree to higher
studies. It is totally free of cost. However, there are some eligibility criteria for students
who want to apply for it and after completion of study, students will get their degrees and
diplomas. 2) Madagascar Capital: Antananarivo. Currency: Malagasy ariary.]

JULY- 42 Visitus:www.sr eedharscce.com


Sreedhar’s C C E CURRENT AFFAIRS -JULY-2022
17. Which of the following has launched ‘AadhaarFaceRd’ mobile app to perform
Aadhaar Face authentication?
1) RBI 2) NITI Aayog 3) UIDAI 4) IRDAI 5) SIDBI
[Explanation: For authentication, Aadhaar card holders no longer need to physically
visit an enrollment centre for iris and fingerprint scans. The UIDAI has started using face
authentication as a method of confirming an Aadhaar holder’s identity. Once your facial
authentication is successful, it verifies your identity. UIDAI CEO: Saurabh Garg; UIDAI
Founded: 28 January 2009; UIDAI Headquarters: New Delhi.]
18. The Central government has clarified that the sale of the Indian National Flag,
adhering to the Flag Code _________ and its subsequent amendments, is exempt

r’s
from GST.
1) 2002 2) 2000 3) 1998 4) 2004 5) 1997
[Explanation: Sale of the Indian national flag, irrespective of whether machine made or
of polyester, is exempt from the Goods and Services Tax (GST). Hand-woven, hand-spun
national flags made of cotton, silk, wool or Khadi are already exempt from GST. the Revenue
Department clarified that polyester or machine-made tricolour too would be exempt from

19.
ha
the levy following amendments to the ‘Flag Code of India, 2002.]
PM Narendra Modi has inaugurated ____ Km four-lane Bundelkhand Expressway
in UP’s Jalaun.
1) 256 2) 340

E 3) 180 4) 296 5) 324


[Explanation: Which passes through seven districts of Uttar Pradesh and has been
d
constructed at a cost of around Rs 14,850 crore.It extends from NH-35 at Gonda village
near Bharatkoop in Chitrakoot district to near Kudrail village in Etawah district, where it
merges with the Agra-Lucknow Expressway.]
ee

20. Center has approved the extension of the scheme for Rebate of State and Central
Taxes and Levies (RoSCTL) till ________for export of apparel, garments.
1) April 01, 2023 2) January 01, 2024 3) March 31, 2024
4) April 30, 2024 5) March 31,2023
C
[Explanation: RoSCTL is a growth-oriented, forward-looking programme that has helped
increase exports and jobs by establishing a stable and predictable policy environment.
Sr

The programme enhanced cost effectiveness and export competitiveness on the global
market. Additionally, it has aided in the development of startups and entrepreneurs in the
industry and encouraged a significant number of MSMEs to enter the garment export
market.]
21. Which minister has launched the trailer for the new television series “Swaraj:
Bharat KeSwatantrata Sangram Ki Samagra Gatha” in New Delhi?
1) Ministry of Home Affairs 2) Ministry of Education
C
3) Ministry of Law and Justice
4) Ministry of Information and Broadcasting
5) Ministry of Culture
[Explanation: Doordarshan will begin airing this series on August 14th, 2022. The
contribution of the liberation warriors and unsung heroes of the freedom movement will be
highlighted in the 75-episode drama. The All India Radio will also air this programme. The
serial logo was unveiled by Mr. Thakur at New Delhi’s Akashwani Bhawan.]
22. Government has launched __________ mascot for empowering consumers and
generating awareness towards their rights.
1)Jagriti 2) Veera 3) Dhakad 4) Miraitowa 5) La’eeb
[Explanation: Jagriti mascot will be displayed along with tagline “JagoGrahakJago” across
all the media campaigns. Jagriti Mascot and “JagoGrahakJago tagline” are new synonymous
with young aware consumers. Both of these seek to bring sharp focus towards consumer
rights knowledge and movement. Consumer right can be defined as the “right to have
information regarding quality, quantity, potency, price, purity, and standard of goods or
services”.]

JULY- 43 Visitus:www.sr eedharscce.com


Sreedhar’s C C E CURRENT AFFAIRS -JULY-2022
23. Which of the following programme to be extended to all districts ?
1) BetiBachaoBetiPadhao 2) One Stop Centre Scheme
3) Nari Shakti Puraskar
4) Mahila Shakti Kendras (MSK)
5) Working Women Hostel
[Explanation: which focuses on education of girl child and improving sex ratio, will now
be extended across the country, according to guidelines issued by the Women and Child
Development (WCD) Ministry. The programme is operational in 405 districts at present.
Aim for promoting sports among girls, self-defence camps, construction of girls’ toilets,

r’s
making available sanitary napkin vending machines and sanitary pads, especially in
educational institutions, awareness about PC-PNDT Act, etc.]
24. UNICEF has joined hands with Which govt for job opportunities to students?
1) Jammu and Kashmir 2) Delhi 3) Karnataka
4) Tamil Nadu ha 5) Ladakh
[Explanation: Delhi Skill and Entrepreneurship University (DSEU) and UNICEF have
introduced ‘Career Awareness Sessions’ for students. Skill Varsity of Delhi joins hands
with YuWaah (Generation Unlimited India) at UNICEF to enable access to employment
opportunities, assist students in becoming job ready as well as listen to and amplify the

E
voices of young people. UNICEF Founded: 1946; UNICEF Headquarters: New York City,
d
USA; UNICEF Director General: Catherine M. Russell; UNICEF Membership: 192.]
25. According to Ministry of commerce and industry, paper import monitoring system
(PIMS) will come into effect from___________.
ee

1) 1st October 2022 2) 1st August 2022 3) 1st November 2022


st
4) 1 March 2023 5) 1st January 2023
[Explanation: he Paper Import Monitoring System (PIMS) was established by the
Directorate General of Foreign Trade (DGFT) by changing the import policy for major paper
C
products from “free” to “free subject to mandatory registration under PIMS. The PIMS
Sr

must be used when a Domestic Territory Area unit imports a variety of paper products,
including newspaper, handmade paper, coated paper, uncoated paper, Litho and offset
paper, tissue paper, toilet paper, cartons, labels, and more.]
26. Cabinet has approved Taranga Hill-Ambaji-Abu Rd rail line linking two key religious
places in Gujarat and __________.
1) Maharashtra 2) Madhya Pradesh 3) Rajasthan
4) Delhi 5) Haryana
C
[Explanation: Taranga Hill-Ambaji-Abu Road Rail Line will be constructed at an estimated
cost of Rs 2,798.16 crore. Project is expected to be completed by 2026-27. It will help in
strengthening the connectivity and boosting socio-economic progress. This project was
approved in line with long-standing demand by people, for improving access to Ambaji.]
27. Defence Minister Rajnath Singh has set to launch ___________ under ‘Project
17A’.
1) INS Dunagiri 2) INS Vagsheer 3) INS Kalvari
4) INS Khanderi 5) INS Vikrant
[Explanation: Union Defence Minister Rajnath Singh has launched the fourth P17A stealth
frigate ‘Dunagiri’ into the Hooghly River in Kolkata. The Project 17A frigate is built by
Garden Reach Shipbuilders Limited. The P-17A class is a follow-on of the P-17 Shivalik
Class with improved stealth features & advanced weapons. he first two ships of the P17A
Project were launched in 2019 and 2020 at MDL and GRSE respectively. The third ship
(Udaygiri) was launched at MDL on 17 May 22 earlier this year.]

JULY- 44 Visitus:www.sr eedharscce.com


Sreedhar’s C C E CURRENT AFFAIRS -JULY-2022
28. Who among the following has launched ‘Mobile Electric Charging’ App ?
1) Narayan Rane 2) Anurag Thakur 3) Virendra Singh
4) Jitendra singh 5) Ashwini Vaishnaw
[Explanation: MSME Narayan Rane introduced Repos Pay, a platform for Mobile Electric
Charging, and Phy-gital a platform for fintech. On the Repos Pay platform, users can order
mobile electric charging vehicles and use them to charge their cars. A Fintech platform
called Phy-gital will use technology to enable consumers to use credit to fuel purchases
(Buy Now Pay Later). Fuel-on-credit options for large users will be facilitated through the
energy fintech platform.]

r’s
29. Government has formulated__________ scheme for cleaning of sewers, septic tank.
1) NAMASTE 2) Jal Jeevan Samvad 3) Jal Shakti Abhiyan
4) AMRUT 5) National Water Mission
[Explanation: The scheme is a joint venture of Department of Drinking Water and
Sanitation, Ministry of Social Justice and Empowerment and the Ministry of Housing and
ha
Urban Affairs.It aims to achieve outcomes like zero fatalities in sanitation work in India, no
sanitation workers come in direct contact with human faecal matter and all Sewer and
Septic tank sanitation workers have access to alternative livelihoods.]
30. 220 children have been admitted in KendriyaVidyalayas (KVs) under__________
scheme.
1) Ayushman Bharat Yojana

E
d
2) PM Cares for Children Scheme
3) Pradhan Mantri Jan DhanYojan
ee

4) Integrated Child Development Service


5) Mission Indradhanush
[Explanation: The PM Cares for Children Scheme provides comprehensive care and
protection of children who have lost their parent(s) to the Covid pandemic. Minister of
C
Education Dharmendra Pradhan stated under this scheme, such children have been
provided support in a sustained manner to empower them through education and equip
Sr

them for self-sufficient existence with financial support on reaching 23 years of age.]
31. Government has informed around 60 lakh beneficiaries has benefited
under___________.
1) Aatmanirbhar Bharat Rojgar Yojana
2) Green Skill Development Programme
3) National Apprenticeship Promotion Scheme
4) Pradhan Mantri Kaushal Vikas Yojana
C
5) National Food for Work Programme
[Explanation: the Minister of State for Labour and Employment Rameshwar Teli Stated
Aatmanirbhar Bharat Yojana has been launched with effect from 1st October 2020 to
incentivize employers for creation of new employment and restoration of loss of employment
during Covid-19 pandemic. the government is providing fiscal stimulus of more than twenty-
seven lakh crore rupees. ]
32. The government has informed around 28 crore unorganised workers have been
registered on ___________ Portal .
1)SAGE 2) ‘MyNEP2020’3) e-Nirman 4) e-Shram5) ‘MyGov-Meri Sarkar’
[Explanation: In a written reply in Lok Sabha, Minister of State for Labour and
Employment RameswarTeli stated the government had launched e-Shram portal on 26th
August 2021 with an objective to create a national Database of unorganised workers. It
also facilitates delivery of Social Security Schemes and Welfare schemes to the unorganised
workers.]

JULY- 45 Visitus:www.sr eedharscce.com


Sreedhar’s C C E CURRENT AFFAIRS -JULY-2022
33. DAY-NRLM of the Ministry of Rural Development has organized
“UdyamitaPakhwara” across the Country from _________in 2022.
1) 7th to 14 July 2) 1st to 7th July 3) 1st to 15th June
st th
4) 21 to 28 June 5) 1st to 15th July
[Explanation: The objective of this UdyamitaPakhwara was to highlight the importance of
enterprise growth and entrepreneurship development under non-farm livelihoods in rural
areas across the country. A series of activities under Start-up Village Entrepreneurship
Programme (SVEP), One Stop Facility (OSF), Clusters, Incubators, SHG products marketing,
PMFME scheme were organised in this duration across the country.]
34. Which ministry has launched ‘NashaMukt Bharat to address the problem of Drug

r’s
Abuse among youth of India ?
1) The Ministry of Social Justice and Empowerment
2) Ministry of Education
3) Ministry of Rural Development
4) Ministry of Panchayati Raj
ha
5) Ministry of Law and Justice
[Explanation: To address the issue of drug abuse among Indian youth, the Ministry of
Social Justice and Empowerment began implementing NashaMukt Bharat Abhiyaan in the

E
272 most susceptible districts in August 2020. Ministry of Social Justice & Empowerment,
Government of India: Dr. Virendra Kumar. Minister of State for Social Justice and
d
Empowerment: A. Narayanaswamy.]
35. Which port has become the first 100 per cent Landlord Major Port of India having
all berths being operated on PPP model ?
ee

1) Adani Mundra Port


2) Jawaharlal Nehru Port
3) Visakhapatnam port
4) Kandla port
C
5) Kolkata Port Trust
Sr

[Explanation: The PPP model of investment in Indian ports has made remarkable progress
in the last 25 years, beginning from the Jawaharlal Nehru Port, resulting in capacity addition
and productivity improvement. JNP is one of the leading container ports in the country and
is ranked 26th among the top 100 global ports (as per Lloyds List Top 100 Ports 2021
Report).]
36. Who among the following has unveiled ‘varuna’, India’s first human passenger
drone?
C
1) Rajnath Singh 2) JyotiradityaScindia 3) Narendra Modi
4) Amit Shah 5) G. Kishan Reddy
[Explanation: The makers of this indigenous pilot-less ‘Varuna’ drone startup Sagar
Defence Engineering demonstrated this in his presence. This ‘Varuna’ drone has been
designed and developed by a startup and has the capacity to carry one person inside.
Varuna, India’s first drone that can carry a human payload, has a range of 25 km. The
drone can carry a 130 kg payload and has 25-33 minutes of flight time.]
37. Centre has likely to Extend the PMAY-Urban Mission Scheme till _______.
1) March 2024 2) June 2023 3) March 2023
4) April 2025 5) June 2024
[Explanation: Ministry of Housing and Urban Affairs is implementing Pradhan Mantri
Awas Yojana-Urban (PMAY-U) – ‘Housing for All’ Mission, since 25.06.2015 for giving
Central assistance to States/ Union Territories (UTs) to provide all-weather pucca houses
to all eligible urban beneficiaries. A proposal seeking an extension of the Mission up to
March 2024.]

JULY- 46 Visitus:www.sr eedharscce.com


Sreedhar’s C C E CURRENT AFFAIRS -JULY-2022
38. Insurance companies has made ?40,000 crore from__________ scheme in 5 years
1) Pradhan Mantri Suraksha Bima Yojana
2) Pradhan Mantri FasalBima Yojana
3) Central Government Health Scheme
4) Employees State Insurance Scheme
5) Pradhan Mantri Jeevan Jyoti Bima Yojana
[Explanation: With the Pradhan Mantri FasalBima Yojana (PMFBY) entering the seventh
year of implementation in the upcoming Kharif season, the government has announced a
new doorstep crop insurance distribution policy known as ‘Meri Policy Mere Hath’.

r’s
Overview: The ‘Meri Policy Mere Hath’ door-step campaign has been launched to ensure
that all farmers are well informed.]
39. Which among the following district is the India’s First Certified ‘Har Ghar Jal’
District?
1) Burhanpur 2) Balaghat. 3) Mewat 4) Sikar 5) BhadradriKothgudem
[Explanation: Burhanpur is the only district in the country where all people from each of
ha
254 villages have access to safe drinking water through taps. Only 36.54 percent of over
one lakh households in Burhanpur had potable drinking water through tap connections at
the time of the launch of Jal Jeevan Mission on 15th August 2019.]
40.

E
Which of the following ministry has launched GOAL Programme (GOAL 2.0)
1)Ministry of Environment, Forests and Climate Change
d
2) Ministry of Micro, Small and Medium Enterprises
3) Ministry of Science and Technology
4) Ministry of Social Justice and Empowerment
ee

5) Ministry of Tribal Affairs


[Explanation: Recently, second phase of the GOAL Programme (GOAL 2.0) was launched
with an aim to digitally upskill tribal youth by promoting entrepreneurship and opening up
C
opportunities for them using digital technology. GOAL (Going Online as Leaders) is a joint
initiative of Ministry of Tribal Affairs and Meta (formerly Facebook). They will be digitally
Sr

trained with regard to market demand, packaging, branding and marketing of their
products.]
41. Who among the following Union minister has inaugurated the Vaccination Centre
for Children and also Launched ‘Bal Raksha’ Mobile App at All Institute of Ayurveda?
1) Amit Shah 2) Smriti Zubin Irani
3) Mansukh L. Mandaviya
4) SarbanandaSonowal 5) Anurag Singh Thakur
C
[Explanation: the Bal Raksha mobile app, which is aimed at creating awareness of
paediatric preventive healthcare through Ayurveda intervention amongst parents. The app
will also collect feedback from parents regarding the impact of the kit on their children’s
health and immunity. The All India Institute of Ayurveda is under Ministry of Ayush, aims
at bringing synergy between the traditional wisdom of Ayurveda and modern tools and
technology.]
42. Who among the following has launched the ‘Digital Jyot’ campaign to give special
tribute to heroes of freedom struggle?
1)Raj Nath Singh 2) Amit Shah 3) Piyush Goyal
4) Narendra Modi 5) Anurag Singh Thakur
[Explanation: the Digital Jyot symbolizes the glow of freedom that illuminates our lives.
It is surrounded by an illuminated metallic floral sculpture inspired by the courage and
determination of our freedom fighters and martyrs. The design finds its inspiration from
nature and flowers linked to the ideas of hope, strength and positivity.]

JULY- 47 Visitus:www.sr eedharscce.com


Sreedhar’s C C E CURRENT AFFAIRS -JULY-2022
43. Who has implemented Suryamitra Skill Development Programme ?
1) National Thermal Power Corporation Pvt Ltd
2) The Ministry of New and Renewable Energy
3) Power Grid Corporation of India
4) Adani Power Ltd
5) Tata Power Solar Systems
[Explanation: Ministry of New and Renewable Energy is implementing Suryamitra Skill
Development Programme through National Institute of Solar Energy, Gurugram since
Financial Year 2015-16 to train youth of age above 18 years as solar PV technicians for
installation, operation and maintenance of solar power projects.]

r’s
44. Recently, How many wetlands have been designated in India?
1) 9 2) 6 3) 5 4) 8 5) 7
[Explanation: the total number of Ramsar sites in the country have been increased from
49 to 54 Ramsar sites. The five newly designated wetlands are: Karikili Bird Sanctuary,
(Tamil Nadu); Pallikaranai Marsh Reserve Forest(Tamil Nadu); Pichavaram Mangrove, (Tamil

45.
ha
Nadu); Pala wetland(Mizoram); Sakhya Sagar(Madhya Pradesh).]
Which Union Ministry has launched the “Bal Raksha” mobile app?
1) AYUSH Ministry 2) Educational Ministry
3) Jal Shakti Ministry
4) Railway Ministry

E 5) Home Ministry
d
[Explanation: Union Minister of AYUSH SarbanandaSonowal inaugurated a Vaccination
Centre for children at the All India Institute of Ayurveda (AIIA). The Minister also launched
the ‘Bal Raksha’ mobile app, to create awareness among parents on paediatric preventive
ee

healthcare through Ayurveda intervention. The app will also collect feedback from parents
regarding the impact of the kit on their children’s health and immunity.]
46. Which of the following ministry has celebrated “Ujjwal Bharat Ujjwal Bhavishya –
Power @2047” as a part of Azadi ka Amrit Mahotsav?
C
1) Ministry of Heavy Industries 2) Ministry of Power
Sr

3) Ministry of Food Processing Industries


4) Ministry of Home Affairs
5) Ministry of Petroleum and Natural Gas
[Explanation: Shri. Nilesh Cabral,Minister for PWD, appealed to people to adopt sources
of renewable energy to meet their power requirements as there is a high rise in demand
for power supply in the State. The Ministry of Power Union Cabinet Minister is Raj Kumar
Singh. Constituency is Arrah.]
47. Tata Group has delivered indigenously developed Quick Reaction Fighting Vehicle
C
(QRFV) to the which of the following?
1) Indian Navy 2) Indian Air Force 3) Indian Army
4) Bharat Electronics Ltd 5) Hindustan Aeronautics Ltd
[Explanation: The contract to provide QRFV-Med vehicles to the Indian Army was
successfully completed. These armoured vehicles will increase the nation’s protector’s
ability to fight in all weather and terrain situations and will offer protection while moving.
Chief of Army Staff: General Manoj Pande, Defence Minister of India: Rajnath Singh.]
48. Grand statue of Chandrashekhar Azad to be installed in_________.
1) Bhopal 2) Prayagraj 3) Lucknow 4) Kanpur 5) Patna
[Explanation: The statue site will be created as a source of inspiration for young people,
with soil from Azad’s hometown of Bhabra being used at its base. On the 116th anniversary
of Amar Shaheed Chandrashekhar Azad’s birth, Chief Minister Shivraj Singh Chouhan
spoke at the opening ceremony of the first state-level Youth Mahapanchayat, which was
held in Bhopal.]

JULY- 48 Visitus:www.sr eedharscce.com


Sreedhar’s C C E CURRENT AFFAIRS -JULY-2022
49. Which ministry has launched Nyaya Mitra Scheme?
1) Ministry of Tribal Affairs 2) Ministry of Law and Justice
3) Ministry of Jal Shakti 4) Ministry of Power 5) Ministry of Railways
[Explanation: Nyaya Mitra provides assistance to the judiciary in reduction of decade
old pending court case, liaising with the different departments to fast disposal of cases,
referring cases to Lok Adalat. etc. Since introduction of Nyaya Mitra programme in 2017,
a total of 38 Nyaya Mitras were engaged in the States of Assam, Bihar, Karnataka,
Maharashtra, Odisha, Rajasthan, Uttar Pradesh and West Bengal.]
50. PM Modi has unveiled India’s first International bullion exchange at _______.
1) Gandhinagar 2) Surat 3) Ahmedabad 4) Jamnagar 5) Kutch

r’s
[Explanation: This exchange will facilitate efficient price discovery with the assurance of
responsible sourcing and quality, apart from giving impetus to the financialisation of gold
in India. The prime minister also laid the foundation stone for the headquarters building of
the International Financial Services Centres Authority. Apart from that, Modi launcedh NSE
IFSC-SGX Connect platform.]
51.
ha
Who has launched “Power Ministry’s flagship Revamped Distribution Sector”
Scheme?
1) Amit Shah 2) Narendra Modi 3) Om Birla
4) RK Singh 5) Venkaiah Naidu

E
[Explanation: The scheme aims at improving the operational efficiencies and financial
d
sustainability of the Distribution Companies (DISCOMs) and power departments. It has an
outlay of over 3 lakh crore rupees over a period of five years from Financial Year 2021-22
to 2025-26.]
ee

52. Who has addressed the inaugural session of First All India District Legal Services
Authorities Meet in Delhi?
1) Smriti Irani 2) Arjun Munda 3) Narendra Modi
4) SarbanandaSonowal 5) Giriraj Singh
C
[Explanation: The first-ever national-level meet of District Legal Services Authorities
Sr

(DLSAs) is being organized at Vigyan Bhawan by National Legal Services Authority. The
meeting will deliberate on the creation of an integrated procedure to bring homogeneity
and synchronization across DLSAs. There are a total of 676 District Legal Services
Authorities in the country. They are headed by the District Judge who acts as Chairman of
the authority.]
53. Which Ministry is implementing a centrally sponsored scheme ‘Mission Vatsalya’?
1) Ministry of Minority Affairs
2) Ministry of Home Affairs 3) Ministry of Tribal Affairs
C
4) Ministry of Women and Child Development
5) Ministry of Women and Child Development
[Explanation: Under it, a monthly grant of Rs. 4000/- per child is provided for family
based non-institutional care including Sponsorship (kinship) or Foster Care or After Care.The
Mission Vatsalya in partnership with States and Districts provides support to a 24x7
helpline service for children as defined under the JJ Act, 2015.]
54. Who has launched floating solar power plant at NTPC-Kayamkulam ?
1) Raj Kumar Singh 2) A. M. Ariff 3) Narendra Modi
4) G. Sajini 5) S.K. Ram
[Explanation: The Kayamkulam plant is the second-largest floating solar power project
of the NTPC after the 100-MW plant at Ramagundam in Telangana. The floating solar system
was built on a 450-acre lake area adjacent to the NTPC’s Rajiv Gandhi Combined Cycle
Power Project (RGCCPP). Implemented at a cost of ?465 crores, the project consists of
around three lakh Made in India solar PV panels floating on water.]

JULY- 49 Visitus:www.sr eedharscce.com


Sreedhar’s C C E CURRENT AFFAIRS -JULY-2022

9.PERSONS IN NEWS
1. Shanan Dhaka, India’s First Woman NDA (National Defence Academy) Topper.
She belongs to which state?
1) Kerala 2) Assam 3) Punjab 4) Bihar 5) Haryana
[Explanation: A 19-year-old girl, Shanan Dhaka, from Rohtak district in Haryana, was
in the headlines as she secured the first rank among female candidates in the entrance
examination for the National Defence Academy (NDA).]

r’s
2. Who among the following has become one of the youngest to learn 17 computer
languages?
1) Rahul Narwekar 2) Arnav Sivram 3) LakshikaDagar
4) Maitri Patel 5) Arya Rajendran
[Explanation: Arnav Sivram from Coimbatore, Tamil Nadu, has become one of the
ha
youngest people to have learnt 17 computer languages at the age of 13.Sivram revealed
that he started learning computers when he was in class 4th.]
3. Who has appointed as the ambassador of Beardo for fragrance segment?
1) Ranbir Kapoor 2) Sunny Kaushal 3) Salman Khan
4) Vicky Kaushal
5) Hrithik Roshan

E
d
[Explanation: Beardo signs Vicky Kaushal as ambassador for fragrance segment. In
2021, the male grooming company appointed Roshan as its brand ambassador. Celebrating
ee

OG masculinity in all its glory, Beardo believes it is possible to own one’s raw and rugged
masculine charm with utmost sophistication – a fact Vicky Kaushal proves.]
4. Who has become AATC’s Youngest Analog Astronaut?
1) Chris Hadfield 2) Musgrave 3) JahnaviDangeti
C
4) Yuri Gagarin 5) John Glenn
Sr

[Explanation: 19-year-old JahnaviDangeti has scripted history by becoming the youngest


to complete the Analog Astronaut programme from the Analog Astronaut Training Centre
(AATC) at Krakow in South Poland. MsJahnavi had completed NASA’s International Air
and Space Programme (IASP) at the Kennedy Space Centra in the U.S. NASA administrator:
Bill Nelson; Headquarters of NASA: Washington D.C., United States; NASA Founded: 1
October 1958.]
5. Who has become the first captain in the history of T20 cricket to record 13 successive
C
T20I wins?
1) Babar Azam 2) Rishabh Pant 3) Hardik Pandya
4) Farkhar Zaman 5) Rohit Sharma
[Explanation: Indian skipper Rohit Sharma has become the first captain in the history of
cricket to win 13 successive T20 Internationals. He achieved the feat after India beat England
by 50 runs in the first T20I to take 1-0 lead in the three-match T20I series at the Rose
Bowl, Southampton on July 7, 2022. This was Rohit Sharma’s first match after recovering
from COVID-19.]
6. Recently Akasa Air has got Air Operator Certificate from DGCA, who is the promoter
of “Akasa Air”?
1) Rakesh Juhunjhunwala 2) Warren Buffett
3) Gautham Adani 4) Harshad Mehta 5) RadhakrishanDamani
[Explanation: Billionaire investor Rakesh Juhunjhunwala-owned Akasa Air has been
cleared for take-off. The no-frills airline received its air operator certificate (AOC) from the
Directorate General of Civil Aviation (DGCA).]

JULY- 50 Visitus:www.sr eedharscce.com


Sreedhar’s C C E CURRENT AFFAIRS -JULY-2022

7. Who has resigned as FIH, IOA president?


1) Narinder Batra 2) Ashish Kumar Chauhan 3) Narendra Batra
4) V K Singh 5) Rajkiran Rai
[Explanation: Narinder Dhruv Batra resigned from his positions as the president of the
Indian Olympic Association (IOA) and the International Hockey Federation (FIH), as well
as the member of the International Olympic Committee (IOC).]
8. Jagdeep Dhankar has submitted resignation to President Kovind, He was the
governor of ___________.
1) West Bengal 2) Jharkhand 3) Sikkim

r’s
4) Meghalaya 5) Punjab
[Explanation: President Ram Nath Kovind has accepted the resignation of West Bengal
Governor Jagdeep Dhankhar, a statement from RashtrapatiBhavan. Manipur Governor La.
Ganesan has been given additional charge of West Bengal governor after Jagdeep Dhankar
resigned.]
9.
ha
England all-rounder Ben Stokes has announced ODI retirement, He is
from__________.
1) South Africa 2) New Zealand 3) Ireland
4) Australia

E 5) Scotland
[Explanation: Star England all-rounder and 2019 World Cup final hero Ben Stokes has
d
announced his retirement from ODI cricket. The 31-year-old Stokes will play his last ODI
against South Africa at his home ground in Durham.]
ee

10. Who among the following Indian has become the 4th richest person in the Forbes
world’s richest list?
1) Bill Gates 2) Bernard Arnault 3) Gautam Adani
4) Mukesh Ambani 5) Jeff Bezos
C
[Explanation: Gautam Adani, India’s richest person, now holds the fourth spot on the
Sr

Forbes world’s richest list. First place in Elon Musk(USA), 2nd place in Bernard Arnault
(France),3rd place in Jeff Bezos(USA) and 5th place in Bill Gates(USA).]
11. Who has been resigned as Italy’s prime minister?
1) Boris Johnson 2) Mario Draghi 3) AskarMamin
4) Mark Rutte 5) Yoshihide Suga
[Explanation: Mario Draghi resigned as Italy’s prime minister. Italy president: Sergio
Mattarella Italy capital: Rome and currency: Euro]
C
12. Karuna Jain has announced retirement recently. She belongs to which sports?
1) Snooker 2) Boxer 3) Cricketer 4) Hockey 5) wrestling
[Explanation: Former India regular feature and wicket-keeper-batter, Karuna Jain, 36,
announced her retirement from all forms of international cricket. Karuna Jain was an integral
part of the Indian team having made an instant impact in her debut appearance on the
international stage in 2004.]
13. Who has become first Bangladeshi to scale world’s second highest peak K2?
1) WasifaNazreen 2) SaminaBaig 3) AfsanehHesamifard
4) Ashraf Aman 5) Sajid Ali Sadpara
[Explanation: WasifaNazreen becomes the first Bangladeshi to scale world’s second
highest peak K2. Mountaineer WasifaNazreen became the first Bangladeshi to scale the
second highest mountain peak K2 in Pakistan. She climbed the 8611 metre or 28,251 feet
high K2 mountain peak and returned.]

JULY- 51 Visitus:www.sr eedharscce.com


Sreedhar’s C C E CURRENT AFFAIRS -JULY-2022
14. Who hasbecomethe first Indian to qualify for the triple jump final at the World
Athletics Championships 2022?
1) Eldhose Paul 2) Neeraj Chopra 3) Rohit Yadav
4) Annu Rani 5) MuraliSreeshankar
[Explanation: Among the 22 athletes - 18 men and four women - representing India at
the world athletics meet in Eugene, Eldhose Paul also stood out by becoming the first
Indian to make a triple jump final at the world championships.]
15. Who has resigned as the CEO of Volkswagen company?
1) Ashneer Grover 2) Daniela Cavallo 3) Herbert Diess
4) Martin Winterkorn 5) Ralf Brandstatter

r’s
[Explanation: Volkswagen CEO Herbert Diess has resigned.Diess, 63, informed the
board of supervisors of his decision. VW announced that Diess would be replaced by 54-
year-old Porsche CEO Oliver Blume, who will serve as CEO of both Volkswagen and
Porsche.]
16. Lendl Simmons has announced retirement from international cricket. He belongs
to which country?
1) Australia
ha
2) New Zealand 3) England
4) Pakistan 5) West Indies
[Explanation: West Indies opening batter Lendl Simmons announced his retirement from

E
international cricket. The Windies cricketer, who last played an international game in October
d
2021 against South Africa.]
17. Former captain DeneshRamdin announces retirement from international cricket.
He belongs to which Country?
ee

1) China 2) France 3) West Indies 4) Germany 5) Japan


[Explanation: Former West Indies captain DeneshRamdin has announced his retirement
from international cricket. Ramdin has played 74 Tests, 139 ODIs and 71 T20Is, and
scored four Test hundreds and 2 ODI hundreds.]
C
18. Recently Gotabaya Rajapaksa has stepped down, he was the President of which
Sr

country?
1) Iran 2) Ukraine 3) Turkey 4) Germany 5) Sri Lanka
[Explanation: Rajapaksa is a lawyer by profession and was first elected to the Parliament
of Sri Lanka in 1970. He served as the leader of the Sri Lanka Freedom Party from 2005
to 2015. Rajapaksa was sworn in for his first six-year term as president on 19 November
2005. He was subsequently re-elected in 2010 for a second term. Present President of
Sri Lanka:RanilWickremesinghe and present PM: Dinesh Gunawardena.]
19. Moab Khotsong, where 1.2-billion-year-old groundwater has been discovered, is
C
located in which country?
1) Brazil 2) Kenya 3) Jamaica 4) South Africa 5) Canada
[Explanation: Geologists have discovered 1.2-billion-year-old groundwater about 3?km
below surface in Moab Khotsong. It is a gold- and uranium-producing mine in South Africa.
This ancient groundwater is enriched in the highest concentrations of radiogenic products.
Uranium and other radioactive elements naturally occur in the surrounding host rock. They
provide information on co-habitating microorganisms previously discovered in the Earth’s
deep subsurface.]
20. Sebastian Vettel has announced retirement, He is from _________.
1) Russia 2) USA 3) Germany 4) Poland 5) Denmark
[Explanation: Sebastian Vettel, 35, made the announcement ahead of this weekend’s
Hungarian Grand Prix, the final race before Formula 1’s summer recess. At 23, Vettel
became Formula 1’s youngest world champion in 2010 and followed it up with three in a
row, including dominant campaigns in 2011 and 2013.]

JULY- 52 Visitus:www.sr eedharscce.com


Sreedhar’s C C E CURRENT AFFAIRS -JULY-2022

10. PLACES IN NEWS


1. Which expressway is being developed as the first elevated urban expressway in
India?
1) Delhi-Meerut Expressway 2) Purvanchal Expressway
3) Lucknow Agra Expressway
4) Dwarka Expressway 5) Bundelkhand Expressway

r’s
[Explanation: The 29-km expressway connecting Dwarka in Delhi to Gurugram in
Haryana is being developed at a total cost of ?9,000 crore and about 19-km of the
expressway stretch falls in Haryana while the remaining 10-km is in Delhi.]
2. Which has become the first Railway station in India equipped with Augmented
ha
reality screens?
1) Visakhapatnam Railway Station 2) Chhatrapati Shivaji Maharaj Terminus
3) New Delhi Railway Station
4) Sealdah Railway Station

E 5) Nagpur Railway Station


d
[Explanation: The futuristic technology will allow passengers at the station to get real-
time digitised time table with a dedicated scrolling screen and railway information for an
engaging experience. Augmented reality magic mirrors create a virtual world around the
ee

person standing in front of the screen.]


3. Which of the following state/UT has celebrated RathYatra Festival?
C
1) Jammu and Kashmir 2) Gujarat 3) Odisha
4) Puducherry 5) Maharashtra
Sr

[Explanation: This festival is dedicated to Lord Jagannath, his brother Balbhadra and
his sister Subhadra. The journey of the deities begins from the iconic Puri Temple and
reaches the Gundicha Shrine after twelve the Lord returns to his adobe. The idols of the
three deities are brought out by three priests out of the sanctum sanctorum or GarbhaGriha
and carried to Gundicha Shrine in three giant wooden chariots.]
C
4. Which state has celebrated Baahudaa Yatra festival?
1) Gujarat 2) Meghalaya 3) Arunachal Pradesh 4) Odisha 5) Maharashtra
[Explanation: The Bahuda Yatra, the return journey of the deities to the Puri Jagannath
temple after the annual Rath Yatra, was celebrated in absence of devotees in the pilgrimage
town of Puri.]
5. Agnikul Cosmos has opened India’s first rocket engine factory in __________.
1) Odisha 2) Chennai 3) Bangalore 4) Telangana 5) New Delhi
[Explanation: The facility will use additive manufacturing technology to build 3D printed
rocket engines and will be used to produce engines for its own in-house rockets. It was
unveiled by Tata Sons chairman N Chandrasekaran and ISRO chairman S Somanath in
the presence of PawanGoenka, the chairman of IN-SPACe (Indian National Space Promotion
and Authorization Centre.]

JULY- 53 Visitus:www.sr eedharscce.com


Sreedhar’s C C E CURRENT AFFAIRS -JULY-2022
6. Cabinet has approved Taranga Hill-Ambaji-Abu Rd rail line linking two key religious
places in Gujarat and __________.
1) Maharashtra 2) Madhya Pradesh 3) Rajasthan 4) Delhi 5) Haryana
[Explanation: Hill-Ambaji- Abu Road new rail line to be constructed by the Ministry of
Railways at an estimated cost of Rs 2798.16 crore. The total length of the new rail line will
be 116.65 km and is expected to be completed in 2026-27. The project would generate
direct employment during construction for about 40 lakh man-days.]
7. Which city will become the Cultural and Tourism Capital of the SCO for 2022-23?
1) Varanasi 2) Surat 3) Chennai 4) Gurugram 5) Mumbai

r’s
[Explanation: Under this initiative, each year a city of the cultural heritage of a member
country that will take over the rotating Presidency of the organisation will get the title to
highlight its prominence. India hosted the Head of the Government of the SCO meet in
2020]
8. Which state has celebrated Kharchi festival?
ha
1) Maharashtra 2) Madhya Pradesh 3) Rajasthan
4) Tripura 5) Haryana
[Explanation: Kharchi festival, offers prayer to 14 gods and goddess, began with
thousands of devotees converging at Khayerpur on the eastern outskirts of the Tripura.

E
Kharchi Puja is predominantly a tribal festival but it owes its origins to Hindu religion.
Devotees and sadhus from across India and neighboring Bangladesh have also participated
d
in the festival.]
9. The Centre has reported to be considering granting “tribal” status to the Trans-
Giri region of ___________ Sirmaur district.
ee

1) Assam 2) Arunachal Pradesh 3) Himachal Pradesh


4) Nagaland 5) Mizoram
[Explanation: The Hattis are a close-knit community who take their name from their
C
traditional occupation of selling home-grown crops, vegetables, meat, and wool at small-
town markets known as ‘haats’. Hatti men traditionally don a distinctive white headgear
Sr

on ceremonial occasions.]
10. Athletics Federation of India has shifted the 61st National Open Athletics
Championships from Jamshedpur to ________.
1) Jaipur 2) Hyderabad 3) Bengaluru 4) Chennai 5) Kolkata
[Explanation: The Karnataka Athletics Association will conduct the National Open on the
same dates at the Kanteerva Stadium in Bengaluru, says an AFI circular to its State
associations.CM: Basavaraj Bommai; Governor: Thawar Chand Gehlot.]
C
11. Indian embassy to organize the 2nd edition of North-East India Festival in______.
1) Bamako 2) Baku 3) Brussels 4) Bangkok 5) Bucharest
[Explanation: The Indian Embassy is organizing the event in association with Trend
MMS of India where the main focus will be on trade, investment and tourism promotion
and exchanges in the field of culture, education and people-to-people connect, according
to the press statement released by the Indian Embassy in Bangkok.]
12. Southern Railway’s 2nd Bharat Gaurav train was flagged off between Madurai
and________.
1) Coimbatore 2) Varanasi 3) Kolkata 4) Prayagraj 5) Vijayawada
[Explanation: Indian Railways launched Bharat Gaurav trains that will be operated by
private players and run on theme-based circuits. Through this policy, which offers operators
the “Right of Use” of its rakes and infrastructure. The train with a capacity of 560 passengers
had got an overwhelming response with 85% booking or 470 passengers travelling in four
3-tier AC coaches and six sleeper class coaches.]

JULY- 54 Visitus:www.sr eedharscce.com


Sreedhar’s C C E CURRENT AFFAIRS -JULY-2022

13. In which city will the grand statue of Chandrasekhar Azad be installed?
1) Indoor 2) Agra 3) Varanasi 4) Bhopal 5) Gwalior
[Explanation: A grand statue of Amar Shaheed Chandrashekhar Azad will be installed
in Bhopal. At the base of the statue, soil brought from Bhabra, the birthplace of Azad, will
be used and the statue site will be developed as a source of inspiration for the youth.]
14. India’s first brain health clinic has launched in __________.
1) New Delhi 2) Gurugram 3) Gandhi Nagar 4) Hyderabad 5) Bengaluru

r’s
[Explanation: On World Brain Day, India’s first brain health clinic inaugurated under
the Karnataka Brain Health Initiative (Ka-BHI) at Jayanagar General Hospital. It was a joint
endeavour of Nimhans and the Department of Health and Family Welfare, to increase
awareness and also prevent and improve treatment for neurological diseases.]
15. In which city, union hospital has launched world’s first fully automatic active
ha
robotic system for knee surgeries?
1) Hyderabad 2) Mumbai 3) Kolkata 4) Chennai 5) Vijayawada

E
[Explanation: The CUVIS robot which is claimed to be the world’s first fully automatic
active robotic system which will perform knee surgeries. This helps the patients recover
d
within no time and most of them can walk home in a day or two.]
16. Sports Authority unveils “Create for India” campaign to cheer for Team India
ee

Commonwealth Games in __________


1) Brasilia 2) Ottawa 3) Hong Kong 4) Thailand 5) Birmingham
[Explanation: A 215-member Indian athlete contingent is set to participate across 16
C
disciplines in this sports event. The motto of the CWG 2022 is “Games for Everyone.”]
Sr

17. In which district Ministry of Power in association with Government of Meghalaya


have organized a Bijli Mahotsav?
1) East Khasi Hills 2) East Jaintia Hills 3) RiBhoi
4) West Garo Hills 5) South Garo Hills
[Explanation: The Bijli Mahotsav highlighted some of the key areas of progress in the
C
country’s power sector - Increase in generation capacity from 2,48,554 MW in 2014 to
4,00,000 MW, which is 1,85,000 MW more than the demand; addition of 1,63,000ckm of
transmission lines connecting the entire country into one grid running on one frequency;
from Ladakh to Kanyakumari and from Kutch to Myanmar border it has emerged as the
largest integrated grid in the world.]
18. Altigreen manufacturing company has partnered with ISI Kolkata for improving
energy efficiency of electric vehicles. Where is the headquarters of Altigreen?
1) Hyderabad 2) Kolkata 3) Bengaluru 4) Mumbai 5) Chennai
[Explanation: The partnership will allow for the analysis of vast amounts of cloud data
generated from the Altigreen EV’s drivetrain stack using state-of-the-art machine learning
algorithms. This will enable the company to continuously improve its energy efficiency in
various scenarios and under different environmental conditions.]

JULY- 55 Visitus:www.sr eedharscce.com


Sreedhar’s C C E CURRENT AFFAIRS -JULY-2022
19. Which among the following district is the India’s First Certified ‘Har Ghar Jal’
District?
1) Burhanpur 2) Balaghat 3) Mewat 4) Sikar 5) BhadradriKothgudem
[Explanation: Burhanpur district of Madhya Pradesh, popularly known as the ‘Darwaza
of Dakhin’, became the first Certified ‘Har Ghar Jal’ district in the Country. Only district in
the country, people from each of 254 villages in Burhanpur have declared their villages as
‘Har Ghar Jal’ through a resolution passed by Gram Sabhas.]
20. 44th FIDE Chess Olympiad is designated to be held in ________.
1) Bengaluru 2) Chennai 3) Hyderabad 4) Mumbai 5) Puducherry

r’s
[Explanation: The 44th Olympiad, which was supposed to take place in Moscow and
Khanty-Mansiysk, was shifted out of Russia following FIDE’s reaction to the war in
Ukraine. Chennai has previously hosted the 2013 World Championship, where Magnus
Carlsen was first crowned World Champion, defeating Anand.]
21. PM Modi has addressed the inaugural session of first All India District Legal

1) Surat
ha
Services Authorities Meet in __________.
2) Delhi 3) Mumbai 4) Kolkata 5) Chennai
[Explanation: There are a total 676 District Legal Services Authorities (DLSAs) in the

E
country which are headed by the District Judges who acts as Chairman of the authority.
The DLSAs also contribute towards reducing the burden on courts by regulating LokAdalats
d
conducted by NALSA.]
22. PM Modi has unveiled India’s first International bullion exchange at _______.
ee

1) Gandhinagar 2) Surat 3) Ahmedabad 4) Jamnagar 5) Kutch


[Explanation: IIBX with its technology-driven solutions, will facilitate transition of Indian
bullion market towards a more organised structure by granting qualified jewellers a direct
C
access to import gold directly through the exchange mechanism.]
23. Tadoba Tiger Reserve has played a host for National Global Tiger Day Celebrations
Sr

2022. This Tiger reserve was located in ____________.


1) Madhya Pradesh 2) Tamil Nadu 3) Maharashtra
4) Karnataka 5) Chhattisgarh
[Explanation: It is a wildlife sanctuary in Chandrapur district of Maharashtra state in
India. It is Maharashtra’s oldest and largest national park. Created in 1955, the reserve
includes the Tadoba National Park and the Andhari Wildlife Sanctuary.]
C
24. 80th Annual Convocation of the Sugar Technologist’s Association of India was
held in which state?
1) Manipur 2) Maharashtra 3) Uttar Pradesh
4) Goa 5) Tripura
[Explanation: The delegates will have an opportunity to meet and interact with nearly 70
service providers ranging from turnkey sugar plant manufacturers to small and medium
enterprises who would be displaying their latest products, processes and services.]
25. Which state has become the first state to launch semiconductor policy 2022-27?
1) Karnataka 2) Telangana 3) Rajasthan 4) Gujarat 5) Maharashtra
[Explanation: The main objective of the policy is to make Gujarat a pioneer in developing
the Electronics System Design & Manufacturing (ESDM) ecosystem. The policy will facilitate
effective and timely execution of the eligible projects for setting up a robust semiconductor
and display fabrication ecosystem.]

JULY- 56 Visitus:www.sr eedharscce.com


Sreedhar’s C C E CURRENT AFFAIRS -JULY-2022

11. SCIENCE AND TECHNOLOGY


1. Which of the following IIT has Inaugurated New Census Data Workstation?
1) IIT Madras 2) IIT Delhi 3) IIT Bombay 4) IIT Kanpur 5) IIT Guwahati
[Explanation: The Census Data Workstation was officially opened by Dr. Vivek Joshi,
the Registrar General and Census Commissioner for India. According to Dr. Joshi, the
new workstation aims to make it easier for academics and researchers to access census

r’s
microdata.]
2. CBSE launches _______ portal to streamline board examination results.
1) E-patashala 2) Swayam Prabha 3) Swayam 4) NPTEL 5) ParikshaSangam
[Explanation: ParikshaSangam is another portal where students will be able to check
ha
Class 10th and Class 12th board results easily. The ParikshaSangam has been divided
into three main sections: Schools (Ganga), Regional offices (Yamuna), and Head office
(Saraswati).]
3.

E
Indian Navy Commissions the 1st ALH Squadron INAS 324 in which of the following
d
city?
1) Kochin 2) Visakhapatnam 3) Mumbai 4) Kolkata 5) Chennai
ee

[Explanation: INAS 324, named ‘Kestrels’, the 1st naval squadron on the eastern seaboard
operating the indigenously designed and built Advanced Light Helicopter (ALH) DHRUV
MK III (MR) helicopters.]
C
4. Which country has developed “CoVarScan” rapid test for detect ‘SARS-CoV-2’?
Sr

1) Germany 2) USA 3) Poland 4) France 5) Spain


[Explanation: CoVar Scan hones in on eight regions of SARS-CoV-2 that commonly differ
between viral variants. It detects small mutations — where the sequence of RNA building
blocks varies — and measures the length of repetitive genetic regions that tend to grow
and shrink as the virus evolves.]
5. Country’s first testbed for autonomous navigation launched at __________.
C
1) IIT Mumbai 2) IIT New Delhi 3) IIT Hyderabad 4) IIT Bangalore 5) IIT Chennai
[Explanation: TiHAN is developing and deploying areal-time Cyber-Physical System
utilizing unmanned aerial vehicles and unmanned ground/surface vehicles for many
application sectors of national importance. Among the most essential factors for making
autonomous and connected vehicles more acceptable to consumers is their effectiveness
in realistic conditions.]
6. Syngenta India has launched Biodiversity Sensor Project with _________.
1) IIT Bombay 2) IIT Ropar 3) IIT Kharagpur 4) IIT Madras 5) IIT Roorkee
[Explanation: Biodiversity Sensor Projectproject will help in studying the behavior of
insects around farms, particularly bees and butterflies. Along with this, the firm flagged
off the Drone Yatra which will cover 10,000 Km across the country.]

JULY- 57 Visitus:www.sr eedharscce.com


Sreedhar’s C C E CURRENT AFFAIRS -JULY-2022
7. DCGI has approved India’s first indigenously developed qHPV vaccine to treat
cervical cancer, this vaccine will be manufactured by __________.
1) Bharat Biotech 2) Biological E 3) Ranbaxy Lab
4) Serum Institute of India 5) Green signal BioPharma Ltd
[Explanation: qHPV vaccine CERVAVAC has demonstrated robust antibody response
that is nearly 1,000 times higher than the baseline against all targeted HPV types and in
all dose and age groups.]
8. Which among the following company has unveiled indigenous Li-ion cell?

r’s
1) Reliance 2) Mahindra 3) Hyundai 4) Ola Electric 5) Panasonic
[Explanation: The Bengaluru-based two-wheeler maker will begin the mass production
of the cell- NMC 2170, from its Chennai-based Gigafactory by 2023. The use of specific
chemicals and materials enables the cell to pack more energy in a given space and also
improves the overall life cycle of the cell.]
ha
9. Which company has developed world’s fastest graphics DRAM chip?
1) Skhynix 2) Nokia 3) Intel 4) Samsung 5) Micron
[Explanation: According to a statement from the manufacturer, the 24-gigabit Graphics

E
Double Data Rate 6 (GDDR6) uses third-generation, 10-nanometer technology and boasts
a data processing speed that is over 30% quicker than competing products.]
d
10. What is the name of First-Ever ‘Made In India’ surgical robotic system installed at
Rajiv Gandhi Institute.
ee

1) C-Astra 2) Mitra 3) SSI-Mantra 4) Asimov 5) Prithvi


[Explanation: “SSI-Mantra” devised by the new-age Indian med-tech start-up SS
Innovations. The ingenious SSI Mantra, a brainchild of World-renowned robotic cardiac
surgeon Dr Sudhir P Srivastava, will be signifying the beginning of a new era of surgical
C
procedures in India, making robotic surgery accessible and affordable for the people of
Sr

our country.]
11. Which of the following IIT has launched Smart Manufacturing and Digital
Transformation Centre (SMDTC)?
1) IIT Delhi 2) IIT Kanpur 3) IIT Bombay 4) IIT Madras 5) IIT Kharagpur
[Explanation: The centre will focus on collaborative research and end-to-end innovation
for smart process intelligence, 5G enablement and smart manufacturing supply chain
collaboration.]
C
12. Which country has confirmed its first outbreak of highly infectious Ebola-like
Marburg virus?
1) Gambia 2) Mali 3) Nigeria 4) Ghana 5) Senegal
[Explanation: The disease, a very infectious hemorrhagic fever in the same family as
Ebola, is spread to people by fruit bats and transmitted among people through direct contact
with bodily fluids of infected people and surfaces.]
13. Which space center has released next wave of images of universe from James
WebbSpace Telescope?
1) SpaceX 2) ISRO 3) NASA 4) European Space Agency 5) ESA
[Explanation: NASA’s James Webb Space Telescope is primarily designed to conduct
infrared astronomy that is expected to enable a broad range of investigations across the
fields of astronomy and cosmology. Learn more about NASA’s James Webb Space
Telescope through the series of questions and the answers provided with them below.]

JULY- 58 Visitus:www.sr eedharscce.com


Sreedhar’s C C E CURRENT AFFAIRS -JULY-2022

14. Which of the following has signed agreement with Cochin Shipyard to boost startups
inMaritime Sector?
1) IIM Ahmedabad 2) IIM Shillong 3) IIM Kozhikode
4) IIM Nagpur 5) IIM Bangalore
[Explanation: CSL and IIM-K’s business incubator Laboratory for Innovation Venturing
and Entrepreneurship (LIVE) will support the startups from the technical, regulatory,
financial, and marketing points of view. CSL said startups can get up to ?50 lakh as seed
grants.]

r’s
15. Which of the following IIT has got approval from MeitY to run IoT project under
Chips to Startups program?
1) IIT Roorkee 2) IIT Bhubaneswar 3) IIT Hyderabad
4) IIT Guwahati 5) IIT Dharwad
ha
[Explanation: The Chips to Startup program of the Ministry of Electronics and Information
Technology (MeitY) aims to train 85,000 high-quality engineers in Very large-scale
integration (VLSI) and Embedded System Design technologies.]
16.

E
Which of the following IIT has developed N-Treat Technology?
d
1) IIT Delhi 2) IIT Ropar 3) IIT Bombay 4) IIT Kanpur 5) IIT Kharagpur
[Explanation: N-Treat is a seven-stage process for waste treatment that uses screens,
ee

gates, silt traps, curtains of coconut fibres for filtration, and disinfection using sodium
hypochlorite. Its set up takes place within the Storm water drains or nullahs channels, that
is through the in-situ or on-site method of treatment, and does not require additional space.]
C
17. __________ has released a consultation paper on ‘Embedded Sim (e-Sim) for M2M
(machine-to-machine) Communications.
Sr

1) Mahanagar Telephone Nigam Ltd. (MTNL)


2) Bharat Sanchar Nigam Ltd. (BSNL)
3) Telecom Regulatory Authority of India (TRAI)
4) Association of Competitive Telecom Operators (ACTO)
5) Cellular Operators Association of India (COAI)
C
[Explanation: It is a form factor physically integrated into the device, mostly by soldering
to the device printedcircuit board (PCB).It requires remote provisioning, which is the ability
to remotely select the sim profile deployed on a Sim without physically changing the sim
card.The technology is standardised and can be implemented on a sim card with any
form factor.]
18. Tata Group has delivered indigenously developed Quick Reaction Fighting Vehicle
(QRFV) to the which of the following?
1) Indian Navy 2) Indian Air Force 3) Indian Army
4) Bharat Electronics Ltd 5) Hindustan Aeronautics Ltd
[Explanation: These armored vehicles will increase the nation’s protector’s ability to fight
in all weather and terrain situations and will offer protection while moving. The initial
group of QRFV was inducted by former Chief of Army Staff General MM Naravane.]

JULY- 59 Visitus:www.sr eedharscce.com


Sreedhar’s C C E CURRENT AFFAIRS -JULY-2022

12. SEMINARS AND MEETINGS


1. Which of the following state/UT is going to host next GST Council meeting?
1) Delhi 2) Tamil Nadu 3) Puducherry 4) Gujarat 5) Ladakh
[Explanation: This is for the first time the GST Council meeting will be held in Tamil
Nadu. According to the data available on the GST Council website and the PIB, majority of
its meetings were held in the national capital since its first meeting in September 2016.
The GST Council has 31 members representing the States and Union Territories. France

r’s
was the first country in the world to implement Goods and Services Tax (GST). More than
140 countries across the globe have implemented the GST.]
2. Indian Army has organized the “Suraksha Manthan 2022” at ______________.
1) Visakhapatnam 2) Kochi 3) Bilaspur
4) Jodhpur ha 5) Surat
[Explanation: The Desert Corps of the Indian Army organised the “Suraksha Manthan
2022”, on aspects of Border & Coastal Security at Jodhpur (Rajasthan). Present were
serving officers from the Army, BSF, and Coast Guard. The manthan was jointly presided
over by Shri Pankaj Kumar Singh, IPS, Director General Border Security Force (BSF),

3.

E
Director General VS Pathania]
Who has inaugurated the Zonal Meeting on ‘8 Years’ Achievements – Impact on
d
Women and Children’ in Hyderabad?
1) SmritiZubinIrani 2) Narendra Modi 3) Arjun Munda
ee

4) K. Chandrashekar Rao 5) Dharmendra Pradhan


[Explanation: The Zonal Meeting on ‘8 Years’ Achievements – Impact on Women and
Children’ was inaugurated in the city by the Union Minister for Women and Child
Development Smt Smriti Irani. The Minister for Tribal Welfare, Women and Child Welfare
C
of Telangana, SmtSatyavatiRathod was also present in the meeting.]
Sr

4. G20 Foreign Ministers’ Meeting (FMM) will be held in___________.


1) Brasília, Brazil 2) Jakarta, Indonesia 3) Bali, Indonesia
4) Delhi, India 5) Rome, Italy
[Explanation: it will be attended by External Affairs Minister S. Jaishankar. According to
a statement from the Ministry of External Affairs, the meeting’s foreign ministers will discuss
topics of current relevance, such as advancing multilateralism and current global concerns,
such as the security of food and energy. Under the Indonesian Presidency, the G20 in
C
2022 will focus on the theme “Recover Together, Recover Stronger”]
5. Who chaired the first meeting of the apex monitoring authority on the National
Industrial Corridor Development and Implementation Trust (NICDIT)?
1) Nitin Jairam Gadkari 2) Virendra Kumar
3) Jyotiraditya M. Scindia 4) Nirmala Sitharaman 5) Hardeep Singh Puri
[Explanation: NICDIT is India’s most ambitious infrastructure programme aiming to develop
new industrial cities as “Smart Cities” and converge next-generation technologies across
infrastructure sectors. Government of India constituted Apex Authority in August 2007.]
6. A two-day Regional Conference on the theme “Bringing Citizens, Entrepreneurs
and Government Closer for Good Governance” will be held at_______.
1) Hyderabad 2) Chennai 3) Gandhinagar 4) Mumbai 5) Bengaluru
[Explanation: The Regional Conference is being organized by the Department of
Administrative Reforms and Public Grievances in collaboration with Government of
Karnataka.More than 400 delegates from PAN-India are expected to participate in the
Conference. The Conference is being held in a semi-virtual mode.]

JULY- 60 Visitus:www.sr eedharscce.com


Sreedhar’s C C E CURRENT AFFAIRS -JULY-2022

7. Union Home and Cooperation Minister Amit Shah has Chaired the 30th meeting
of the Northern Zonal Council in __________.
1) Jaipur 2) Lucknow 3) Shimla 4) Kanpur 5) Agra
[Explanation: To discuss various issues of inter-state and Centre-state concern, including
the need for a coordinated strategy to fight the borderless cyber-crimes and the continued
expansion of banking network in rural areas. A total of 47 issues were discussed during
the northern zonal council meeting. ]
8. In which state PM Narendra Modi has organized a Natural Farming Conclave?
1) Surat 2) Gandhinagar 3) Ahmedabad 4) Vadodara 5) Jamnagar

r’s
[Explanation: Consequently, at least 75 farmers were identified in each gram panchayat
and were motivated and trained to undertake natural farming. The farmers were trained
in 90 different clusters resulting in training of more than 41,000 farmers across the district.
The conclave will also be attended by the Governor and Chief Minister of Gujarat.]
ha
9. Which countries are participating in the I2U2 summit?
1) India 2) Israel 3) USA 4) UAE 5) All of the Above
[Explanation: Prime Minister Narendra Modi participated in the first virtual I2U2 Summit.
The I2U2 is a four-nation grouping, where “I” stands for India and Israel, and “U” for the

E
US and the UAE. PM Modi was joined by United States President Joe Biden, Israeli PM
d
YairLapid and UAE President Mohammed bin Zayed Al Nahyan. ?I2U2 is aimed to
encourage joint investments in six mutually identified areas such as water, energy,
transportation, space, health, and food security.]
ee

10. “Sea Guardians-2” is a joint maritime exercise conducted by which countries?


1) China-Nepal 2) Pakistan-Nepal 3) Bangladesh-Bhutan
4) China-Russia 5) China-Pakistan
C
[Explanation: The joint exercise aims to enhance defence cooperation, exchange expertise
Sr

and experience, deepen the traditional friendship between the two countries and militaries,
and promote the development of the China-Pakistan all-weather strategic cooperative
partnership. This four-day joint maritime exercise between China and Pakistan kicked off
at a military port in Shanghai.]
11. A two-day Meeting of BIMSTEC expert group on cyber security cooperation has
begun in ________.
1) Gujarat 2) Maharashtra 3) New Delhi
C
4) West Bengal 5) Ladakh
[Explanation: The BIMSTEC Expert Group will create an Action plan to address cyber
security concerns in the BIMSTEC Region, as agreed upon during the conference of the
BIMSTEC National Security Chiefs held in Bangkok in March 2019. This summit on cyber
security cooperation was organised by India, the lead country for the security sector in
the BIMSTEC forum, who also came up with a plan of action for cyber security.]
12. Third Annual Conference of Indian Information Service Officers has begun in
_________.
1) Maharashtra 2) New Delhi 3) Gujarat
4) Ladakh 5) Jammu and Kashmir
[Explanation: Union Minister for Information and Broadcasting, Anurag Singh Thakur
inaugurated the Third Annual Conference of Indian Information Service Officers at
VigyanBhavan.]

JULY- 61 Visitus:www.sr eedharscce.com


Sreedhar’s C C E CURRENT AFFAIRS -JULY-2022

13. Which of the following country will host the SCO summit in 2023?
1) Iran 2) Belarus 3) India 4) Kazakhstan 5) China
[Explanation: Shanghai Cooperation Organisation (SCO) summit 2022 is planned to held
in September 2022 in Samarkand, Uzbekistan. Varanasi has been selected as the SCO
region’s first “Tourism and Cultural Capital”, a title it will hold next year coinciding with
India chairing the SCO grouping. The current SCO Secretary-General, General Zhang Ming,
a veteran Chinese diplomat.]
14. A Maritime Partnership Exercise (MPX) was conducted between Which country
Defense Force and Indian Navy in the Andaman Sea?

r’s
1) USA 2) Sri Lanka 3) Japan 4) Russia 5) France
[Explanation: The exercises were aimed at enhancing interoperability and streamlining
seamanship and communication procedures. This exercise is part of the ongoing efforts
between the two navies towards ensuring safe and secure international shipping and
ha
trade in the Indian Ocean Region (IOR).The two countries have been carrying out regular
exercises in IOR towards reinforcing maritime association.]
15. Who among the following has chaired the National Family Planning Summit2022?
1) Amit Shah 2) Narendra Modi 3) Bharati Pravin Pawar
4) Mansukh L. Mandaviya

E 5) AnuragThakur
d
[Explanation: This year’s theme, “Family Planning & Universal Health Coverage: Innovate.
Collaborate. Accelerate.” Family planning saves lives. It preserves women’s and girls’
health, and empowers them to pursue education and work. It boosts their ability to save,
ee

contribute to the economy, and invest in the health and education of their children.]
16. Which nation will host the SCO Foreign Ministers Meeting?
1) Uzbekistan 2) Indonesia 3) Australia 4) Slovenia 5) Syria
C
[Explanation: External Affairs Minister S. Jaishankar will come face to face with Pakistan’s
Foreign Minister Bilawal Bhutto for the first time on July 28, at the Shanghai Cooperation
Sr

Organisation (SCO) Ministerial meeting.]


17. PM Modi has addressed the inaugural session of first All India District Legal
Services Authorities Meet in __________.
1) Surat 2) Delhi 3) Mumbai 4) Kolkata 5) Chennai
[Explanation: District Legal Services Authorities (DLSAs) is being organised from 30-31
July 2022 at VigyanBhawan by National Legal Services Authority (NALSA). There are a
C
total of 676 District Legal Services Authorities (DLSAs) in the country.]
18. Which of the following countries have hold a trilateral meeting to boost maritime
security?
1) India, Japan & Russia 2) India, France&UAE 3) China, USA & India
4) Russia, China & Japan 5) France, India & Japan
[Explanation: A trilateral meeting of the Focal Points of India, France and the United Arab
Emirates where three sides exchanged views on the Indo-Pacific region and explored the
prospects of maritime security among other areas of cooperation.]
19. Chahabar Day conference has begun in________.
1) New Delhi 2) Gandhinagar 3) Leh 4) Mumbai 5) Bangalore
[Explanation: Union Shipping Minister Sarbanand Sonowal and Minister of State for
Shipping ShripadNaik inaugurated the Chabahar Day conference in Mumbai. The Chabahar
port is a key pillar of India’s Indo-Pacific vision to connect with Eurasia with Indian Ocean
Region.]

JULY- 62 Visitus:www.sr eedharscce.com


Sreedhar’s C C E CURRENT AFFAIRS -JULY-2022

13. SPECIAL DAYS


1. National Doctors’ Day 2022 is observed on__________.
1) July 2 2) June 30 3) July 1 4) July 3 5) June 29
[Explanation: July 1 happens to be the birth anniversary of DrBidhan Chandra Roy. He
was conferred with the Bharat Ratna, the country’s topmost civilian honour, in 1961, a
year before his death. The purpose of National Doctor’s Day is to recognise the duties and
responsibilities of medical professionals and to increase public awareness of the critical
role they play in saving lives.]
2. National Chartered Accountants Day 2022is observed on__________.

r’s
1) June 1 2) June 5 3) July 2 4) July 1 5) July 3
[Explanation: The day is observed in order to celebrate the formation of the Institute of
Chartered Accountants of India (ICAI) on July 1, 1949. The ICAI is the only licensing and
regulatory body for the financial audit and accounting profession in the country.]
3. World Unidentified Flying Objects Day is celebrated on_______.
1) July 11 2) July 01
ha 3) July08 4) July 02 5) July 05
[Explanation: The day is celebrated with an aim to raise awareness about the undoubted
existence of UFOs, and also the intelligent beings from outer space. Unidentified Flying
Objects (UFO) are the special aircrafts on which people believe that aliens and other heavenly
bodies come to visit the Earth. On World UFO Day people keep a look at the sky to get the
view of these objects. In this article we have discussed the History, Significance of World
4.
UFO Day 2022.]

E
International Day of Cooperatives day annually celebrated on _________.
d
1) The first Tuesday of July 2) The second Sunday of July
3) First Monday of July 4) First Saturday of July 5) First Friday of July
[Explanation: International Day of Cooperatives is marked annually on the first Saturday
ee

of July. This year, the day will be observed on 2 July to highlight the contributions of the
cooperative movement. The aim of this celebration is to increase awareness about
cooperatives by highlighting how the United Nations shares similar goals and objectives
as the international cooperative movement.]
C
5. World Sports Journalists Day 2022 is celebrated on _________.
1) July 03 2) July 02 3) July 10 4) July 15 5) July 11
Sr

[Explanation: World Sports Journalists Day marks the creation of the International Sports
Press Association (AIPS). In 1994, World Sports Journalists Day was first celebrated by
the AIPS to celebrate the anniversary of its establishment and commemorate the great work
done by sports journalists around the world. It is observed to acknowledge and encourage
the work of sports journalists across the globe.]
6. International Plastic Bag Free Day 2022 is observed on__________.
1) July 02 2) July 11 3) July 18 4) July 03 5) July 12
[Explanation: The aim of International Plastic Bag Free Day is to create awareness about
the environmental harm we are causing by using single-use plastics. Aim to encourage
C
countries to ban single-use plastic. Humans produce roughly 300 million tons of plastic
each year, most of which cannot be recycled. World-wide only 10-13% of plastic items are
recycled. Moreover, the process of recycling plastics is complicated.]
7. World Zoonosis Day 2022 is observed on___________.
1) July 03 2) July 11 3) July01 4) July 06 5) July18
[Explanation: World Zoonoses Day is held every year on July 6 to commemorate the
scientific achievement of the first vaccination against zoonotic disease. Zoonoses are
infectious diseases caused by bacteria, viruses, and parasites spread between animals
and humans. World Zoonoses Day gives us an idea of the fact how the health of humans
is dependent on the health of animals.]
8. National Fish Farmers Day is being celebrated on _________.
1) July 01 2) July 11 3) July 02 4) July 10 5) July 18
[Explanation: National Fish Farmers Day is celebrated in the memory of scientists Dr K.
H. Alikunhi and Dr H. L. Chaudhury. Both of them successfully demonstrated
Hypophysation (technology of induced breeding) in Indian Major Carps (common name
for several species of fish) on 10th July 1957. National Fish Farmer’s Day is celebrated
every year to demonstrate solidarity with all fisher folk, fish farmers and concerned
stakeholders throughout the country.]

JULY- 63 Visitus:www.sr eedharscce.com


Sreedhar’s C C E CURRENT AFFAIRS -JULY-2022
9. World Population Day 2022 is observed globally on ___________.
1) July 11 2) July 01 3) July 18 4) July 12 5) July 03
[Explanation: world Population Day is observed every year to raise awareness about
global population issues. The theme of World Population Day 2022 is for a universe of 8
billion which expects a strong future for all guaranteeing freedom and decisions for all.
The slogan of the day is ‘Our earth is in danger of over population, control the population
and save the earth’.]
10. International Malala Day is celebrated every year on________.
1) July 11 2) July 01 3) July 07 4) July 12 5) July 02
[Explanation: Every year on July 12th, the world celebrates the birthday of Malala
Yousafzai. At age 17, Malala was the youngest recipient ever of the Nobel Peace Prize. The

r’s
day also honors the rights of children and women. She along with her father started the
Malala Fund in 2013 ‘to raise awareness on impact of girls’ education. ]
11 World Paper Bag Day 2022 is observed on_________.
1) July 11 2) July 02 3) July 12 4) July 01 5) July 07
[Explanation: The theme of the day ‘to raise awareness about important environmental
issues like climate change and to promote a more sustainable lifestyle by introducing paper
ha
bags as alternatives to plastic bags’. In 1871, inventor Margaret E. Knight designed a
machine that could create flat-bottomed paper bags.]
12. World Youth Skills Day 2022 is observed on__________.
1) July 11 2) July 01 3) July 12 4) July 15 5) January 12
[Explanation: The primary purpose of World Youth Skills Day is to empower young

E
people with important skills. The theme of World Youth Skills Day 2022 is ‘Transforming
youth skills for the future’. The youth forms the backbone of any nation and is instrumental
d
in shaping a better future for the country.]
13. The World Day for International Justice is observed every year on__________.
1) July 16 2) July 17 3) June 24 4) May 12 5) March 15
ee

[Explanation: The day aims ‘to raise awareness about the crimes that put peace, security
and well-being of the world at risk’. World Day for International Justice marks the
anniversary of the adoption of the Rome Statute. The day focuses on the advocacy of
fundamental human rights and the promotion of international criminal justice.]
C
14. World Snake day is observed every year on __________.
1) July 13 2) Feb 21 3) April 18 4) July 16 5) June 27
Sr

[Explanation: World Snake Day was created ‘to raise awareness among people about
these reptiles and how much they contribute to the world and its different ecosystems’.
Without snakes, the number of prey species would increase to unnatural levels and destroy
the stability of the ecosystem.]
15. World Emoji Day is observed globally on __________
1) July 16 2) June 21 3) July 17 4) May 16 5) April 25
[Explanation: The purpose of World Emoji Day is ‘to promote the use emojis and spread
the enjoyment that they bring to all of those around us’. The emoji were created for a very
specific purpose: ease of communication on a nascent mobile internet system.]
C
16 Nelson Mandela International Day 2022 is observed on_________.
1) July 11 2) July 17 3) July 01 4) July 18 5) July 12
[Explanation: July 18 happens to be the birth anniversary of Nelson Mandela. Nelson
Mandela International Day celebrates the idea that each individual has the power to
transform the world and the ability to make an impact. Nelson Mandela Day, under the
theme “Do what you can, with what you have, where you are”.]
17. Which day was celebrated as the Tamil Nadu Day?
1) July 12 2) July 02 3) July 17 4) July 18 5) July 01
[Explanation: It is celebrated in the South Indian state of Tamil Nadu to commemorate the
formation of the state. Tamil Nadu was created on 1 November 1956 with the name Madras
State. On 18 July 1967, Madras State was officially renamed to Tamil Nadu.]
18. Mangal Pandey birth anniversary is Celebrated on ____________.
1) July 06 2) July 11 3) July 19 4) July 17 5) July 29
[Explanation: Mangal Pandey is one of India’s most celebrated freedom fighters. His
birth anniversary(July 19) as the nation remembers his sacrifice for the freedom of the
country. In 1849, Pandey joined the army of the British East India Company and served
as a sepoy in the 6th Company of the 34th Bengal Native Infantry in Barrackpore.]

JULY- 64 Visitus:www.sr eedharscce.com


Sreedhar’s C C E CURRENT AFFAIRS -JULY-2022
19. International Chess Day 2022 is observed on _______.
1) July 18 2) July 26 3) July 12 4) July 20 5) July 11
[Explanation: World Chess Day is also known as International Chess Day because it is
celebrated globally. The main reason behind International Chess Day being celebrated on
20 July is because on this date the International Chess Federation (FIDE) was founded in
the year 1924. The motto of chess is “Gens unasumus”, which is Latin for “We are one
people”.chess was invented by Grand Vizier Sissa Ben Dahir, and given as a gift to King
Shirham of India.]
20. International Moon Day is observed annually on _________.
1) July 17 2) July 20 3) July 01 4) July 29 5) July 22

r’s
[Explanation: Annually, the International Moon Day will be celebrated to educate and
promote the need for exploration of lunar activities and the sustainable use of the moon. n
1971, then US President Richard Nixon proclaimed July 20 as National Moon Day to honour
the anniversary of man’s first moon landing. The US celebrates National Moon Day every
year on July 20.]
21. World Brain Day was celebrated globally on __________.
1) July 12
ha
2) July 18 3) July 22 4) July 11 5) July 26
[Explanation:World Brain Day is observed on July 22 every year to create awareness
on brain health. The World Federation of Neurology which was established on the same
day in 1957 had proposed to observe the day as World Brain Day in the year 2014.World

E
Brain Day 2021 is dedicated to multiple sclerosis (MS), a neurological disease that affects
d
2.8 million people of all ages globally.]
22. National Broadcasting Day 2022 is observed on _______.
1) July 17 2) July 22 3) July 18 4) July 23 5) July 12
ee

[Explanation: India celebrates National Broadcasting Day to commemorate the first radio
broadcast in India from the Bombay Station under the Indian Broadcasting Company in
1927. It aims to remind us about the impact of radio on our lives.]
23. National Flag Adoption Day 2022 is observed on __________.
C
1) July 20 2) July 26 3) July 22 4) July 28 5) July 12
[Explanation: It was adopted in its present form during a meeting of the Constituent
Sr

Assembly held on 22 July 1947, and it became the official flag of the Dominion of India on
15 August 1947. The flag was subsequently retained as that of the Republic of India. This
led to a new flag which was designed by PingaliVenkayya, an Indian freedom fighter.]
24. Pi Approximation Day 2022 is observed on __________.
1) July 14 2) May 14 3) July 22 4) March 14 5) July 11
[Explanation:U.S. Pi or pie, whether you’re a baker or a math whiz, July 22is your day
— Pi Approximation Day on July 22 honors the concept of pi, which is denoted by the
Greek letter pi and approximates to 3.14, in the most mathematically-pleasing way. Ancient
C
Babylonians used it — approximated to 3.125, to calculate the dimensions of circles. It
was around 250 B.C. that pi was first calculated by one of the greatest ancient
mathematicians, Archimedes of Syracuse.]
25. World Fragile X Day 2022 was observed on ________.
1) July 20 2) July 12 3) July 11 4) July 28 5) July 22
[Explanation: On this day, communities around the world come together to shine a light
on Fragile X, by illuminating monuments and landmarks globally. World Fragile X Day
celebrates families impacted by Fragile X and highlights advancements of research to find
effective treatments and ultimately a cure.]
26. World Drowning Prevention Day is observed on _________.
1) July 24 2) July 20 3) July 26 4) July 23 5) July 25
[Explanation:The main purpose of the day is to encourage individuals, governments, UN
agencies and civil society organisations to collaborate and provide critical measures to
prevent drowning deaths. According to a World Health Organization report, an estimated
236,000 people lose their lives each year by drowning.

JULY- 65 Visitus:www.sr eedharscce.com


Sreedhar’s C C E CURRENT AFFAIRS -JULY-2022
27. The 163rd anniversary of Income Tax Day is observedon _________.
1) July 22 2) July 11 3) July 17 4) July 12 5) July 24
[Explanation: The 163rd anniversary of Income Tax Day was observed by Central Board
of Direct Taxes (CBDT). The Finance Minister complimented the Department for having
achieved the highest ever revenue collection of over 14 lakh crore in the last fiscal.]
28. 23rd anniversary of Kargil Vijay Diwas 2022 is observed on __________.
1) July 23 2) July 20 3) July 24 4) July 26 5) July 18
[Explanation: The day is observed as ‘Kargil Vijay Diwas’ to commemorate India’s victory
over Pakistan in the war. This day celebrates the victory and sacrifices of our soldiers.
One of the significant wars fought in the Indian history, Kargil war, was fought between

r’s
May to July 1999 in the Kargil- Drass sector of Jammu and Kashmir when the Pakistani
troops had unlawfully invaded certain areas alongside the Line of Control (LOC).]
29. Dr. APJ Abdul Kalam’s 6thDeath Anniversary is celebrated on ___________.
1) July 15 2) July 11 3) July 27 4) July 26 5) July 28
[Ex planation: seventh death anniversary of the “Missil e Man of India”
AvulPakirJainulabdeen Abdul Kalam or Dr APJ Abdul Kalam. DrKalam is hailed for his
ha
contribution to the space and science projects of the country while he also played an
instrumental role in conducting the 1998 nuclear weapon test. APJ Abdul Kalam also known
as the Missile Man of India who served as the 11th President of India was a huge inspiration
for lakhs of Indians.]
30.

E
International Day for the Conservation of the Mangrove Ecosystem 2022 was
observed on _________.
d
1) July 24 2) July 26 3) July 20 4) July 28 5) July 23
[Explanation: The International Day for the Conservation of the Mangrove Ecosystem,
adopted by the General Conference of UNESCO in 2015 and celebrated each year on 26
ee

July, aims to raise awareness of the importance of mangrove ecosystems as “a unique,


special and vulnerable ecosystem” and to promote solutions for their sustainable .]
31. 83rd Raising Day of Central Reserve Police Force(CRPF) was being celebrated on
__________.
C
1) July 29 2) July 28 3) July 23 4) July 27 5) July 20
[Explanation: The motto “tejsvinavdhitmastu” means- “Let Brilliance Emerge out of our
Sr

Endeavours”. July 27 is CRPF foundation day as India’s largest central armed police
force came into existence on this day in 1939. One of the foremost central police forces in
India CRPF is amongst the most gallant security forces.]
32. World Nature Conservation Day is celebrated on ________.
1) July 29 2) July 11 3) July 01 4) July 28 5) July 20
[Explanation: It aims to create awareness about the importance of natural resources and
the conservation around them. The World Nature Conservation Day is a reminder to us to
C
stop compromising the natural resources unnecessarily and stay in sync with Mother
Nature. The day is observed globally to pay tribute the Mother Nature for its bounty.]
33. World Hepatitis Day is observed on _________.
1) 27 July 2) 25July 3) 28 July 4) 15 July 5) 12 July
[Explanation: Hepatitis means inflammation of the liver. World Hepatitis Day, celebrated
annually on July 28th, is an opportunity to raise awareness of the global burden of viral
hepatitis and to recognize the discovery of the Hepatitis B virus in 1967 by Nobel prize-
winning scientist Dr. Baruch Blumberg and his subsequent achievements in developing a
diagnostic test.]
34. “International Tiger Day” is observed every year on ___________.
1) July 17 2) July 26 3) July 11 4) July 29 5) July 18
[Explanation: Global Tiger Day, often called International Tiger Day, is an annual
celebration to raise awareness for tiger conservation, held annually on 29 July. It was
created in 2010 at the Saint Petersburg Tiger Summit in Russia. Every year the day is
celebrated under different themes, and this year the theme for Global Tiger Day is “India
launches Project Tiger to revive the tiger population”.]

JULY- 66 Visitus:www.sr eedharscce.com


Sreedhar’s C C E CURRENT AFFAIRS -JULY-2022

14. SPORTS
1. Who hasset a new national record with an opening throw of 89.94m at the
Stockholm Diamond League?
1) Shivpal Singh 2) Davinder Singh Kang
3) Neeraj Chopra 4) Annu Rani 5) Anderson Peters
[Explanation: Neeraj Chopra returned to Diamond League action for the first time in 4
years in style as he broke his own national record for the second time in a month. Neeraj

r’s
finished 2nd at the Stockholm Diamond League. Tokyo Olympics champion Neeraj Chopra
nearly achieved his mission of throwing 90m as he hurled the javelin to his personal best
distance in his first attempt in the Diamond League.]
2. Who has set a National Record in women’s 3000 metres at UA Sunset Tour in Los
Angeles? ha
1) Mithali Dorai Raj 2) Parul Chaudhary 3) Suriya Loganathan
4) Dutee Chand 5) Rahi Sarnobat
[Explanation: Indian runner Parul Chaudhary smashed the national record at the Sound
Running meet in Los Angeles as she became the first athlete from the country to clock a

E
sub-nine minute time in the women’s 3000m event. She had won the gold in women’s
3000m steeplechase at the nationals]
d
3. Who has won the Men’s singles title at the 2022 Malaysia Open?
1) KentoMomota 2) Viktor Axelsen 3) TakuroHoki
ee

4) Yugo Kobayashi 5) Zheng Siwei


[Explanation: Tokyo champion Viktor Axelsen and Thailand’s Ratchanok Intanon won
the singles title at the 2022 Malaysia Open. World champions Takuro Hoki and Yugo
Kobayashi (Japan) won the men’s doubles title.Viktor Axelsen Belongs to Denmark]
C
4. Who has won German junior open squash championship super series event, held
Sr

at Hamburg?
1) Malak Samir 2) Whitney Isabelle Wilson 3) Anahat Singh
4) Savannah Moxham 5) Anika Dubey
[Explanation: India’s 14-year-old Anahat Singh, did the country proud by winning the
German Junior Open Squash Championship Super Series held in Hamburg, Germany. At
the age of 6, Anahat was playing badminton and won several tournaments in Delhi. She
won a bronze at the Asian Juniors and Silver in the Malaysian Junior Open.]
C
5. Who among the following has won the British F1 Grand Prix 2022?
1) Lewis Hamilton 2) Carlos Sainz Jr. 3) Sergio Pérez
4) Max Verstappen 5) Charles Leclerc
[Explanation: Carlos Sainz cruised to British Grand Prix victory, earning his first career
Formula 1 win in 150 races. Red Bull’s Sergio Pérez crossed the line seconds ahead of
Mercedes’ Lewis Hamilton to round out the podium.Silverstone hosted the first Formula
One Grand Prix championship race in May 1950, won by Giuseppe “Nino” Farina in an
Alfa Romeo.]
6. 15th edition of the Asia Cup Cricket Tournament 2022 is going to be held in which
country?
1) Australia 2) Sri Lanka 3) Pakistan 4) Brazil 5) Austria
[Explanation: The first Asia Cup was held in 1984 in Sharjah, United Arab Emirates.India
and Pakistan are likely to clash in the Asia Cup 2022 ahead of the T20 World Cup 2022 in
Australia, as per reports in Sri Lanka media. capital Sri: Jayewardenapura Kotte, currency:
Rupee]

JULY- 67 Visitus:www.sr eedharscce.com


Sreedhar’s C C E CURRENT AFFAIRS -JULY-2022

7. Which country’s Cricket board has announced agreement for equal pay to male
and female cricketers?
1) Australia 2) Pakistan 3) New Zealand 4) Ireland 5) England
[Explanation: The five-year-long deal has been signed between NZC, six Major
Associations, and the New Zealand Cricket Players Association (NZCPA). According to it,
the White Ferns and domestic women’s players will receive the same match fees as the
men across all their formats and competitions.]
8. Who among the following has won gold medal at Elorda Cup in Kazakhstan?

r’s
1) Gitika 2) AlfiyaPathan 3) Kalaivani Srinivasan
4) Both 1 & 2 5) Both B & C
[Explanation: Two other Indian female boxers Kalaivani Srinivasan and Jamuna Boro
signed off with silver medals. With the addition of two gold, two silver, and 10 bronze
medals on the final day, the 33-member Indian contingent concluded its campaign with an
ha
overall tally of 14 medals. The tournament witnessed top players in action from countries
such as India, Uzbekistan, and hosts Kazakhstan, Cuba, China, and Mongolia.]
9. Which state Govt will host the 36th National Games 2022?
1) Haryana

E
2) Odisha 3) Gujarat 4) Kerala 5) Manipur
d
[Explanation: Due to a number of factors, including the coronavirus pandemic since 2020,
the prestigious event is being hosted after a seven-year hiatus; the previous one was in
Kerala in 2015. The chief minister claimed in another post that Gujarat has world-class
ee

athletic facilities and that the sporting world is experiencing a resurgence. ]


10. Who won the Austrian F1 grand prix 2022?
1) Esteban Ocon 2) George Russell 3) Max Verstappen
C
4) Charles Leclerc 5) Lewis Hamilton
Sr

[Explanation: Red Bull’s Max Verstappen had to settle for second and Mercedes’ seven-
time world champion Lewis Hamilton was a distant third, after starting eighth, to complete
the podium for the third race in a row. Capital city: Vienna, currency: Euro. ]
11. Who among the following has won Wimbledon Men’s Singles title2022?
1) Novak Djokovic 2) Nick Kyrgios 3) Rafael Nadal
4) Roger Federer 5) Andy Murray
C
[Explanation: Serbia Novak Djokovic won a seventh Wimbledon men’s title and 21st
Grand Slam crown with a four-set triumph over Nick Kyrgios.Novak Djokovic was
Serbian tennis player. His 21 Grand Slam singles titles included a record nine Australian
Open championships]
12. Who among the following has won bronze medal in the compound mixed team
event at the World Games ?
1) Abhishek Verma 2) Jyothi Surekha Vennam
3) Andrea Becerra 4) Both 1&2 5) Both 2&3
[Explanation: According to an Archery Association of India, this was India’s first medal
at the World Games. It was Verma’s 50th podium finish at the international level. Verma is
now the only Indian archer to win medals at the World Games, World Championship,
World Cup Final, World Cup, Asian Games and Asian Championship. the compound mixed
team archery event at the World Games 2022 in Birmingham, Alabama, USA .]

JULY- 68 Visitus:www.sr eedharscce.com


Sreedhar’s C C E CURRENT AFFAIRS -JULY-2022

13. Who has won the gold medal at World Maters Athletics Championships in Finland?
1) Bhagwani Devi 2) MJ Jacob 3) Warren Weir
4) Justin Gatlin 5) Steven Gardiner
[Explanation: Bhagwani Devi Dagar, a 94-year-old from Najafgarh won the gold medal
in 100m sprint, bronze in shot put and another bronze in the discus throw at World Master
Athletics Championship in Finland. She also made a national record of 24.74 sec in the
100m sprint in the event. Bhagwani Devi had won three gold medals at the National Masters
Athletics Championships in Chennai].
14. Who has won the Gijon Chess Masters title?

r’s
1) P. Harikrishna 2) D. Gukesh 3) Alexandr Fier
4) Viswanathan Anand 5) Pedro Antonio
[Explanation: Gukesh could well become the sixth Indian to do so after Viswanathan
Anand, Krishnan Sasikiran, P. Harikrishna, Vidit Gujarati and B. Adhiban. India’s D.
ha
Gukesh won the Gijon Chess Masters, with eight points out of nine rounds. GM Alexandr
Fier of Brazil finished second while International Master Pedro Antonio Gines of Spain
came third.]
15.

E
Who has won first gold medal at ISSF World Cup in South Korea?
1) Arjun Babuta 2) Lucas Kozeniesky 3) Sergey Richter
d
4) PaarthMakhija 5) Sergey Richter
[Explanation: The first gold medal at ISSF World Cup was held in 1897. Arjun Babuta of
ee

India earned his first gold medal in shooting at the Changwon, South Korea, International
Shooting Sport Federation (ISSF) World Cup stage. Final saw him overcome silver medalist
for Tokyo 2020 Lucas Kozeniesky]
16. Who won ICC Men’s Player of the Month for June 2022?
C
1) Jonny Bairstow 2) Keshav Maharaj 3) Angelo Mathews
Sr

4) Daryl Mitchell 5) Babar Azam


[Explanation: It was the second-fastest century ever scored by an English player in Test
cricket. Jonathan Marc Bairstow is an English cricketer who plays internationally for
England in all formats. The International Cricket Council (ICC) is the world governing body
of cricket. The 2023 ICC Men’s Cricket World Cup will be the 13th edition of the men’s
Cricket World Cup, scheduled to be hosted by India during October and November 2023.]
C
17. Who among the following havebagged ICC Player of the Month award for June?
1) Jonny Bairstow 2) Marizanne Kapp 3) Angelo Mathews
4) Both 1&2 5) Both 2&3
[Explanation: Angelo Mathews and Tuba Hassan crowned ICC Players of the Month for
May. South Africa’s Kapp was selected the ICC Women’s Player of the Month. England’s
Sophie Ecclestone has become the ICC Player of the Month for June 2021. · New Zealand
opener Devon Conway bagged the honour in the men’s category. ]
18. World Athletics Championships 2022 has begun in __________.
1) Beijing 2) Berlin 3) Eugene 4) Ulaanbaatar 5) Cairo
[Explanation: The World Championships were started in 1976. The 2022 Championships
is the 18th edition and the US is hosting the most prestigious event for the first time. For
India, Olympic javelin champion Neeraj Chopra will lead the country’s charge. World
Athletics Championships Organised by World Athletics.]

JULY- 69 Visitus:www.sr eedharscce.com


Sreedhar’s C C E CURRENT AFFAIRS -JULY-2022
19. Who has won bronze medalin the women’s 50m rifle 3 positions event at the ISSF
Shooting World Cup.?
1) Anna Janssen 2) Barbara Gambaro 3) Anjum Moudgil
4) Abhinav Bindra 5) Jaspal Rana
[Explanation: This is Anjum Moudgil’s second straight shooting World Cup medal of the
year. This is the 28-year-old shooter’s second individual medal in consecutive World Cup
stages. She had won silver in the same event at the Baku World Cup. Anjum, a world
championship silver medallist, had qualified for the finals after finishing sixth in the ranking
round.]

r’s
20. Who has won gold medal in Men’s 50m Rifle 3 at the ISSF shooting world cup in
Changwon, Korea?
1) ZakanPekler 2) Manu Bhaker 3) Vijay Kumar
4) Aishwary Pratap Singh Tomar 5) Saurabh Chaudhary
[Explanation: Aishwary Pratap Singh Tomar is an Indian sport shooter. He won the bronze
ha
medal in the 50 metre rifle three positions event at the 2019 Asian Shooting Championships
and secured a quota place for India at the 2020 Summer Olympics. He won ‘Eklavya
Award’ by Government of Madhya Pradesh on National Sports Day References in 2019
21. Who among the following has clinched Singapore Super 500 trophy?
1) P.V. Sindhu

E 2) Wang Zhi Yi 3) Akane Yamaguchi


d
4) An Se-young 5) Carolina Marín
[Explanation: PV Sindhu won the first Super 500 title of her career when she defeated
reigning Asian champion Wang Zhi Yi of China in the final of the Singapore Open. This is
ee

her first 500 or better since the world championship win in 2019.she won awards Arjuna
Award (2013), Padma Shri (2015), Major Dhyan Chand Khel Ratna(2016), BBC
Sporstwoman of the Year (2019).]
22. Who among the following has won Paracin Open Chess title in Serbia?
C
1) R Praggnanandhaa 2) AlexandrPredke 3) Alisher Suleymenov
Sr

4) AL Muthaiah 5) D Gukesh
[Explanation: The Indian chess prodigy, Praggnanandhaa is just 16 years old and has
already achieved great success in the game. Praggnanandhaa, who became chess
Grandmaster at 12. Praggnanandhaa scored 8 points from nine rounds. Alexandr Predke
took the second spot.]
23. Who has become first Indian to enter men’s long jump final at World Athletics
Championships?
C
1) Murali Sreeshankar 2) Avinash Sable 3) Sandeep Kumar
4) Tajinderpal Singh 5) Priyanka Goswami
[Explanation: Murali Sreeshankar became the first Indian male long jumper to qualify for
World Athletics Championship.The men’s long jump at the 2022 World Athletics
Championships was held at the Hayward Field in Eugene. Anju Bobby George was the
first Indian to make it to the World Championships long jump finals and first to win a medal
— bronze in 2003 edition in Paris.]
24. ________has been selected as host city for the 2025 World Athletics Championships.
1) Eugene 2) Tokyo 3) Silesia 4) Nairobi 5) Budapest
[Explanation: The 2022 World Athletics Championships, the eighteenth edition of the
World Athletics Championships, are currently being held at Hayward Field in Eugene,
Oregon, United States. The nineteenth edition of the World Athletics Championships, are
scheduled 2023 in Budapest, Hungary. The World Athletics Cross Country
Championships will return to the USA for the first time since 1992.]

JULY- 70 Visitus:www.sr eedharscce.com


Sreedhar’s C C E CURRENT AFFAIRS -JULY-2022
25. Who has won FIH Women’s Hockey World Cup 2022?
1) Netherlands 2) Argentina 3) Germany
4) India 5) Australia
[Explanation: The Fédération Internationale de Hockey (English: International Hockey
Federation), commonly known by the acronym FIH. Its headquarters are in Lausanne,
Switzerland. England claimed the bronze medal in the third-place match after defeating
India in the shoot-out. Capital: Amsterdam, Currency: Euro]
26. Who has become the first Indian towin gold medal in the men’s skeet event at
ISSF?

r’s
1) Mairaj Ahmad Khan 2) Minsu Kim 3) Ben Llewellin
4) Sanjeev Rajput 5) Aishwary Tomar
[Explanation: the first Indian shooter to win the gold medal in the men’s skeet event at
the ISSF World Cup in Changwon, South Korea. Mairaj, who hails from Uttar Pradesh,
finished ahead of Korea’s Minsu Kim (36) and Britain’s Ben Llewellin (26), who won the
ha
silver and bronze respectively.He won the Olympics quota in the Shotgun Skeet Event in
Lonato, Italy.]
27. The 2028 Summer Olympic and Paralympic Games will be hosted in ______.
1) Los Angeles
4) Brisbane

E 2) Paris
5) Tokyo
3) Beijing
d
[Explanation: Los Angeles has previously hosted the Olympics in 1984 and 1932. 15,000
athletes are expected to compete in the Olympic and Paralympic Games in Los Angeles. In
ee

Beijing, People’s Republic of China, from 4 to 20 February 2022. In Paris, France, from 26
July to 11 August 2024. In Milan and Cortina d’Ampezzo, in Italy, in 2026. Baron Pierre
de Coubertin founded the International Olympic Committee (IOC) in 1894, leading to the
first modern Games in Athens in 1896.]
C
28. The 131stEdition of the Durand Cup will be held with a multi-city format. This cup
Sr

is related to which sports?


1) Cricket 2) Tennis 3) Hockey 4) Football 5) Golf
[Explanation: The tournament, which is jointly organised by Indian Army and Government
of West Bengal. The 134-year-old Durand Cup football tournament will be hosted in Assam
from coming August 17 to September 4, 2022. Chief Minister Himanata Biswa Sarma
unveiled the three Durand Cup trophies, namely Durand Cup, Shimla Trophy and
President’s Cup, at Guwahati. ]
C
29. The 2024 world Athletics Cross Country Championships will be held in which
country?
1) Japan 2) Canada 3) Germany 4) Croatia 5) USA
[Explanation: The Council also confirmed that following next year’s World Athletics Cross
Country Championships in Bathurst, Australia, the next two editions will be held in Medulin
and Pula, Croatia (2024) and Tallahassee, Florida (2026). Capital: Zagreb, Currency:
Croatian kuna (HRK).]
30. India has won how many medals ISSF Shooting World Cup 2022?
1) 15 2) 5 3) 6 4) 4 5) 12
[Explanation: India finished yet another ISSF Shooting World Cup campaign by topping
the medal tally with a haul of 15 medals, including five gold, six silver and four bronze.
The Indian men’s 25m rapid fire pistol team comprising Anish Bhanwala, Vijayveer Sidhu
and Sameer won the silver medal on the last day of the ISSF World Cup Changwon 2022
in South Korea.]

JULY- 71 Visitus:www.sr eedharscce.com


Sreedhar’s C C E CURRENT AFFAIRS -JULY-2022

31. Athletics Federation of India has shifted the 61st National Open Athletics
Championshipsfrom Jamshedpur to ________.
1) Jaipur 2) Hyderabad 3) Bengaluru 4) Chennai 5) Kolkata
[Explanation: The Athletics Federation of India has shifted the 61st National Open Athletics
Championships, which were scheduled to be held in Jamshedpur Bengaluru. The AFI
came into existence in 1946 and the federation organises the National Championships,
trains the Indian Athletics National Campers. Adille J Sumariwalla is the head of Athletics
Federation of India.]
32. Who has won world gold for Kazakhstan in the women’s 3000m steeplechase?

r’s
1) Norah Jeruto 2) Werkuha Getachew 3) Ruth Jebet
4) Beatrice Chepkoech 5) Soufiane El Bakkali
[Explanation: Norah Jeruto Tanui is a Kazakhstani (formerly Kenyan) steeplechase
runner. She won the gold medal in the 3000 metres steeplechase at the 2022 World
ha
Athletics Championships. she became the first athlete representing Kazakhstan to win at
the World Championships in Eugene.]
33. Which Country will host the Asia Cup 2022?
1) Sri Lanka 2) Australia 3) UAE 4) South Africa 5) India

E
[Explanation: The 2022 edition of the Asia Cup, which was scheduled to be held in Sri
d
Lanka, has been shifted to the UAE. Sri Lanka Cricket (SLC) had informed the Asian
Cricket Council (ACC) about their inability to host the upcoming edition of the Asia Cup
T20 because of the economic and political crisis in the island nation.]
ee

34. Who among the following has become the first Indian to win a silver medal at the
World Athletics Championships?
1) Anderson Peters 2) Rohit Yadav 3) Eldhose Paul
C
4) Jakub Vadljech 5) Neeraj Chopra
Sr

[Explanation:Neeraj Chopra scripts history by becoming first Indian to win silver medal
at World Athletics Championships,Grabs 2nd position in Men’s Javelin Throw final.In the
prestigious competition held at Eugene in Oregon, United States.As world championships
meets are scheduled for Budapest in 2023 and Tokyo in 2025.]
35. Who has won the 400m women’s hurdles gold medal at World Athletics
Championships 2022?
1) Dalilah Muhammad 2) Sydney McLaughlin 3) Femke Bol
C
4) Yuliya Pechonkina 5) Shamier little
[Explanation: This is the first time a woman athlete has clocked under 51 seconds. Her
previous record stood at 51.41 seconds, and she has handsomely obliterated that. The
defending champion, Dalilah Muhammad, representing the USA,seconds and took home
the bronze medal. Femke Bol of the Netherlands took silver. She won a silver medal at the
2019 World Championships. At both the 2019 World Championships and Tokyo Games,
she also took gold as part of a women’s 4×400 meters relay team.]
36. Who has won the French Grand Prix 2022?
1) Lewis Hamilton 2) George Russell 3) Charles Leclerc
4) Valtteri Bottas 5) Max Verstappen
[Explanation:Max Verstappen won the 2022 French Grand Prix for Red Bull after Ferrari’s
Charles Leclerc crashed out of the race on Lap 18 – walking away unscathed – as the
Mercedes duo of Lewis Hamilton and George Russell completed the podium at Paul Ricard.]

JULY- 72 Visitus:www.sr eedharscce.com


Sreedhar’s C C E CURRENT AFFAIRS -JULY-2022

37. Casper Ruud has won the Swiss Open 2022. He belongs to Which country?
1) Italy 2) Norway 3) France 4) Switzerland 5) Indonesia
[Explanation: Casper Ruud is a Norwegian professional tennis player. He has won nine
ATP Tour singles titles, eight of which were on clay courts. Top seed and defending
champion Casper Ruud came from a set down to beat second seed Matteo Berrettini(Italy).
Capital: Oslo, Currency: Norwegian Krone (NOK).]
38. Who has become the tenth player to hit a century in the 100th ODI match?
1) Rohit Sharma 2) Nicholas Pooran 3) Shai Hope

r’s
4) Babar Azam 5) Darren Bravo
[Explanation: West Indies batter Shai Hope became the tenth batter to score a century in
his 100th ODI. The stylish West Indies opener achieved this accomplishment during the
second ODI against India at Queens’ Park Oval at Port of Spain. he was named the Men’s
ha
Cricketer of the Year, Test Cricketer of the Year and the ODI Cricketer of the Year at the
annual Cricket West Indies’ Awards.]
39. Who among the following has won Tour de France 2022?
1) Jonas Vingegaard
4) Alexander Kristoff

E 2) Jasper Philipsen 3) Dylan Groenewegen


5) Tom Dumoulin
d
[Explanation: Second winner Tadej Pogaèar ( SLO ),Third winner Geraint Thomas ( GBR
). Tadej Pogaèar won Tour de France 2021. Jonas Vingegaard,from Denmark. Capital:
ee

Copenhagen, Currency: Danish krone.]


40. Who has been appointed as CEO of Indian Deaf Cricket Association?
1) Roma Balwani 2) Sumit Jain 3) Ajay Kumar
C
4) Imran Sheikh 5) Mahantesh
[Explanation: Indian Deaf Cricket Association names Roma Balwani as CEO & brand
Sr

custodian. Indian Deaf Cricket Association (IDCA), the governing body for deaf cricket in
India, has named Roma Balwani as CEO and brand custodian.]
41. US has finished on top of the medal at the World Athletics Championships 2022.
What is the rank of India in medal rally?
1) 22 2) 12 3) 08 4) 33 5) 28
[Explanation: The World Athletics Championships were held in America for the first time.
C
United States finished the competition with a total of 33 medals, winning 13 gold, nine
silver and 11 bronze medals. Ethiopia won 10 medals, 2nd rank. Jamaica 10 medals, 3rd
rank. India finished joint 33rd with one silver medal.]
42. Which athlete has pulled out of CommonwealthGames 2022 due to injury?
1) SainaNehwal 2) P.V. Sindhu 3) P.T. Usha
4) Neeraj Chopra 5) Pullela Gopichand
[Explanation: The Commonwealth Games 2022 opening ceremony will be held in front
of 30,000 strong audiences at Birmingham’s iconic Alexander Stadium. Olympic gold
medallist Neeraj Chopra received the Padma Shri, the fourth-highest civilian award of
India. Neeraj had won the gold medal in men’s javelin at the last edition of the CWG in the
Gold Coast, Australia in 2018. Neeraj Chopra emerged as India’s first track-and-field
Olympic gold-medallist.]

JULY- 73 Visitus:www.sr eedharscce.com


Sreedhar’s C C E CURRENT AFFAIRS -JULY-2022
43. PV Sindhu and __________ has named as India’s flag-bearers for opening ceremony
of the Commonwealth Games 2022?
1) Manpreet Singh 2) Neeraj Chopra 3) MirabaiChanu
4) LovlinaBorgohain 5) Anju Bobby George
[Explanation: Manpreet Singh led the Indian contingent during the Tokyo Olympics. PV
Sindhu was India’s flagbearer during the 2018 Commonwealth Games.IOA had considered
Mirabai Chanu and Lovlina Borgohain as India’s CWG 2022 flagbearers.]
44. Which former Indian cricketer has been appointed to ICC Cricket Committee?
1) Sachin Tendulkar 2) V.V.S. Laxman 3) Virender Sehwag
4) Anil Kumble 5) Sourav Ganguly

r’s
[Explanation: VVS Laxman, who is also the head of the National Cricket Academy, “Daniel
Vettori and VVS Laxman have also been appointed to the International Cricket
Council(ICC)Men’s Cricket Committee as current player representatives. It is headquartered
in Dubai, UAE. There are 104 Member countries of the International Cricket Council (ICC).]
45. Which nation will host ICC Women’s ODI World Cup 2025?
1) Germany 2) Russia 3) Italy 4) India 5) Singapore
ha
[Explanation: India, Bangladesh, England and Sri Lanka were named as the four hosts
for ICC women’s white-ball events from 2024-27. The 2022 ICC Women’s Cricket World
Cup was the twelfth edition of the Women’s Cricket World Cup, which was held in New

E
Zealand. The 2023 ICC Women’s T20 World Cup, which is scheduled to be held in South
Africa. Bangladesh will be hosting the 2024 Women’s T20 World Cup while England will
d
be the 2026 edition of the same.]
46. What is the official slogan for the 2024 Paris Olympics?
1) Games Wide Open 2) Held Your Hope High 3) Move On With Games
ee

4) Let’s Make It Happen 5) Young and Dynamic


[Explanation: The President of Paris 2024 Tony Estanguet explains the slogan “Games
wide open”, which was released on Monday 25 July 2022, as part of the 2 years to go to
Paris 2024 Olympic games celebrations.]
C
47. Who has won second gold medal for India at the Commonwealth Games 2022?
1) SanketSargar 2) Jeremy Larinnunga 3) EdidongUmoafia
Sr

4) Vaipava Ione 5) Talha Talib


[Explanation: India’s Mirabai Chanu won the gold medal in the weightlifting 49kg final.
Sanket Sargar and Gururaja Poojary clinched India’s first two medals at CWG 2022. Jeremy
wins India its second gold medal.]
48. Rishi Chakraborty from _________ has won Commonwealth Games Quiz 2022.
1) Odisha 2) Bihar 3) Manipur 4) West Bengal 5) Haryana
[Explanation: The Commonwealth Games are starting from 28th of July and will continue
C
till 8th of August. The Sports Scan program of All India News has started Commonwealth
Games Quiz.]
49. Who has won the silver medal in the women’s 55 kg weightlifting at the
Commonwealth Games 2022?
1) MirabaiChanu 2) Bindyarani Devi 3) AdijatOlarinoye
4) Gururaja Poojary 5) SanketSargar
[Explanation: Bindyarani Devi won India a fourth medal on a successful day at the
Commonwealth Games weightlifting arena in Birmingham, bagging a silver medal in the
Women’s 55 kg weight class. India’s Tally At 1 Gold, 2 Silver, 1 Bronze.]
50. Who has won first Gold medal for India at Commonwealth Games 2022?
1) Bindyarani Devi 2) MirabaiChanu 3) SanketSargar
4) Gururaja Poojary 5) LovlinaBorgohain
[Explanation: Mirabai Chanu Wins Gold in Weightlifting with Personal Best and
Commonwealth Games Record. India’s weightlifting icon Mirabai Chanu claimed India’s
first gold medal at the Birmingham Commonwealth Games.]

JULY- 74 Visitus:www.sr eedharscce.com


Sreedhar’s C C E CURRENT AFFAIRS -JULY-2022

15. STATES
1. Maharashtra State Cabinet has approved to rename Aurangabad as________.
1) Shiva Nagar 2) Dharashiv 3) Sambhajinagar 4) Osmanabad 5) Ali Khan Nagar
[Explanation: The Maharashtra Cabinet passed the proposal to rename Aurangabad city
as Sambhaji Nagar Osmanabad as Dharashivin honour of the Maratha king. The decision
comes amid threat to the MVA government following a rebellion by EknathShinde-led group.
Maharashtra-Chief Minister: EknathShinde; Governor: Bhagat Singh Koshyari.]

r’s
2. Which of the following state Government is going to issue Family cards to every
family, to deal with unemployment?
1) Karnataka 2) Telangana 3) Maharashtra
4) Madhya Pradesh 5) Uttara Pradesh
[Explanation: Chief Minister Yogi Adityanath stated that these cards will be known as
ha
Family Cards and with the help of these cards the government will identify the families
who do not have any government job or any kind of employment. UP-Governor: Anandiben
Patel.]
3.

E
FMC has launched pre-emergent herbicide to support sugarcane farmers in which
state?
d
1) Gujarat 2) Bihar 3) Punjab 4) Maharashtra 5) Madhya Pradesh
[Explanation: Austral herbicide provides a new level of broad-spectrum weed control in
the critical growth stage of sugarcane, ensuring the strong establishment of the crop for a
ee

better yield. India is the second-largest producer of sugarcane in the world.]


4. which of the following state/UT has started ‘Drass Cycling Expedition Yatra’ to
Commemorate 75 years of India’s Independence?
C
1) Jammu and Kashmir 2) Gujarat 3) Delhi
4) Uttarakhand 5) Maharashtra
Sr

[Explanation: The team comprises of 20 soldiers and air warriors and will be led by two
bright lady officers of Army and Air Force. The expedition was flagged off jointly by
Lieutenant General M U Nair AVSM SM, Signal Officer-in-Chief & Senior Colonel
Commandant, Corps of Signals and Air Marshal R Radhish AVSM VM, Senior Air Staff
Officer, Western Air Command from National War Memorial, New Delhi.]
5. Which of the following state/UT has inaugurated EV rally for Single-use plastic
C
ban?
1) Goa 2) Delhi 3) Karnataka 4) Uttar Pradesh 5) Puducherry
[Explanation: CPCB is committed to facilitating the Single Use Plastic Ban of notified
items through active collaboration with key stakeholders to achieve this ambitious goal of
phasing out SUP plastics for India’s green future. Karnataka-Chief minister: Basavaraj
Bommai; Governor: Thawar Chand Gehlot.]
6. Which of the following state/UT’s has introduced ‘Deshbhakti, mindset curricula’
for students?
1) Puducherry 2) Gujarat 3) Delhi 4) Uttar Pradesh 5) Tamil Nadu
[Explanation: The Delhi government introduced new assessment guidelines for school
students. Under the new criteria, students will now be assessed for the impact of mindset
curricula in their behavior, along with other scholastic subjects. It includes Happiness and
Deshbhakti curricula for classes 3-8, while students of Class 9 and Class 11 will be
assessed for Deshbhakti and Entrepreneurship Mindset Curricula.]

JULY- 75 Visitus:www.sr eedharscce.com


Sreedhar’s C C E CURRENT AFFAIRS -JULY-2022
7. PM Modi has unveiled legendary freedom fighter Alluri Sitarama Raju’s statue in
which state?
1) Gujarat 2) Andhra Pradesh 3) Assam 4) Kerala 5) Maharashtra
[Explanation: Prime Minister Narendra Modi unveiled a bronze statue of freedom fighter
Alluri Sitarama Raju, at a special programme on his 125th birth anniversary celebrations,
at Bhimavaram in West Godavari District, Andhra Pradesh. AP- CM- Y. S. Jagan Mohan
Reddy; Governor: Biswabhusan Harichandan.]
8. Which state government has approved the draft of “Khadi Workers Economic
Incentive Scheme”?
1) Maharashtra 2) Rajasthan 3) Assam

r’s
4) Gujarat 5) Manipur
[Explanation: The state government will set up a Khadi Plaza in Jaipur. The plaza will
help to promote khadi products and facilitate weavers of the state to showcase and market
their products. Rajasthan- CM: Ashok Gehlot; Governor: Kalraj Mishra.]
9. Which state govt has announced to Free health facilities for HIV patients under
ha
PPP mode?
1) Telangana 2) Haryana 3) Assam 4) Karnataka 5) Maharashtra
[Explanation: The HIV patients have been included in seven categories of the State which
are being provided various health facilities free of cost under PPP mode. Thus, all the

E
laboratory and radiological tests, including USG, CT scan and MRI which are provided in
Government Hospitals under PPP mode will be available free of cost to HIV patients.]
d
10. Which state government is planning to open cattle hostels in all villages to ramp
up milk production?
1) Andhra Pradesh 2) Telangana 3) Karnataka
ee

4) Haryana 5) Odisha
[Explanation: The State government as part of ‘white revolution’ is planning to increase
milk production to meet the growing demand. Like sheep distribution scheme, the
C
government would open cattle hostels to increase milk production.]
11. PM Modi has inaugurated Golden Jubilee celebrations of Agradoot newspaper
Sr

group recently. Agradoot was started in which state?


1) Kerala 2) Assam 3) Haryana 4) Gujarat 5) Karnataka
[Explanation: Agradoot was started as an Assamese bi-weekly. It was established by
Kanak Sen Deka, senior journalist of Assam. In 1995, Dainik Agradoot, a daily newspaper,
was started and it has developed as a trusted and influential voice of Assam. Chief Minister:
Himanta Biswa Sarma Governor: Jagdish Mukhi.]
12. PM Modi has inaugurated and laid foundation stone of development initiatives
C
worth over Rs 1800 crores in which of the following state/UT?
1) Delhi 2) Jammu and Kashmir 3) Uttar Pradesh
4) Maharashtra 5) Gujarat
[Explanation: Prominent among the projects that were unveiled included the Rs 13.91
crore state-of-the-art Akshaya Patra Kitchen, which will prepare midday meals for one
lakh children, studying in government schools of Varanasi, in one day. PM Modi also laid
a stone for tourism-related projects worth Rs 151.68 crore, roads and bridges projects
worth Rs 765.39 crore and other projects worth Rs 303.51 crore.]
13. Prime Minister Narendra Modi will be inaugurated the Deoghar Airport, that has
been constructed in which state?
1) Uttar Pradesh 2) Gujarat 3) Jharkhand
4) Meghalaya 5) Uttarakhand
[Explanation: The airport has a 2,500-metre-long runway, which can handle the landing
and takeoff of Airbus A320 planes. It is spread across 653.75 acres of land and built in an
area of 4,000 square meters. This is only the second international airport in Jharkhand
making it one of the most important projects in the state.]

JULY- 76 Visitus:www.sr eedharscce.com


Sreedhar’s C C E CURRENT AFFAIRS -JULY-2022
14. Which state becomes first state to have 13 expressways in the nation?
1) Maharashtra 2) Manipur 3) Gujarat
4) Uttar Pradesh 5) Haryana
[Explanation:The expressway will reduce the distance between East and the West and
will be helpful for seamless people to people contacts. In addition to it, the Lucknow-
Kanpur Expressway, Bundelkhand Expressway, Gorakhpur Link Expressway,
Ghazipur-Ballia-Manjhighat Expressway and the Delhi-Dehradun Expressway are also
under construction, which will immensely benefit the people of the region.]
15. Which state has become first state in India to have Exclusive R & D Policy?
1) Rajasthan 2) Tamil Nadu 3) Karnataka

r’s
4) Chhattisgarh 5) Gujarat
[Explanation: The policy seeks to allocate at least 0.1 per cent of its Gross State Domestic
Product towards R&D and innovation. Karnataka is the first state to have an exclusive
aerospace and defence R&D policy. It also has a separate semiconductor and renewable
energy policy.]
16.
ha
In which state PM Narendra Modi has organized a Natural Farming Conclave?
1) Surat 2) Gandhinagar 3) Ahmedabad
4) Vadodara 5) Jamnagar
[Explanation:Surat district undertook a concerted effort to sensitise and motivate different

E
stakeholders and institutions like farmer groups, elected representatives, agriculture
d
produce marketing committees, cooperatives and banks to help farmers. Consequently, at
least 75 farmers were identified in each gram panchayat and were motivated and trained
to undertake natural farming.]
ee

17. Which state Chief Minister has inaugurated the state’s first gas-based tea
processing centre?
1) Rajasthan 2) Gujarat 3) Maharashtra 4) Uttar Pradesh 5) Tripura
[Explanation: The state’s first gas-based tea processing centres at Durgabari tea estate
C
in Tripura. The tea industry in Tripura is over 100 years old with Hiracherra tea estate
Sr

dating back to 1916. At present, the state has 14,000 tea workers and 2,800 small tea
growers.]
18. West Bengal has signed a new “Reciprocal Transport Agreement” with which of
the following state?
1) Arunachal Pradesh 2) Meghalaya 3) Nagaland 4) Manipur 5) Sikkim
[Explanation: Commercial vehicles running between Gangtok and Jorethang in South
Sikkim crosses a part of the route in West Bengal. Vehicles running between Rangpo
andMelli faces difficulties in case they don’t have counter signature permits. Sikkim -Chief
C
Minister: Prem Singh Tamang Governor: Ganga Prasad.]
19. Which State has set to invest Rs 300 crore in early childhood education
programmes?
1) Manipur 2) Sikkim 3) Kerala 4) Meghalaya 5) Goa
[Explanation: To revamp elementary education in Meghalaya, Chief Minister Conrad K
Sangma announced that the government has earmarked Rs 300 crore from externally aided
projects to make investment in early childhood education programmes. Meghalaya-Chief
Minister: Conrad Kongkal Sangma; Meghalaya Governor: Satya Pal Malik.]
20. Which state/UT has introduced ‘three-coach’ train network for the first time?
1) Uttar Pradesh 2) Puducherry 3) Meghalaya 4) Tamil Nadu 5) Delhi
[Explanation: The ‘three-car’ Metro train service from Narela to Rithala section. The three-
coach system shall have the provision for commensurate expansion in future, say to a six-
coach system, so that platforms could be expanded to accommodate the increased volume
of traffic in future.]

JULY- 77 Visitus:www.sr eedharscce.com


Sreedhar’s C C E CURRENT AFFAIRS -JULY-2022
21. Which state has reported India’s first confirmed monkeypox case?
1) Maharashtra 2) New Delhi 3) Kerala
4) Gujarat 5) Telangana
[Explanation: It is a viral zoonotic disease (transmission from animals to humans) and is
identified as a pox-like disease among monkeys hence it is named Monkeypox. It is endemic
to Nigeria.It is caused by monkeypox virus, a member of the Orthopoxvirus genus in the
family Poxviridae. The natural host of the virus remains undefined. But the disease has
been reported in many animals.]
22. Which among the following state has set up climate change missions in all

r’s
districts?
1) Tamil Nadu 2) Kerala 3) New Delhi 4) Assam 5) Manipur
[Explanation: District Missions is part of the larger agenda that the government is taking
a 360 degrees approach in order to create a climate resilient Tamil Nadu. Government has
accorded Rs 3.80 crore towards district level mission activities.]
ha
23. Which state government has launched MGNREGS in census towns?
1) Meghalaya 2)New Delhi 3) Tripura 4) Nagaland 5) Mizoram
[Explanation: Meghalaya is the first states to have women reservation of 15 per cent in
the committees of the MGNREGS and it has close to 2500 women secretaries in different

E
VECs and that reflects the equality and gender participation on important decisions that
are made at the grassroots level.]
d
24. Which state has become first state to implement NEP at pre-primary level?
1) Uttar Pradesh 2) Jharkhand 3) Madya Pradesh
ee

4) Uttarakhand 5) New Delhi


[Explanation: It offers children an opportunity to pursue choice-based education. They
can now study the subjects of their choice in the language they are conversant with.
Uttarakhand CM: Shri Pushkar Singh Dhami Governor: Gurmit Singh.]
C
25. Which among the following government has announced project with UNICEF for
job opportunities to students?
Sr

1) New Delhi 2) Meghalaya 3) Gujarat 4) Telangana 5) Goa


[Explanation: DSEU and UNICEF have introduced ‘Career Awareness Sessions’ for
students. Skill Varsity of Delhi joins hands with YuWaah (Generation Unlimited India) at
UNICEF to enable access to employment opportunities, assist students in becoming job
ready as well as listen to and amplify the voices of young people.]
26. Union cabinet has approved Gati Shakti University in which among the following
state?
C
1) Rajasthan 2) Gujarat 3) Tamil Nadu
4) Jammu and Kashmir 5) Panjab
[Explanation: The Gati Shakti scheme will subsume the Rs 110 lakh crore National
Infrastructure Pipeline that was launched in 2019. It will help in fulfilling the ambitious
targets set by the government for 2024-25, including expanding the length of the national
highway network to 2 lakh kms, creation of more than 200 new airports, heliports and
water aerodromes.]
27. Which of the following state CMs has agreed to realign border based on 1960
papers?
1) Assam 2) Meghalaya 3) Arunachal Pradesh
4) Both 1&2 5) Both 1&3
[Explanation: Meghalaya was carved out of Assam in 1972, which is when the dispute
arose. As a result of different readings of the demarcation of boundaries in the initial
agreement for Meghalaya’s creation, border issues were created.]

JULY- 78 Visitus:www.sr eedharscce.com


Sreedhar’s C C E CURRENT AFFAIRS -JULY-2022
28. Which among the following state will be malaria-free state by 2025?
1) Telangana 2) Karnataka 3) Tripura 4) Odisha 5) Bihar
[Explanation: Malaria is a life-threatening mosquito borne blood disease caused by
plasmodium parasites.It is predominantly found in the tropical and subtropical areas of
Africa, South America as well as Asia.The parasites spread through the bites of infected
female Anopheles mosquitoes.]
29. Which state has become the first state in the country to have its own internet
service?
1) Gujarat 2) Tamil Nadu 3) Karnataka 4) Kerala 5) Arunachal Pradesh

r’s
[Explanation: The Department of Telecommunications’ granting of an ISP licence to the IT
infrastructure project of the Kerala Fiber Optic Network Ltd (K-Fon), which aims to provide
everyone in the state access to the internet.The principle behind the establishment of K-
Fon is “non -discriminatory” treatment, which means that no service provider or business
segment receives preferential treatment, as recommended by the Telecom Regulatory
ha
Authority of India (TRAI).]
30. Which state Govt. has launched the e-FIR service and a police app?
1) Tamil Nadu 2) Andhra Pradesh 3) Uttarakhand
4) Jharkhand 5) Gujarat

E
[Explanation: The launch of the e-FIR facility will help people file an online FIR in cases
related to the theft among others. All online services provided by the State police to the
d
public have been integrated in the Uttarakhand police app. These include Gaura Shakti,
Traffic Eye, Public Eye, Meri Yatra and Lakshya Nasha Mukt Uttarakhand.]
ee

31. Which State has Achieved the 100% Saturation of Grey Water Management?
1) Goa 2) Tamil Nadu 3) Kerala 4) Odisha 5) Jharkhand
[Explanation: ‘100% Saturation of Grey Water Management in Pappankuzhi Village, Tamil
Nadu. The system which includes individual household soak pits and community soak
C
pits effectively treats about 42,000 liters of greywater that is generated per day in the
village.]
Sr

32. In which state PM Modi has announced the ex-gratia from PMNRF for the victims
of bus tragedy?
1) Gujarat 2) Uttar Pradesh 3) Madhya Pradesh
4) Haryana 5) Maharashtra
[Explanation: An ex-gratia of Rs. 2 lakh each from PMNRF would be given to the next of
kin of those who lost their lives due to the bus tragedy in Dhar, Madhya Pradesh. The
injured would be given Rs. 50,000. Madhya Pradesh-CM: Shivraj Singh Chouhan
C
Governor: Mangubhai 3) Patel.]
33. Which of the following state eleven districts has reported cases of ‘Kala Azar’?
1) Kerala 2) Odisha 3) West Bengal 4) Nagaland 5) Mizoram
[Explanation: Leishmaniasis (Kala Azar), It is a neglected tropical disease affecting almost
100 countries including India. Neglected tropical diseases are a diverse group of
communicable diseases that prevail in tropical and subtropical conditions in 149 countries.
It is caused by a parasite called Leishmania, which is transmitted through the bite of sand
flies.]
34. Which state Jal Board has developed AI-run technology to clean sewage water?
1) Goa 2) Odisha 3) Delhi 4) Punjab 5) Haryana
[Explanation: This technology, called ISASMA-CD (Intelligence Self-Administered Self
Monitored Automatic Chemical Dosing), has been introduced at four DJB plants at Yamuna
Vihar and Okhla with the objective of reducing BOD (Biological Oxygen Demand) and TSS
(Total Suspended Solids) to less than 10 PPM (parts per million).]

JULY- 79 Visitus:www.sr eedharscce.com


Sreedhar’s C C E CURRENT AFFAIRS -JULY-2022
35. Which state government has announced DiCRA in partnership with UNDP?
1) Bihar 2) Haryana 3) Telangana 4) Tamil Nadu 5) Assam
[Explanation: The platform, which uses artificial intelligence to power it, aims to improve
food security and food systems. DiCRA becoming a digital public good, according to IT
Minister KT Rama Rao, is a significant step in our commitment to open data policy, service
delivery to farmers, and anticipatory governance to address the world problem of food
security.]
36. Which state Govt. has launched India’s 1st AI powered Lok Adalat for fast disposal
of cases?
1) Gujarat 2) Rajasthan 3) West Bengal 4) Manipur 5) Sikkim

r’s
[Explanation: India’s increasing litigation backlog has recently made attention, particularly
during the pandemic when courts were temporarily halted. The oldest pending case in
the nation was recently resolved by a district court in Bihar after 108 years of deliberation.
According to the survey of Niti Aayog, between 5 million and 40 million legal issues arise
each month but only between 75% and 97% of them end up in court.]
ha
37. Which state has become the First state in country to connect all registered
commercial vehicles equipped with VLTD with ERSS?
1) Goa 2) Kerala 3) Himachal Pradesh
4) Telangana 5) Gujarat

E
[Explanation: Through this mechanism, these vehicles can be tracked anywhere in India.
Over 9,423 vehicles have been registered and connected with ERSS. Under this
d
mechanism, both Police and transport department can now monitor the vehicles. This
system was launched by Chief Minister Jai Ram Thakur at Peterhof, in Shimla.]
38. Which state/UT has celebrated ‘Happiness Utsav’ to honour the 4th anniversary
ee

of its Happiness Curriculum for schools?


1) Kerala 2) Manipur 3) Ladakh 4) Chandigarh 5) Delhi
[Explanation: Each student in a government school in Delhi would interact with at least
C
five members of their local community and impart happiness-related knowledge on them.
The objective is to assist Delhi’s millions of citizens in finding happiness.Students’ mindsets
Sr

have undergone a revolutionary shift as a result of Happiness Classes.]


39. Which state has introduced ‘Swanirbhar Naari ‘Scheme for supporting the
indigenous weavers?
1) Arunachal Pradesh 2) Telangana 3) Assam
4) Manipur 5) Kerala
[Explanation: The prime objective of the Swanirbhar Nari Atmanirbhar scheme is to
empower financially disadvantaged women. Under the Swanirbhar Nari Atmanirbhar
C
scheme, four lakh families are given benefits in the first phase. The Government has created
more than 3.72 lakh permanent individual and 822 community properties under this
scheme.]
40. Which state has topped in the NITI Aayog’s India Innovation Index 2021 in ‘Major
States’ category?
1) Gujarat 2) Odisha 3) West Bengal 4) Karnataka 5) Uttar Pradesh
[Explanation: Karnataka topped in the ‘Major States’ category and Manipur emerged as
a winner in the ‘North East and Hill States’ category. Chandigarh was the top performer in
the ‘Union Territories and City States’ category. Karnataka’s high score can be attributed
to its peak performance in attracting FDI and a large number of venture capital deals.]
41. In which of the following UT, National Investment and Infrastructure Fund (NIIF)
has acquired Navayuga Quazigund Expressway for Rs 3,036 crore?
1) Ladakh 2) Chandigarh 3) Delhi 4) Jammu Kashmir5) Puducherry
[Explanation: The road provides a vital link between the two important cities in the region,
Jammu and Srinagar, enabling seamless movement of traffic and goods, with enhanced
inter-region connectivity.]

JULY- 80 Visitus:www.sr eedharscce.com


Sreedhar’s C C E CURRENT AFFAIRS -JULY-2022
42. Which State has topped in the Smart City Fund utilization?
1) Uttar Pradesh 2) Karnataka 3) Andhra Pradesh
4) Gujarat 5) Tamil Nadu
[Explanation: While Tamil Nadu has spent over Rs 3932 crore out of the Rs 4333 crore
released by the Centre, Uttar Pradesh is in the second position with the utilization of Rs
2699 cr out of the Central share release of Rs 3142 crore. As on 8 July 2022, the Centre
has released Rs 30,751.41 crore for 100 Smart Cities, out of which Rs 27,610.34 crore
(90%) has been utilized.]
43. Which UT has topped the NITI Aayog’s India Innovation Index 2022 among UT’s?
1) Delhi 2) Puducherry 3) Ladakh 4) Chandigarh 5) Jammu Kashmir

r’s
[Explanation: Karnataka topped in the ‘Major States’ category and Manipur emerged as
a winner in the ‘North East and Hill States’ category. Chandigarh was the top performer in
the ‘Union Territories and City States’ category. Karnataka’s high score can be attributed
to its peak performance in attracting FDI and a large number of venture capital deals.]
44. Which state has inaugurated the ‘Youth Model Assembly programme’ as part of
ha
Azadi Ka Amrit Mahotsav?
1) Meghalaya 2) Gujarat 3) Assam 4) Uttar Pradesh 5) Arunachal Pradesh
[Explanation: Azadi Ka Amrit Mahotsav is an initiative of the Government of India to
celebrate and commemorate 75 years of independence and the glorious history of it’s

45.

E
people, culture and achievements.]
Which state Govt has started distribution of foodgrains under PMGKAY?
d
1) Telangana 2) Maharashtra 3) Gujarat 4) Kerala 5) Madhya Pradesh
[Explanation: The free food grain scheme or the Pradhan Mantri Garib Kalyan Anna
Yojana (PMGKAY) was introduced by the Narendra Modi government during the first
ee

nationwide lockdown due to Covid-19 in March 2020.Under this scheme, the center
provides 5kg of free food grains per month to the poor.]
46. Which State govt. has joined hands with IFC, a member of World bank group for
C
implementation of asset monetization?
1) Rajasthan 2) Uttarakhand 3) Odisha 4) Meghalaya 5) Gujarat
Sr

[Explanation: The programme will facilitate and accelerate the monetisation process in
the State by unlocking the value from large scale public investment in brownfield
infrastructure assets and bringing in private sector efficiencies. This aims to attract large
scale public investment and bring in private sector efficiencies]
47. Which state has approved its new Employment Policy?
1) Gujarat 2) Madhya Pradesh 3) Karnataka
4) Telangana 5) Maharashtra
C
[Explanation: The vision is to ensure ample availability of the relevant workforce across
Karnataka and provide job opportunities to a larger base of the State’s workforce to reduce
unemployment in Karnataka.]
48. Which State govt. has got Industry tag for tourism units?
1) Gujarat 2) Rajasthan 3) Assam
4) Manipur 5) Uttar Pradesh
[Explanation:Earlier this year has given a sure and certain fillip to the State’s tourism
units that had incurred huge losses during the pandemic. The power tariff and other taxes
levied on tourism operators have now been brought on par with other industries, against
the much-higher commercial rates levied earlier. Chief Minister: Ashok Gehlot, Governor:
Kalraj Mishra.]
49. which state govt. has launched states tourism policy 2022?
1) Assam 2) Gujarat 3) Manipur 4) Jharkhand 5) Maharashtra
[Explanation: The policy aimed at attracting tourists to the state would also lead to
employment generation, improvement in happiness quotient and achieving sustainable
development. Chief minister Hemant Soren; Governor: Shri Ramesh Bais.]

JULY- 81 Visitus:www.sr eedharscce.com


Sreedhar’s C C E CURRENT AFFAIRS -JULY-2022
50. Which state govt has approved creation of second wildlife sanctuary in Barak
Valley?
1) Maharashtra 2) Telangana 3) Bihar 4) Assam 5) Uttarakhand
[Explanation: Barak Valley is home to several kinds of plants and animals. The new
Barak Bhuban Wildlife Sanctuary will be opened between the Barak River and the Sonai
River. The new wildlife sanctuary will be spread over an area of 320 sq. km.]
51. Droupadi Murmu had served as the 9th Governor of which state between 2015
and 2021?
1) Jharkhand 2) Maharashtra 3) Odisha 4) Gujarat 5) Meghalaya
[Explanation: She was the first candidate to be nominated for the post of President of

r’s
India who belongs to a scheduled tribe. She is the first person of tribal origin and only the
second woman after Pratibha Patil to serve as the President of India.]
52. Which state government has recently signed MoU between Flipkart and Skill
Development Mission?
1) Meghalaya 2) Gujarat 3) Maharashtra 4) Bihar 5) Tamil Nadu
ha
[Explanation:The programme aims to develop a talent pool of skilled supply chain
operations personnel and disseminate pertinent training and expertise of the business.
This will contribute to closing the skills gap and opening up employment chances in the
nation’s booming supply chain sector.]
53.

E
Which state has signed a MoU with North East Space Application Center?
d
1) Manipur 2) Nagaland 3) Kohima 4) Tripura 5) Sikkim
[Explanation: Early warning system is important to mitigate any disaster. The state will
work together with NESAC to develop an early warning system so that the state could
ee

predict any disaster well before it takes place.]


54. ADB has approved $96 mn loan for safe drinking water project in ___________.
1) Assam 2) Telangana 3) Himachal Pradesh
4) Uttarakhand 5) Madya Pradesh
C
[Explanation: The Manila-headquartered funding agency announced the project will
Sr

improve water supply and sanitation services through, among others, the construction of
48 groundwater wells, 80 surface water intake facilities, 109 watertreatment plants, 117
pumping stations, and 3,000 kilometers of water distribution pipelines.]
55. _____ has signed MoU with IOA and State Olympic Association for 36th National
Games.
1) New Delhi 2) Panjab 3) Haryana 4) Gujarat 5) Manipur
[Explanation: The 36th National Games, Gujarat 2022 is scheduled between September
27- October 10 in six cities of Gujarat with all the 28 states and eight Union Territories are
C
set to participate. There will be a total of 36 sports on the roster.]
56. Which state government has launched” Family Doctor Project”?
1) Andhra Pradesh 2) Tamil Nadu 3) Gujarat
4) Maharashtra 5) Uttar Pradesh
[Explanation: As a part of this project, a doctor from Primary Health Centre (PHC) will be
made available to look after health needs of people in Ward and Village Sachivalayam.
Before Doctor’s visit, ANMs, ASHA workers and Mid-Level Health Providers (MLHPs) will
visit door-to-door and identify people who require doctor’s services.]
57. Which state government has launched the “Smart E-Beat”system for police
attendance?
1) Haryana 2) Bihar 3) Madhya Pradesh 4) Gujarat 5) Karnataka
[Explanation: The app-based system has been introduced in Gurugram under the Smart
Policing Initiative (SPI) and it will help these policemen to mark their attendance and in the
monitoring of their rides.]

JULY- 82 Visitus:www.sr eedharscce.com


Sreedhar’s C C E CURRENT AFFAIRS -JULY-2022
58. Lok Sabha has passed bill to validate family courts in Nagaland and ____________.
1) Himachal Pradesh 2) Assam 3) Madya Pradesh
4) Uttarakhand 5) Sikkim
[Explanation: It allows state governments to establish family courts. The Himachal Pradesh
and Nagaland governments have set up family courts under the Act. However, the central
government had not extended the application of the Act to these states.]
59. Which of the following states have adopted The Model Tenancy Act?
1) Andhra Pradesh 2) Tamil Nadu 3) Uttar Pradesh
4) Assam 5) All the above

r’s
[Explanation: The Act aims to regularize the rental landscape of India. It will ensure a
transparent ecosystem for renting premises, reduce disputes between tenants and
landlords and minimize litigations. Maharashtra has the highest number of tenants, but
the state government has decided not to follow the MTA.]
60. Which state has signed MOU with Rajasthan to set up first all women-run
ha
cooperative banks in Rajasthan?
1) Gujarat 2) Madhya Pradesh 3) Karnataka
4) Uttar Pradesh 5) Telangana
[Explanation: Mahila Nidhi will strengthen the women’s self-help groups in the State by

E
extending timely loan facilities for their enterprises. The institution will be operated entirely
by women and will act as a complementary body to the formal banking system. It is also
d
expected to bring down the number of loan applications pending in the banks.]
61. Which state govt. has decided to launch measures to achieve complete digital
ee

literacy in the State?


1) Tripura 2) Mizoram 3) Goa 4) Kerala 5) Andhra Pradesh
[Explanation: Since smartphones have become part of everyone’s daily life and it has
become a significant tool for online education in the post-COVID-19 period, children cannot
C
be kept away from digital space forever but their safety and security should be ensured
through awareness.]
Sr

62. Which city has launched India’s Incredible India Kargil Marathon and Tourism
Festival, 2022?
1) Kolkata 2) Bengaluru 3) Kargil 4) Jaipur 5) Mumbai
[Explanation: To promote and present the tradition and culture of Kargil district to
outsiders, it was decided that wide range of cultural programs will be organized. Traditional
food stalls will also be installed along with the display of traditional sports including horse
polo and Archery sports for which the ADC directed concerned officers to make all
C
necessary arrangements.]
63. Which state has planned to give cash incentive of Rs 2,500 per acre to farmers for
not burning stubble?
1) Haryana 2) Manipur 3) Tripura 4) Punjab 5) Goa
[Explanation: the Punjab Government has proposed that it will give Rs 500, Delhi
Government will give Rs 500 and the Central Government can give Rs 1.5 thousand.
Whenever the Air Quality Commission will take decision, the Delhi Government will
implement the same.]
64. Which state govt. has launched ‘Savari’, an online e-taxi service?
1) Karnataka 2) Telangana 3) Kerala 4) Gujarat 5) Rajasthan
[Explanation: App is a very safe system for children and women to use. The app is
designed with utmost care in security standards. The app also has a panic button system
that can be pressed in case of a car accident or any other type of danger. One can do it
privately.]

JULY- 83 Visitus:www.sr eedharscce.com


Sreedhar’s C C E CURRENT AFFAIRS -JULY-2022

16. SURVEYS
1. According to the data released by AdEx India, which of the following has emerged
as the most advertised brand across sectors in Jan-Mar 2022?
1) AMFI 2) LIC 3) HDFC Life 4) SBI Life 5) Visa Card
[Explanation: The Banking Finance Investments (BFSI) sector saw a 7% growth in ad
volumes on Television. LICHeadquarters: Mumbai, India, Chairman M.R. Kumar, Founded:
1 September 1956.]
2. According to Knight Frank India’s Affordability Index for H1 2022, which of the

r’s
following has the most affordable housing market?
1) Pune 2) Ahmedabad 3) Chennai 4) Kolkata 5) Bengaluru
[Explanation: Mumbai remains the most expensive residential market in the country. In
H1 2022, the affordability index of the city increased to 56 per cent from 53 percent in
2021. After Ahmedabad, Pune and Chennai are the second most affordable residential

3.
ha
markets in the country, with a ratio of 26 per cent each in the H1 of2022.]
Which of the following cityhas emerged as the best city for students in ‘QS Best
Student Cities Ranking 2023’?
1) London 2) Seoul 3) Munich 4) Zurich 5) Melbourne

E
[Explanation: This report is based on factors like affordability and desirability. According
to the list, no Indian city has made it to the top 100. Mumbai bagged the 103rd rank, to
d
emerge as India’s highest-ranked student city for affordability but it struggled on metrics
for student mix and desirability.]
ee

4. Which of the following has been recognized as one of the “Most Preferred Workplaces
of 2022”?
1) ONGC 2) NTPC 3) HAL 4) IOCL 5) BHEL
[Explanation: Organizations have been appraised on the parameters such as Employee
C
Centricity, Organizational Purpose, Intrapreneurial Ecosystem, Work Flexibility, Diversity
and Equality, Growth and Rewards, and Social Cohesion.]
Sr

5. Which state has topped the ranking of States for implementation of the National
Food Security Act (NFSA)?
1) Assam 2) Odisha 3) Karnataka 4) Telangana 5) Haryana
[Explanation: Odisha has topped the ranking followed by Uttar Pradesh and Andhra
Pradesh. Among the special category states (the North Eastern states, Himalayan states,
and the Island states) Tripura has obtained the first rank. Himachal Pradesh and Sikkim
are at the second and third positions.]
C
6. The “State Ranking Index for NFSA “2022 was released by ______________.
1) Rajnath Singh 2) Amit Shah 3) Piyush Goyal
4) Nirmala Sitharaman 5) Ramesh Pokhiryal
[Explanation: Odisha has topped the ranking followed by Uttar Pradesh and Andhra
Pradesh. Among the special category states (the North Eastern states, Himalayan states,
and the Island states) Tripura has obtained the first rank. Himachal Pradesh and Sikkim
are at the second and third positions.]
7. Recently which states have been ranked as the “Best Performer” in the Start Up
Ranking 2021?
1) Gujarat 2) Karnataka 3) Meghalaya 4) A &C 5) All of the Above
[Explanation: The Department for Promotion of Industry and Internal Trade (DPIIT)
organised the third edition of the States Startup Ranking 2021.Piyush Goyal stated that
thecountry has emerged as the third-largest startup ecosystem globally over the lastfive
years, as in Financial Year (FY) 2017, only 726 startups were recognised by
thegovernment.]

JULY- 84 Visitus:www.sr eedharscce.com


Sreedhar’s C C E CURRENT AFFAIRS -JULY-2022
8. According to The NIRF Rankings 2022which is the best educational institute in
India?
1) IIT Madras 2) IIT Delhi 3) IIT Bombay 4) IIT Kanpur 5) IIT Guwahati
[Explanation: The top three slots in the ‘Overall’ category remain unchanged since last
year. IIM Ahmedabad topped in the ‘Management’ category while IISc Bengaluru topped
in the ‘Universities’ category.AIIMS Delhi is the best medical college in India while National
Law School of India University, Bengaluru, is the top law school in the country. Miranda
House topped in the ‘Colleges’ category.]
9. What is the rank of India in Global Gender Gap Index 2022?
1) 121 2) 135 3) 90 4) 110 5) 81
[Explanation: India’s overall score has improved from 0.625 (in 2021) to 0.629, which is

r’s
its seventh-highest score in the last 16 years. In 2021, India was ranked 140 out of 156
countries. Iceland has retained its position as the world’s most gender-equal country,
among 146 nations on the index. Finland, Norway, New Zealand and Sweden are the top
five countries on the list respectively. Afghanistan is the worst-performing country in the
report.] ha
10. According to UN projection World population to touch___________ billion by 2022.
1) 10 2) 15 3) 8 4) 9 5) 12
[Explanation: The global population is expected to grow to around 8.5 billion in 2030,
9.7 billion in 2050 and 10.4 billion in 2100.In 2020, the global growth rate fell under 1%

E
per year for the first time since 1950.The share of the global population aged 65 years or
above is projected to rise from 10% in 2022 to 16% in 2050.]
d
11. What is India’s rank in the PHDCCI International Economic Resilience (IER)?
1) 2nd 2) 1st 3) 3rd 4) 4th 5) 5th
[Explanation: The ranking has been topped by Germany, among the top-10 leading
ee

economies. South Korea is at the third position in the list.The ranking is based on analysis
of five indicators they are Real GDP growth rate,Merchandise export growth rate,Current
account balance (as a percentage of GDP),General government net lending/borrowing (as
a percentage of GDP) andGross debt-to-GDP ratio.]
C
12. According to the Report of WHO & UNICEF, the number of children in India who
were unvaccinated the first dose of DTP vaccine, rising from 1.4 million in 2019 to
Sr

_____in 2021.
1) 2.7 million 2) 3.7 million 3) 2.8 million 4) 2.4 million 5) 2.5 million
[Explanation: The percentage of children who received three doses of the vaccine against
diphtheria, tetanus and pertussis (DTP3) — a marker for immunisation coverage within
and across countries — fell five percentage points between 2019 and 2021 to 81%.]
13. According to UNDP report Inflation has pushed_________ million people into poverty
since Ukraine war?
1) 71 million 2) 65 million 3) 91 million 4) 80 million 5) 75 million
C
[Explanation: The UNDP noted that 125 million people experienced poverty over about
18 months during the pandemic’s lockdowns and closures, compared with more than 71
million in just three months after Russia’s invasion of Ukraine in late February.]
14. What is the rank of India in Expat Insider Rankings for 2022?
1) 52 2) 36 3) 48 4) 35 5) 27
[Explanation: Mexico has topped the Expat Insider Rankings for 2022, which was
released by the InterNations recently, while India has been ranked on 36th position out of
the 52 countries on the list, with a high affordability score. Kuwait is the worst country for
expats in the ranking.]
15. According to the RBI’s 5th round of survey on remittances for FY21, India gets the
highestremittances from which country?
1) Singapore 2) UAE 3) USA 4) Russia 5) Mexico
[Explanation: The share of the traditional remittance recipient states of Kerala, Tamil
Nadu and Karnataka, which had strong dominance in the Gulf countries, has almost halved
in 2020-21, accounting for only 25% of total remittances compared to over 42% in 2016-
17, while Maharashtra has emerged as the top recipient state surpassing Kerala.]

JULY- 85 Visitus:www.sr eedharscce.com


Sreedhar’s C C E CURRENT AFFAIRS -JULY-2022
16. What is the rank of India in Henley Passport Index 2022?
1) 84 2) 90 3) 86 4) 87 5) 78
[Explanation: Three Asian nations Japan, Singapore and South Korea bagged the top
three positions on the list, reversing pre-pandemic rankings that were dominated by
European nations. Among other countries from Asia, India, along with Mauritius and
Tajikistan, has been ranked 87th, with its passport providing access to 67 countries.]
17. What is the rank of India in Ookla’s Speedtest Global Index in June 2022?
1) 115 2) 135 3) 118 4) 120 5) 106
[Explanation: India’s rankings fell by three spots in the median mobile speeds. India
has at the number 115th spot in May this year, but it dropped down to 118th in June. In

r’s
April and May, India’s mobile broadband speeds improved. But the month of June 2022,
saw the median mobile download speeds in India decrease from 14.28 Mbps in May to
14.00 Mbps, according to the data.]
18. According to World Health Organization (WHO) report, Which Country was the top
remittance recipient in 2021, with $87 billion?
1) India 2) China
ha 3) Mexico 4) Philippines 5) Egypt
[Explanation: China ($53 billion), Mexico ($53 billion), the Philippines ($36 billion) and
Egypt ($33 billion) were the top five nations in the list.The United States was the largest
source country for remittances in 2020, followed by the United Arab Emirates, Saudi

E
Arabia and Switzerland. Remittances increase or maintain consumer spending and soften
the blow of economic hardship, such as during the COVID-19 pandemic.]
d
19. According to the NITI Aayog report, which Country has the lowest R&D
expendituresworldwide?
ee

1) Brazil 2) Russia 3) China 4) South Africa 5) India


[Explanation: The biggest R&D expenditure per GDP is made by Israel, which is 4.5
percent.The United States, Sweden, and Switzerland are developed nations that spend
roughly 2.9 percent, 3.2 percent, and 3.4 percent, respectively.]
C
20. According to NitiAayog report India’s battery storage potential to be 600 GWh
by__________.
Sr

1) 2050 2) 2030 3) 2070 4) 2025 5) 2024


[Explanation: By 2030, the demand for batteries is anticipated to develop 4 folds to
succeed in an annual price of three,100 GWh, it stated, including this exhibits a development
of 16 per cent CAGR by way of 2020–2030.]
21. IDEX- Defence Innovation Organization (DIO) has signed ______ Contract for
Defence Innovation.
1) 50 th 2) 25 th 3) 100 th 4) 75 th 5) 150 th
C
[Explanation: iDEX is funded and managed by the “Defence Innovation Organization”
(DIO), which is a ‘not for profit’ company as per Section 8 of the Companies Act 2013. Its
founder members are the two biggest defence public sector undertakings (DPSUs):
Hindustan Aeronautics Ltd (HAL) and Bharat Electronics Ltd (BEL).]
22. Which airport has emerged as 13th busiest globally in the world airport traffic
dataset 2021?
1) Hartsfield-Jackson Atlanta International Airport
2) Dallas Forth Worth and Denver
3) Indira Gandhi International Airport
4) Istanbul International Airport
5) Chhatrapati Shivaji International Airport
[Explanation: Hartsfield-Jackson Atlanta International Airport (ATL) has topped the 2021
Ranking with 7.6 crore passengers in terms of passenger footfall in the year 2021.IGIA
witnessed 3.7 crore passenger footfall in 2021, which is 30.3% more than 2.8 crore in
2020 when IGI was ranked 16th.]

JULY- 86 Visitus:www.sr eedharscce.com

You might also like